You are on page 1of 167

FUNDAMENTAL POWERS OF THE STATE

POLICE POWER

Philippine Association of Service Exporters Vs. DRILON


163 SCRA 386; L-81958; 30 JUN 1988

Facts:
Petitioner, Phil association of Service Exporters, Inc., is engaged principally in the recruitment of Filipino workers, male
and female of overseas employment. It challenges the constitutional validity of Dept. Order No. 1 (1998) of DOLE
entitled “Guidelines Governing the Temporary Suspension of Deployment of Filipino Domestic and Household Workers.”
It claims that such order is a discrimination against males and females. The Order does not apply to all Filipino workers
but only to domestic helpers and females with similar skills, and that it is in violation of the right to travel, it also being
an invalid exercise of the lawmaking power. Further, PASEI invokes Sec 3 of Art 13 of the Constitution, providing for
worker participation in policy and decision-making processes affecting their rights and benefits as may be provided by
law. Thereafter the Solicitor General on behalf of DOLE submitting to the validity of the challenged guidelines involving
the police power of the State and informed the court that the respondent have lifted the deployment ban in some states
where there exists bilateral agreement with the Philippines and existing mechanism providing for sufficient safeguards
to ensure the welfare and protection of the Filipino workers.

Issue:
Whether or not there has been a valid classification in the challenged Department Order No. 1.

Held:
SC in dismissing the petition ruled that there has been valid classification, the Filipino female domestics working abroad
were in a class by themselves, because of the special risk to which their class was exposed. There is no question that
Order No.1 applies only to female contract workers but it does not thereby make an undue discrimination between
sexes. It is well settled hat equality before the law under the constitution does not import a perfect identity of rights
among all men and women. It admits of classification, provided that:

1. Such classification rests on substantial distinctions


2. That they are germane to the purpose of the law
3. They are not confined to existing conditions
4. They apply equally to al members of the same class

In the case at bar, the classifications made, rest on substantial distinctions.

Dept. Order No. 1 does not impair the right to travel. The consequence of the deployment ban has on the right to travel
does not impair the right, as the right to travel is subjects among other things, to the requirements of “public safety” as
may be provided by law. Deployment ban of female domestic helper is a valid exercise of police power. Police power as
been defined as the state authority to enact legislation that may interfere with personal liberty or property in order to
promote general welfare. Neither is there merit in the contention that Department Order No. 1 constitutes an invalid
exercise of legislative power as the labor code vest the DOLE with rule making powers.
ICHONG VS. HERNANDEZ
101 Phil 1117; G.R. No. L-7995; 31 May 1957

Facts:
Republic Act 1180 or commonly known as “An Act to Regulate the Retail Business” was passed. The said law provides for
a prohibition against foreigners as well as corporations owned by foreigners from engaging from retail trade in our
country. This was protested by the petitioner in this case. According to him, the said law violates the international and
treaty of the Philippines therefore it is unconstitutional. Specifically, the Treaty of Amity between the Philippines and
China was violated according to him.

Issue:
Whether or Not Republic Act 1180 is a valid exercise of police power.

Held:
According to the Court, RA 1180 is a valid exercise of police power. It was also then provided that police power can not
be bargained away through the medium of a treaty or a contract. The Court also provided that RA 1180 was enacted to
remedy a real and actual danger to national economy posed by alien dominance and control. If ever the law infringes
upon the said treaty, the latter is always subject to qualification or amendment by a subsequent law and the same may
never curtain or restrict the scope of the police power of the state.
LUTZ VS. ARANETA
GR L-7859, 22 December 1955

Facts:
A Walter Lutz, as Judicial Administrator of the Intestate Estate of Antonio JaymeLedesma, sought to recover the sum of
P14,666.40 paid by the estate as taxes from theCommissioner under Section E of Commonwealth Act 567 (the Sugar
Adjustment Act),alleging that such tax is unconstitutional as it levied for the aid and support of the sugarindustry
exclusively, which is in his opinion not a public purpose.

Issue:
Whether the tax is valid in supporting an industry.

Held:
The tax is levied with a regulatory purpose, i.e. to provide means for the rehabilitationand stabilization of the threatened
sugar industry. The act is primarily an exercise of policepower, and is not a pure exercise of taxing power. As sugar
production is one of the greatindustries of the Philippines; and that its promotion, protection and advancement
redoundsgreatly to the general welfare, the legislature found that the general welfare demanded thatthe industry
should be stabilized, and provided that the distribution of benefits therefrom bereadjusted among its component to
enable it to resist the added strain of the increase in taxthat it had to sustain. Further, it cannot be said that the
devotion of tax money toexperimental stations to seek increase of efficiency in sugar production, utilization of by-
products, etc., as well as to the improvement of living and working conditions in sugar mills and plantations, without any
part of such money being channeled directly to private persons,constitute expenditure of tax money for private
purposes. The tax is valid.
ASSO. OF SMALL LANDOWNERS VS. SEC. OF DAR
175 SCRA 343; G.R. NO. L-78742; 14 JUL 1989

Facts:
Several petitions are the root of the case:
a. A petition alleging the constitutionality of PD No. 27, EO 228 and 229 and RA 6657. Subjects of the petition are a
9-hectare and 5 hectare Riceland worked by four tenants. Tenants were declared full owners by EO 228 as
qualified farmers under PD 27. The petitioners now contend that President Aquino usurped the legislature’s
power.
b. A petition by landowners and sugarplanters in Victoria’s Mill Negros Occidental against Proclamation 131 and EO
229. Proclamation 131 is the creation of Agrarian Reform Fund with initial fund of P50Billion.
c. A petition by owners of land which was placed by the DAR under the coverage of Operation Land Transfer.
d. A petition invoking the right of retention under PD 27 to owners of rice and corn lands not exceeding seven
hectares.

Issue:
Whether or Not the aforementioned EO’s, PD, and RA were constitutional.

Held:
The promulgation of PD 27 by President Marcos was valid in exercise of Police power and eminent domain.
The power of President Aquino to promulgate Proc. 131 and EO 228 and 229 was authorized under Sec. 6 of the
Transitory Provisions of the 1987 Constitution. Therefore it is a valid exercise of Police Power and Eminent Domain.
RA 6657 is likewise valid. The carrying out of the regulation under CARP becomes necessary to deprive owners of
whatever lands they may own in excess of the maximum area allowed, there is definitely a taking under the power of
eminent domain for which payment of just compensation is imperative. The taking contemplated is not a mere
limitation of the use of the land. What is required is the surrender of the title and the physical possession of said excess
and all beneficial rights accruing to the owner in favour of the farmer.
A statute may be sustained under the police power only if there is concurrence of the lawful subject and the method.
Subject and purpose of the Agrarian Reform Law is valid, however what is to be determined is the method employed to
achieve it.
Lozano v. Martinez
GR L-63419, 18 December 1986

Facts:
A motion to quash the charge against the petitioners for violation of the BP 22 was made, contending that no offense
was committed, as the statute is unconstitutional. Such motion was denied by the RTC. The petitioners thus elevate the
case to the Supreme Court for relief. The Solicitor General, commented that it was premature for the accused to elevate
to the Supreme Court the orders denying their motions to quash. However, the Supreme Court finds it justifiable to
intervene for the review of lower court's denial of a motion to quash.

Issue:
Whether or not BP 22 is constitutional as it is a proper exercise of police power of the State.

Held:
The enactment of BP 22 a valid exercise of the police power and is not repugnant to the constitutional inhibition against
imprisonment for debt.
The offense punished by BP 22 is the act of making and issuing a worthless check or a check that is dishonored upon its
presentation for payment. It is not the non-payment of an obligation which the law punishes. The law is not intended or
designed to coerce a debtor to pay his debt.
The law punishes the act not as an offense against property, but an offense against public order. The thrust of the law is
to prohibit, under pain of penal sanctions, the making of worthless checks and putting them in circulation. An act may
not be considered by society as inherently wrong, hence, not malum in se but because of the harm that it inflicts on the
community, it can be outlawed and criminally punished as malum prohibitum. The state can do this in the exercise of its
police power.
DECS v San Diego
G.R. No. 89572, December 21, 1989

FACTS:
Roberto Rey San Diego, the private respondent is a graduate of the University of the East with a degree of Bachelor of
Science in Zoology. The petitioner claims that he took the NMAT three times and flunked it as many times.

ISSUE:
Petition whether the private respondent who has thrice failed the National Medical Admission Test (NMAT) is entitled to
take it again as it is a requirement for admission to any Medical School in the Philippines. He invoked of his
constitutional rights to academic freedom and quality education, squarely challenging the constitutionality of MECS
Order No. 12, Series of 1972.

HELD:
The private respondent cannot take the NMAT again and pursue his medical profession because of the following
grounds:
1. For the purpose of gauging at least initially by the admission test and by the three-flunk rule, as tudent shall not
be allowed to take the NMAT again after three successive failures.
2. The State ensures that medical profession is not permeated by incompetents to whom patients may unwarily
hand over their lives and health.
3. It is not enough to simply invoke the right to quality education as a guarantee of the Constitution, while one has
the right to aspire to be a doctor, he does not have the constitutional right to be a doctor; one must show that
he is entitled to it because of his preparation and promise.
4. The conflict that the challenged rule violates the equal protection clause is not well-taken. Conformable to
Article III, Section 1 of the Constitution, a law does not have to operate with equal force on all person or things.
YNOT VS. IAC
148 SCRA 659; G.R. NO. 74457; 20 MAR 1987

Facts:
Executive Order No. 626-A prohibited the transportation of carabaos and carabeef from one province to another. The
carabaos of petitioner were confiscated for violation of Executive Order No 626-A while he was transporting them from
Masbate to Iloilo. Petitioner challenged the constitutionality of Executive Order No. 626-A. The government argued that
Executive Order No. 626-A was issued in the exercise of police power to conserve the carabaos that were still fit for farm
work or breeding.

Issue:
Whether or Not EO No. 626-A is a violation of Substantive Due Process.

Held:
The challenged measure is an invalid exercise of police power, because it is not reasonably necessary for the purpose of
the law and is unduly oppressive. It is difficult to see how prohibiting the transfer of carabaos from one province to
another can prevent their indiscriminate killing. Retaining the carabaos in one province will not prevent their slaughter
there. Prohibiting the transfer of carabeef, after the slaughter of the carabaos, will not prevent the slaughter either.
CITY GOVERNMENT OF QUEZON CITY VS. ERICTA
122 SCRA 759; G.R. No. L-34915; 24 Jun 1983

Facts:
Section 9 of Ordinance No. 6118, S-64, entitled "Ordinance Regulating The Establishment, Maintenance And Operation
Of Private Memorial Type Cemetery Or Burial Ground Within The Jurisdiction Of Quezon City And Providing Penalties For
The Violation Thereof" provides:
Sec. 9. At least six (6) percent of the total area of the memorial park cemetery shall be set aside for charity burial of
deceased persons who are paupers and have been residents of Quezon City for at least 5 years prior to their death, to be
determined by competent City Authorities. The area so designated shall immediately be developed and should be open
for operation not later than six months from the date of approval of the application.
For several years, the aforequoted section of the Ordinance was not enforced but seven years after the enactment of
the ordinance, the Quezon City Council passed a resolution to request the City Engineer, Quezon City, to stop any
further selling and/or transaction of memorial park lots in Quezon City where the owners thereof have failed to donate
the required 6% space intended for paupers burial.
The Quezon City Engineer then notified respondent Himlayang Pilipino, Inc. in writing that Section 9 of the ordinance
would be enforced.
Respondent Himlayang Pilipino reacted by filing a petition for declaratory relief, prohibition and mandamus with
preliminary injunction seeking to annul Section 9 of the Ordinance in question. Respondent alleged that the same is
contrary to the Constitution, the Quezon City Charter, the Local Autonomy Act, and the Revised Administrative Code.

Issue:
Whether or Not Section 9 of the ordinance in question is a valid exercise of police power.

Held:
Section 9 of the City ordinance in question is not a valid exercise of police power. Section 9 cannot be justified under the
power granted to Quezon City to tax, fix the license fee, and regulate such other business, trades, and occupation as
may be established or practiced in the City.
Bill of rights states that 'no person shall be deprived of life, liberty or property without due process of law' (Art. Ill,
Section 1 subparagraph 1, Constitution). On the other hand, there are three inherent powers of government by which
the state interferes with the property rights, namely-. (1) police power, (2) eminent domain, (3) taxation.
The police power of Quezon City is defined in sub-section 00, Sec. 12, Rep. Act 537 that reads as follows:
“To make such further ordinance and regulations not repugnant to law as may be necessary to carry into effect and
discharge the powers and duties conferred by this act and such as it shall deem necessary and proper to provide for the
health and safety, …, and for the protection of property therein; and enforce obedience thereto with such lawful fines or
penalties as the City Council may prescribe under the provisions of subsection (jj) of this section.”
The power to regulate does not include the power to prohibit. The power to regulate does not include the power to
confiscate. The ordinance in question not only confiscates but also prohibits the operation of a memorial park cemetery,
because under Section 13 of said ordinance, 'Violation of the provision thereof is punishable with a fine and/or
imprisonment and that upon conviction thereof the permit to operate and maintain a private cemetery shall be revoked
or cancelled’. The confiscatory clause and the penal provision in effect deter one from operating a memorial park
cemetery.
Moreover, police power is defined by Freund as 'the power of promoting the public welfare by restraining and regulating
the use of liberty and property'. It is usually exerted in order to merely regulate the use and enjoyment of property of
the owner. If he is deprived of his property outright, it is not taken for public use but rather to destroy in order to
promote the general welfare.
It seems to the court that Section 9 of Ordinance No. 6118, Series of 1964 of Quezon City is not a mere police regulation
but an outright confiscation. It deprives a person of his private property without due process of law, nay, even without
compensation.
White Light Corporation vs. City of Manila
G.R. No. 122846. January 20, 2009

Facts:
On December 3, 1992, City Mayor Alfredo S. Lim signed into law and ordinance entitled “An Ordinance Prohibiting Short-
time Admission, Short-time Admission Rates, and Wash-up Schemes in Hotels, Motels, Inns, Lodging Houses, and Similar
Establishments in the City of Manila.”
On December 15, 1992, the Malate Tourist and Development Corporation (MTDC) filed a complaint for declaratory relief
with prayer for a writ of preliminary injunction and/or temporary restraining order (TRO) with the Regional Trial Court of
Manila, Branch 9 and prayed that the Ordinance be declared invalid and unconstitutional.
On December 21, 1992, petitioners White Light Corporation, Titanium Corporation and Sta. Mesa Tourist Development
Corporation filed a motion to intervene, which was granted by the RTC. MTDC moved to withdraw as plaintiff which was
also granted by the RTC.
On January 14, 1993, the RTC issued a TRO directing the City to cease and desist from enforcing the Ordinance.
On October 20, 1993, the RTC rendered a decision declaring the Ordinance null and void.
The City then filed a petition for review on certiorari with the Supreme Court. However, the Supreme Court referred the
same to the Court of Appeals.
The City asserted that the Ordinance is a valid exercise of police power pursuant to Local government code and the
Revised Manila charter.

The Court of Appeals reversed the decision of the RTC and affirmed the constitutionality of the Ordinance.

Issue:
Whether the Ordinance is constitutional.

Held:
No, it is not constitutional. The apparent goal of the Ordinance is to minimize if not eliminate the use of the covered
establishments for illicit sex, prostitution, drug use and the like. These goals, by themselves, are unimpeachable and
certainly fall within the ambit of the police power of the State. Yet the desirability of these ends does not sanctify any
and all means for their achievement.
However well-intentioned the Ordinance may be, it is in effect an arbitrary and whimsical intrusion into the rights of the
establishments as well as their patrons. The Ordinance needlessly restrains the operation of the businesses of the
petitioners as well as restricts the rights of their patrons without sufficient justification.
MIRASOL VS CA
351 SCRA 44; G.R. No. 128448; 1 Feb 2001

Facts:
The Mirasols are sugarland owners and planters. Philippine National Bank (PNB) financed the Mirasols' sugar production
venture FROM 1973-1975 under a crop loan financing scheme. The Mirasols signed Credit Agreements, a Chattel
Mortgage on Standing Crops, and a Real Estate Mortgage in favor of PNB. The Chattel Mortgage empowered PNB to
negotiate and sell the latter's sugar and to apply the proceeds to the payment of their obligations to it.
President Marcos issued PD 579 in November, 1974 authorizing Philippine Exchange Co., Inc. (PHILEX) to purchase sugar
allocated for export and authorized PNB to finance PHILEX's purchases. The decree directed that whatever profit PHILEX
might realize was to be remitted to the government. Believing that the proceeds were more than enough to pay their
obligations, petitioners asked PNB for an accounting of the proceeds which it ignored. Petitioners continued to avail of
other loans from PNB and to make unfunded withdrawals from their accounts with said bank. PNB asked petitioners to
settle their due and demandable accounts. As a result, petitioners, conveyed to PNB real properties by way of dacion en
pago still leaving an unpaid amount. PNB proceeded to extrajudicially foreclose the mortgaged properties. PNB still had
a deficiency claim.
Petitioners continued to ask PNB to account for the proceeds, insisting that said proceeds, if properly liquidated, could
offset their outstanding obligations. PNB remained adamant in its stance that under P.D. No. 579, there was nothing to
account since under said law, all earnings from the export sales of sugar pertained to the National Government.
On August 9, 1979, the Mirasols filed a suit for accounting, specific performance, and damages against PNB.

Issue:
-Whether or not the Trial Court has jurisdiction to declare a statute unconstitutional without notice to the Solicitor
General where the parties have agreed to submit such issue for the resolution of the Trial Court.
-Whether PD 579 and subsequent issuances thereof are unconstitutional.
-Whether or not said PD is subject to judicial review.

Held:
It is settled that Regional Trial Courts have the authority and jurisdiction to consider the constitutionality of a statute,
presidential decree, or executive order. The Constitution vests the power of judicial review or the power to declare a
law, treaty, international or executive agreement, presidential decree, order, instruction, ordinance, or regulation not
only in this Court, but in all Regional Trial Courts.
The purpose of the mandatory notice in Rule 64, Section 3 is to enable the Solicitor General to decide whether or not his
intervention in the action assailing the validity of a law or treaty is necessary. To deny the Solicitor General such notice
would be tantamount to depriving him of his day in court. We must stress that, contrary to petitioners' stand, the
mandatory notice requirement is not limited to actions involving declaratory relief and similar remedies. The rule itself
provides that such notice is required in "any action" and not just actions involving declaratory relief. Where there is no
ambiguity in the words used in the rule, there is no room for construction. 15 In all actions assailing the validity of a
statute, treaty, presidential decree, order, or proclamation, notice to the Solicitor General is mandatory.

Petitioners contend that P.D. No. 579 and its implementing issuances are void for violating the due process clause and
the prohibition against the taking of private property without just compensation. Petitioners now ask this Court to
exercise its power of judicial review.
Jurisprudence has laid down the following requisites for the exercise of this power: First, there must be before the Court
an actual case calling for the exercise of judicial review. Second, the question before the Court must be ripe for
adjudication. Third, the person challenging the validity of the act must have standing to challenge. Fourth, the question
of constitutionality must have been raised at the earliest opportunity, and lastly, the issue of constitutionality must be
the very lis mota of the case.
Social Justice Society (SJS), Vladimir Alarique T. Cabigao, and Bonifacio S. Tumbokon vs. Hon. Jose L. Atienza, jr., in his
capacity as Mayor of Manila
G.R. No. 156052,March 7, 2007

Facts:
Pursuant to the police power delegated to local government units. the City of Manila enacted Ordinance No. 8027.
The said ordinance, in essence, reclassified portions of Pandacan and Sta. Ana as well as its adjoining areas from
industrial to commercial areas [reservoir of oils of big oil companies are located in this area- this is called as the
Pandacan terminals] and owners or operators of industries and other businesses, of the Pandacan terminals are given a
period of 6 months from the date of effectivity of the Ordinance within which to cease and desist from the operation of
businesses which are disallowed.
Subsequent to the approval of the ordinance, the City of Manila and the Department of Energy (DOE) entered into a
memorandum of understanding (MOU) with the oil companies in which they agreed that the scaling down of the
Pandacan Terminals was the most viable and practicable option and not total removal of the Pandacan terminals as
demanded by Ordinance 8027. Under the MOU, the oil companies agreed to scale down the oils reservoir and agreed
that the joint operations of the OIL COMPANIES in the Pandacan Terminals shall be limited to the common and
integrated areas/facilities. The Sangguniang Panlungsod ratified the memorandum of understanding in Resolution No.
97.and declared that the memorandum of understanding was effective only for a period of six months and then was
extended again until April 30, 2003 and authorizing Mayor Atienza to issue special business permits to the oil
companies.
Petitioners filed a mandamus compelling respondent in his capacity as Mayor of Manila to enforce the said ordinance
and order the immediate removal of the terminals of the oil companies.
Atienza contended that Ordinance 8027 was superseded by the MOU, hence he cannot enforce it.

Issue:
Whether or not respondent can be compelled to enforce Ordinance 8027.
Whether or not the MOU superseded Ordinance 8027.

Ruling:
Yes, the Local Government Code imposes upon respondent the duty, as city mayor, to 'enforce all laws and ordinances
relative to the governance of the city.' One of these is Ordinance No. 8027. As the chief executive of the city, he has the
duty to enforce Ordinance No. 8027 as long as it has not been repealed by theSanggunian or annulled by the courts. He
has no other choice. It is his ministerial duty to do so. The Court ratiocinated, "these officers cannot refuse to perform
their duty on the ground of an alleged invalidity of the statute imposing the duty. The reason for this is obvious. It might
seriously hinder the transaction of public business if these officers were to be permitted in all cases to question the
constitutionality of statutes and ordinances imposing duties upon them and which have not judicially been declared
unconstitutional. Officers of the government from the highest to the lowest are creatures of the law and are bound to
obey it."
As to the second issue, assuming that the terms of the MOU were inconsistent with Ordinance No. 8027, the
resolutions which ratified it and made it binding on the City of Manila expressly gave it full force and effect only until
April 30, 2003. Thus, at present, there is nothing that legally hinders respondent from enforcing Ordinance No. 8027.
MMDA vs VIRON TRANSPORTATION
530 SCRA 341 (2007), EN BANC, (CARPIO MORALES, J.)

It is the DOTC, and not the MMDA, which is authorized to establish and implement a project such as the mass transport
system.

FACTS:
To solve the worsening traffic congestions problem in Metro Manila the President issued Executive Order (E.O.) 179,
―Providing for the Establishment of Greater Manila Mass TransportaQon System. As determined in E.O. 179, the
primary cause of traffic congestion in Metro Manila has been the numerous buses plying the streets that impede the
flow of vehicles and commuters and the inefficient connectivity of the different transport modes. To decongest traffic,
petitioner Metropolitan Manila Development Authority (MMDA) came up with a recommendation, proposing the
elimination of bus terminals located along major Metro Manila thoroughfares, and the construction of mass transport
terminal facilties to provide a more convenient access to mass transport system to the commuting public.

The project provided for under this E.O. was called ―Greater Manila Transport System‖ (Project) wherein the MMDA
was designated as the implementing agency. Accordingly, the Metro Manila Council the governing board of the MMDA
issued a resolution, expressing full support of the project.

The respondents, which are engaged in the business of public transportation with a provincial bus operation, Viron
Transport Co., Inc. and Mencorp Transportation System, Inc., assailed the constitutionality of E.O. 179 before the
Regional Trial Court of Manila. They alleged that the E.O., insofar as it permitted the closure of existing bus terminal,
constituted a deprivation of property without due process; that it contravened the Public Service Act which mandates
public utilities to provide and maintain their own terminals as a requisite for the privilege of operating as common
carriers; and that Republic Act 7924, which created MMDA, did not authorize the latter to order the closure of bus
terminals. The trial court declared the E.O. unconstitutional.

The MMDA argued before the Court that there was no justiciable controversy in the case for declaratory relief filed by
the respondents; that E.O. 179 was only an administrative directive to government agencies to coordinate with the
MMDA, and as such did not bind third persons; that the President has the authority to implement the Project pursuant
to E.O. 125; and that E.O. 179 was a valid exercise of police power.

ISSUE:
Whether or not E.O, 179 is constitutional

HELD:
By designating the MMDA as implementing agency of the “Greater Manila Transport System,” the President clearly
overstepped the limits of the authority conferred by law, rendering E.O. 179 ultra vires

Executive Order 125, invoked by the MMDA, was issued by former President Aquino in her exercise of legislative
powers. This executive order reorganized the Ministry (now Department) of Transportation and Communications
(DOTC), and defined its powers and functions. It mandated the DOTC to be the primary policy, planning, programming,
coordinating, implementing, regulating and administrative entity to promote, develop and regulate networks of
transportation and communications.

The grant of authority to the DOTC includes the power to establish and administer comprehensive and integrated
programs for transportation and communications. Accordingly, it is the DOTC Secretary who is authorized to issue such
orders, rules, regulations and other issuances as may be necessary to ensure the effective implementation of the law.
The President may also exercise the same power and authority to order the implementation of the mass transport
system project, which admittedly is one for transportation. Such authority springs from the President‘s power of control
over all executive departments as well as for the faithful execution of the laws under the Constitution.

Thus, the President, although authorized to establish or cause the implementation of the Project, must exercise the
authority through the instrumentality of the DOTC, which, by law, is the primary implementing and administrative entity
in the promotion, development and regulation of networks of transportation. It is the DOTC, and not the MMDA, which
is authorized to establish and implement a project such as the mass transport system. By designating the MMDA as
implementing agency of the Project, the President clearly overstepped the limits of the authority conferred by law,
rendering E.O. 179 ultra vires.

In the absence of a specific grant of authority to it under R.A. 7924, MMDA cannot issue order for the closure of existing
bus terminals Republic Act (R.A.) 7924 authorizes the MMDA to perform planning, monitoring and coordinative
functions, and in the process exercises regulatory and supervisory authority over the delivery of metro-wide services,
including transport and traffic management. While traffic decongestion has been recognized as a valid ground in the
exercise of police power, MMDA is not granted police power, let alone legislative power. Unlike the legislative bodies of
the local government units, there is no provision in R.A. 7924 that empowers the MMDA or the Metro Manila Council to
enact ordinances, approve resolutions and appropriate funds for the general welfare of the inhabitants of Metro Manila.

In light of the administrative nature of its powers and functions, the MMDA is devoid of authority to implement the
Greater Manila Transport System as envisioned by E.O. 179; hence, it could not have been validly designated by the
President to undertake the project. It follows that the MMDA cannot validly order the elimination of respondents‘
terminals.

Even assuming arguendo that police power was delegated to the MMDA, its exercise of such power does not satisfy the
two sets of a valid police power measure: (1) the interest of the public generally, as distinguished from that of a
particular class, requires its exercise; and (2) the means employed are reasonably necessary for the accomplishment of
the purpose and not unduly oppressive upon individuals.

In various cases, the Court has recognized that traffic congestion is a public, not merely a private concern. Indeed, the
E.O. was issued due to the felt need to address the worsening traffic congestion in Metro Manila which, the MMDA so
determined, is caused by the increasing volume of buses plying the major thoroughfares and the inefficient connectivity
of existing transport system.

With the avowed objective of decongesting traffic in Metro Manila the E.O. seeks to eliminate the bus terminals now
located along major Metro Manila thoroughfares and provide more convenient access to the mass transport system to
the commuting public through the provision of mass transport terminal facilities. Common carriers with terminals along
the major thoroughfares of Metro Manila would thus be compelled to close down their existing bus terminals and use
the MMDA-designated common parking areas. The Court fails to see how the prohibition against respondents‘ terminals
can be considered a reasonable necessity to ease traffic congestion in the metropolis. On the contrary, the elimination
of respondents‘ bus terminals brings forth the distinct possibility and the equally harrowing reality of traffic congestion
in the common parking areas, a case of transference from one site to another.

Moreover, an order for the closure of bus terminals is not in line with the provisions of the Public Service Act. The
establishment, as well as the maintenance of vehicle parking areas or passenger terminals, is generally considered a
necessary service by provincial bus operators, hence, the investments they have poured into the acquisition or lease of
suitable terminal sites.
POWER OF EMINENT DOMAIN

CITY OF MANILA VS. CHINESE COMMUNITY


40 Phil 349; No. 14355; 31 Oct 1919

Facts:
The City of Manila, plaintiff herein, prayed for the expropriation of a portion private cemetery for the conversion into an
extension of Rizal Avenue. Plaintiff claims that it is necessary that such public improvement be made in the said portion
of the private cemetery and that the said lands are within their jurisdiction.

Defendants herein answered that the said expropriation was not necessary because other routes were available. They
further claimed that the expropriation of the cemetery would create irreparable loss and injury to them and to all those
persons owing and interested in the graves and monuments that would have to be destroyed.

The lower court ruled that the said public improvement was not necessary on the particular-strip of land in question.
Plaintiff herein assailed that they have the right to exercise the power of eminent domain and that the courts have no
right to inquire and determine the necessity of the expropriation. Thus, the same filed an appeal.

Issue:
Whether or not the courts may inquire into, and hear proof of the necessity of the expropriation.

Held:
The courts have the power of restricting the exercise of eminent domain to the actual reasonable necessities of the case
and for the purposes designated by the law. The moment the municipal corporation or entity attempts to exercise the
authority conferred, it must comply with the conditions accompanying the authority. The necessity for conferring the
authority upon a municipal corporation to exercise the right of eminent domain is admittedly within the power of the
legislature. But whether or not the municipal corporation or entity is exercising the right in a particular case under the
conditions imposed by the general authority, is a question that the courts have the right to inquire to.
REPUBLIC VS. PLDT
26 SCRA 320; G.R. No. L-18841; 27 Jan 1969

Facts:
The plaintiff Republic of the Philippines is a political entity exercising government powers through one of its branches,
the Bureau of Telecommunication. Herein defendant, PLDT is a public service corporation holding a franchise to install
operates and maintains a telephone system. After its creation, the BOT set up its own government telephone system by
utilizing its own appropriations and other equipment and by renting trunk lines of the PLDT to enable the govt offices to
call privately. BOT entered into an agreement with the RCA communications for joint overseas telephone service
whereby BOT would convey overseas calls received by RCA to local residents. PLDT complained to the BOT that it was a
violation of the condition of their agreement since the BOT had used trunk lines only for the use of government offices
but even to serve private persons or the general public in competition with the business of PLDT. Subsequently, the
plaintiff commenced suit against PLDT asking the court judgment be rendered ordering the PLDT to execute a contract
with the plaintiff, through the BOT for the use of the facilities of PLDT's telephone system throughout the country under
such conditions as the court may consider reasonable. The CFI rendered judgment stating that it could not compel PLDT
to enter into such agreement. Hence this petition.

Issue:
Whether or Not PLDT may be compelled to enter into such agreement.

Held:
Yes, the state, may, in the interest of national welfare transfer utilities to public ownership upon payment of just
compensation, there is no reason why the state ma not require a public utility to render services in the general interest
provided just compensation is paid.
PEOPLE VS. FAJARDO
104 Phil 443; G.R. No. L-12172; 29 Aug 1958

Facts:
The municipal council of baao, camarines sur stating among others that construction of a building, which will destroy the
view of the plaza, shall not be allowed and therefore be destroyed at the expense of the owner, enacted an ordinance.
Herein appellant filed a written request with the incumbent municipal mayor for a permit to construct a building
adjacent to their gasoline station on a parcel of land registered in Fajardo's name, located along the national highway
and separated from the public plaza by a creek. The request was denied, for the reason among others that the proposed
building would destroy the view or beauty of the public plaza. Defendants reiterated their request for a building permit,
but again the mayor turned down the request. Whereupon, appellants proceeded with the construction of the building
without a permit, because they needed a place of residence very badly, their former house having been destroyed by a
typhoon and hitherto they had been living on leased property. Thereafter, defendants were charged in violation of the
ordinance and subsequently convicted. Hence this appeal.

Issue:
Whether or Not the ordinance is a valid exercise of police power.

Held:
No. It is not a valid exercise of police power. The ordinance is unreasonable and oppressive, in that it operates to
permanently deprive appellants of the right to use their own property; hence, it oversteps the bounds of police power,
and amounts to a taking of appellant’s property without just compensation. We do not overlook that the modern
tendency is to regard the beautification of neighborhoods as conducive to the comfort and happiness of residents.

As the case now stands, every structure that may be erected on appellants' land, regardless of its own beauty, stands
condemned under the ordinance in question, because it would interfere with the view of the public plaza from the
highway. The appellants would, in effect, be constrained to let their land remain idle and unused for the obvious
purpose for which it is best suited, being urban in character. To legally achieve that result, the municipality must give
appellants just compensation and an opportunity to be heard.
REPUBLIC VS. CASTELVI
58 SCRA 336; G.R. No. L-20620; 15 Aug 1974

Facts:
In 1947, the republic, through the Armed Forces of the Philippines (AFP), entered into a lease agreement with Castelvi
on a year-to-year basis. When Castelvi gave notice to terminate the lease in 1956, the AFP refused. She then instituted
an ejectment proceeding against the AFP. In 1959, however, the republic commenced the expropriation proceedings for
the land in question.

Issue:
Whether or Not the compensation should be determined as of 1947 or 1959.

Held:
The Supreme Court ruled that the “taking” should not be reckoned as of 1947, and that just compensation should not be
determined on the basis of the value of the property as of that year.

The requisites for taking are: 1) the expropriator must enter a private property, 2) the entry must be for more than a
momentary period, 3) it must be under warrant or color of authorities, 4) the property must be devoted for public use or
otherwise informally appropriated or injuriously affected, and 5) the utilization of the property for public use must be
such a way as to oust the owner and deprive him of beneficial enjoyment of the property. Under Sec. 4 Rule 67 of the
Rules of Court, “just compensation” is to be determined as of the date of the filing of the complaint. The Supreme Court
has ruled that when the taking of the property sought to be expropriated coincides with the commencement of the
expropriation proceedings, or takes place subsequent to the filing of the complaint for eminent domain, the just
compensation should be determined as of the date of the filing of the complaint. In the instant case, it is undisputed
that the Republic was placed in possession of the Castelvi property, by authority of court, on August 10, 1959. The
“taking” of the Castelvi property for the purposes of determining the just compensation to be paid must, therefore, be
reckoned as of June 26, 1959 when the complaint for eminent domain was filed. There is no basis to the contention of
the Republic that a lease on a year-to-year basis can give rise to permanent right to occupy since by express provision a
lease made for a determinate time, as was the lease of Castelvi land in the instant case, ceases upon the day fixed,
without need of a demand (Art. 1669, New Civil Code). The Supreme Court, however, did not apply Art. 1250 of the New
Civil Code for the adjustment of the peso rate in times of extraordinary inflation or deflation because in eminent domain
cases the obligation to pay arises from law independent of contract.
AMIGABLE VS. CUENCA
43 SCRA 360; G.R. No. L-26400; 29 Feb. 1972

Facts:
Victoria Amigable is the registered owner of a particular lot. At the back of her Transfer Certificate of Title (1924), there
was no annotation in favor of the government of any right or interest in the property. Without prior expropriation or
negotiated sale, the government used a portion of the lot for the construction of the Mango and Gorordo Avenues. On
1958, Amigable’s counsel wrote the President of the Philippines, requesting payment of the portion of the said lot. It
was disallowed by the Auditor General in his 9th Endorsement. Petitioner then filed in the court a quo a complaint
against the Republic of the Philippines and Nicolas Cuenca, in his capacity as Commissioner of Public Highways for the
recovery of ownership and possession of the lot. According to the defendants, the action was premature because it was
not filed first at the Office of the Auditor General. According to them, the right of action for the recovery of any amount
had already prescribed, that the Government had not given its consent to be sued, and that plaintiff had no cause of
action against the defendants.

Issue:
Whether or Not, under the facts of the case, appellant may properly sue the government.

Held:
In the case of Ministerio v. Court of First Instance of Cebu, it was held that when the government takes away property
from a private landowner for public use without going through the legal process of expropriation or negotiated sale, the
aggrieved party may properly maintain a suit against the government without violating the doctrine of governmental
immunity from suit without its consent. In the case at bar, since no annotation in favor of the government appears at
the back of the certificate of title and plaintiff has not executed any deed of conveyance of any portion of the lot to the
government, then she remains the owner of the lot. She could then bring an action to recover possession of the land
anytime, because possession is one of the attributes of ownership. However, since such action is not feasible at this time
since the lot has been used for other purposes, the only relief left is for the government to make due compensation—
price or value of the lot at the time of the taking.
PHILIPPINE PRESS INSTITUTE VS. COMELEC
244 SCRA 272; G.R. No. 119694; 22 May 1995

Facts:
Respondent Comelec promulgated Resolution No. 2772 directing newspapers to provide free Comelec space of not less
than one-half page for the common use of political parties and candidates. The Comelec space shall be allocated by the
Commission, free of charge, among all candidates to enable them to make known their qualifications, their stand on
public Issue and their platforms of government. The Comelec space shall also be used by the Commission for
dissemination of vital election information.

Petitioner Philippine Press Institute, Inc. (PPI), a non-profit organization of newspaper and magazine publishers, asks the
Supreme Court to declare Comelec Resolution No. 2772 unconstitutional and void on the ground that it violates the
prohibition imposed by the Constitution upon the government against the taking of private property for public use
without just compensation. On behalf of the respondent Comelec, the Solicitor General claimed that the Resolution is a
permissible exercise of the power of supervision (police power) of the Comelec over the information operations of print
media enterprises during the election period to safeguard and ensure a fair, impartial and credible election.

Issue:
Whether or not Comelec Resolution No. 2772 is unconstitutional.

Held:
The Supreme Court declared the Resolution as unconstitutional. It held that to compel print media companies to donate
“Comelec space” amounts to “taking” of private personal property without payment of the just compensation required
in expropriation cases. Moreover, the element of necessity for the taking has not been established by respondent
Comelec, considering that the newspapers were not unwilling to sell advertising space. The taking of private property for
public use is authorized by the constitution, but not without payment of just compensation. Also Resolution No. 2772
does not constitute a valid exercise of the police power of the state. In the case at bench, there is no showing of
existence of a national emergency to take private property of newspaper or magazine publishers.
Sumulong and Vidanes-Balaoing vs Guerrero and NHA
G.R. No. L-48685 September 30, 1987

FACTS:
On December 5, 1977 the National Housing Authority (NIIA) filed a complaint for expropriation of parcels of land
covering approximately twenty five (25) hectares, (in Antipolo, Rizal) including the lots of petitioners Lorenzo Sumulong
and Emilia Vidanes-Balaoing with an area of 6,667 square meters and 3,333 square meters respectively. The land sought
to be expropriated were valued by the NHA at one peso (P1.00) per square meter adopting the market value fixed by the
provincial assessor in accordance with presidential decrees prescribing the valuation of property in expropriation
proceedings.

Together with the complaint was a motion for immediate possession of the properties. The NHA deposited the amount
of P158,980.00 with the Philippine National Bank, representing the "total market value" of the subject twenty five
hectares of land, pursuant to Presidential Decree No. 1224 which defines "the policy on the expropriation of private
property for socialized housing upon payment of just compensation."

Petitioners filed a motion for reconsideration on the ground that they had been deprived of the possession of their
property without due process of law. This was however, denied

ISSUES:
1) Respondent Judge acted without or in excess of his jurisdiction or with grave abuse of discretion by issuing the Order
of January 17, 1978 without notice and without hearing and in issuing the Order dated June 28, 1978 denying the
motion for reconsideration.

2) Pres. Decree l224, as amended, is unconstitutional for being violative of the due process clause, specifically:
• The Decree would allow the taking of property regardless of size and no matter how small the area to be
expropriated;
• "Socialized housing" for the purpose of condemnation proceeding, as defined in said Decree, is not really for a
public purpose;
• The Decree violates procedural due process as it allows immediate taking of possession, control and disposition
of property without giving the owner his day in court;
• The Decree would allow the taking of private property upon payment of unjust and unfair valuations arbitrarily
fixed by government assessors;
• The Decree would deprive the courts of their judicial discretion to determine what would be the "just
compensation" in each and every raise of expropriation.

HELD:
The exercise of the power of eminent domain is subject to certain limitations imposed by the constitution, to wit:
Private property shall not be taken for public use without just compensation (Art. IV, Sec. 9);
No person shall be deprived of life, liberty, or property without due process of law, nor shall any person be denied the
equal protection of the laws (Art. IV, sec. 1).
Petitioners contend that "socialized housing" as defined in Pres. Decree No. 1224, as amended, for the purpose of
condemnation proceedings is not "public use" since it will benefit only "a handful of people, bereft of public character."
The "public use" requirement for a and exercise of the power of eminent domain is a flexible and evolving concept
influenced by changing conditions.
The restrictive view of public use may be appropriate for a nation which circumscribes the scope of government
activities and public concerns and which possesses big and correctly located public lands that obviate the need to take
private property for public purposes. Neither circumstance applies to the Philippines. We have never been a laissez faire
State. And the necessities which impel the exertion of sovereign power are all too often found in areas of scarce public
land or limited government resources. (p. 231)
Specifically, urban renewal or redevelopment and the construction of low-cost housing is recognized as a public
purpose, not only because of the expanded concept of public use but also because of specific provisions in the
Constitution. The 1973 Constitution made it incumbent upon the State to establish, maintain and ensure adequate social
services including housing [Art. 11, sec. 7]. The 1987 Constitution goes even further by providing that:
The State shall promote a just and dynamic social order that will ensure the prosperity and independence of the nation
and free the people from poverty through policies that provide adequate social services, promote full employment, a
rising standard of living and an improved quality of life for all. [Art. II, sec. 9]
MANOSCA VS. COURT OF APPEALS
252 SCRA 412; G.R. NO. 106440, 29 JAN. 1996

Facts:
The National Historical Institute declared the parcel of land owned by Petitioners as a national historical landmark,
because it was the site of the birth of Felix Manalo, the founder of Iglesia ni Cristo. The Republic of the Philippines filed
an action to appropriate the land. Petitioners argued that the expropriation was not for a public purpose.

Issue:
Whether or Not the taking or exercise of eminent domain may be granted.

Held:
Public use should not be restricted to the traditional uses. The taking is for a public use because of the contribution of
Felix Manalo to the culture and history of the Philippines.
EPZA VS. DULAY
148 SCRA 305; G.R. No. L-59603; 29 Apr 1987

Facts:
The four parcels of land which are the subject of this case is where the Mactan Export Processing Zone Authority in Cebu
(EPZA) is to be constructed. Private respondent San Antonio Development Corporation (San Antonio, for brevity), in
which these lands are registered under, claimed that the lands were expropriated to the government without them
reaching the agreement as to the compensation. Respondent Judge Dulay then issued an order for the appointment of
the commissioners to determine the just compensation. It was later found out that the payment of the government to
San Antonio would be P15 per square meter, which was objected to by the latter contending that under PD 1533, the
basis of just compensation shall be fair and according to the fair market value declared by the owner of the property
sought to be expropriated, or by the assessor, whichever is lower. Such objection and the subsequent Motion for
Reconsideration were denied and hearing was set for the reception of the commissioner’s report. EPZA then filed this
petition for certiorari and mandamus enjoining the respondent from further hearing the case.

Issue:
Whether or Not the exclusive and mandatory mode of determining just compensation in PD 1533 is unconstitutional.

Held:
The Supreme Court ruled that the mode of determination of just compensation in PD 1533 is unconstitutional.

The method of ascertaining just compensation constitutes impermissible encroachment to judicial prerogatives. It tends
to render the courts inutile in a matter in which under the Constitution is reserved to it for financial determination. The
valuation in the decree may only serve as guiding principle or one of the factors in determining just compensation, but it
may not substitute the court’s own judgment as to what amount should be awarded and how to arrive at such amount.
The determination of just compensation is a judicial function. The executive department or the legislature may make the
initial determination but when a party claims a violation of the guarantee in the Bill of Rights that the private party may
not be taken for public use without just compensation, no statute, decree, or executive order can mandate that its own
determination shall prevail over the court’s findings. Much less can the courts be precluded from looking into the
justness of the decreed compensation.
MUNICIPALITY OF PARAÑAQUE VS. VM REALTY CORPORATION
292 SCRA 676; G. R. NO. 127820; 20 JUL 1998

Facts:
Petitioner sought to exercise its power of eminent domain based on a resolution by the municipal council. Petitioner
cites a previous case wherein a resolution gave authority to exercise eminent domain. Petitioner also relies on the
Implementing Rules, which provides that a resolution authorizes a Local Government Unit to exercise eminent domain.

Issue:
Whether or Not an LGU can exercise its power of eminent domain pursuant to a resolution by its law-making body.

Held:
Under Section 19, of the present Local Government Code (RA 7160), it is stated as the first requisite that LGUs can
exercise its power of eminent domain if there is an ordinance enacted by its legislative body enabling the municipal chief
executive. A resolution is not an ordinance, the former is only an opinion of a law-making body, the latter is a law. The
case cited by Petitioner involves BP 337, which was the previous Local Government Code, which is obviously no longer in
effect. RA 7160 prevails over the Implementing Rules, the former being the law itself and the latter only an
administrative rule which cannot amend the former.
MANILA MEMORIAL PARK, INC. vs. SECRETARY OF THE DSWD
G.R. No. 175356 : December 3, 2013

Facts:
On February 26, 2004, RA 9257 or the Expanded Senior Citizens Act amended certain provisions of RA 7432, granting
20% discount to qualified senior citizens and the establishments may claim the discounts granted as tax deduction based
on the net cost of the goods sold or services rendered: Provided, That the cost of the discount shall be allowed as
deduction from gross income for the same taxable year that the discount is granted. Provided, further, That the total
amount of the claimed tax deduction net of value added tax if applicable, shall be included in their gross sales receipts
for tax purposes and shall be subject to proper documentation and to the provisions of the National Internal Revenue
Code, as amended.
The Secretary of Finance issued RR No. 4-2006 and the DSWD issued its own IRR for their implementation of the tax
provision of RA 9257.
Petitioners Manila Memorial Park, Inc. and La Funeraria Paz-Sucat, Inc., domestic corporations engaged in the business
of providing funeral and burial services, feeling aggrieved by the tax deduction scheme, petitioners prayed that Section 4
of RA 9257 and the IRR issued by DSWD and the DOF be declared unconstitutional insofar as these allow business
establishments to claim the 20% discount given to senior citizens as a tax deduction; that the DSWD and the DOF be
prohibited from enforcing the same; and that the tax credit treatment of the 20% discount under the former Section 4
(a) of RA 7432 be reinstated.

ISSUES:
Whether or not Section 4 of RA No. 9257 and its Implementing Rules and Regulations, insofar as they provide that the
20% discount to Senior Citizens may be claimed as tax deduction by the private establishments are invalid and
unconstitutional.

Ruling:
The Senior Citizens Act was enacted primarily to maximize the contribution of senior citizens to nation-building, and to
grant benefits and privileges to them for their improvement and well-being as the State considers them an integral part
of our society.
The priority given to senior citizens finds its basis in the Constitution as set forth in the law itself.
As a form of reimbursement, the law provides that business establishments extending the twenty percent discount to
senior citizens may claim the discount as a tax deduction. The law is a legitimate exercise of police power which, similar
to the power of eminent domain, has general welfare for its object.
While the Constitution protects property rights, petitioners must accept the realities of business and the State, in the
exercise of police power, can intervene in the operations of a business which may result in an impairment of property
rights in the process.
Without sufficient proof that Section 4 (a) of R.A. No. 9257 is arbitrary, and that the continued implementation of the
same would be unconscionably detrimental to petitioners, the Court will refrain from quashing a legislative act.
Apo Fruits vs. LBP
GR 164195, APRIL 5, 2011

FACTS:
We resolve Land Bank of the Philippines (LBP’s) 2nd motion for reconsideration of December 14, 2010 that addresses
one resolution of October 12, 2010 and November 23, 2010. This motion prays as well for the holding of oral arguments.
We likewise resolve the office of the Solicitor General’s motion for leave to intervene and to admit motion for
reconsideration-in-intervention dated February 15, 2011 in behalf of the Republic of the Philippines.

Issue:
Should just compensation under the agrarian reform program be differently from the just compensation in any other
case of expropriation?

RULING:
the constitutional basis of the Agrarian Reform Program is Section 4, Article XIII of the constitution. This provision
expressly provides that the taking of the land for use in the government’s agrarian reform program is conditioned on the
payment of just compensation. Nothing in the wording of this provision even remotely suggests that the just
compensation required from the taking of land for the agrarian reform program should be treated any differently from
the just compensation required in any other case of expropriation. The term "just compensation" is used in several parts
of the Constitution, and, therefore, it must have a uniform meaning.
POWER OF TAXATION

PASCUAL VS. SEC. OF PUBLIC WORKS


110 PHIL 331; G.R. NO.L-10405; 29 DEC 1960

Facts:
Petitioner, the governor of the Province of Rizal, filed an action for declaratory relief with injunction on the ground that
RA 920, Act appropriating funds for public works, providing P85,000 for the construction, reconstruction, repair,
extension and improvement of Pasig feeder road terminals, were nothing but projected and planned subdivision roads
within Antonio Subdivision. Antonio Subdivision is owned by the respondent, Jose Zulueta, a member of the Senate of
the Philippines. Respondent offered to donate the said feeder roads to the municipality of Pasig and the offer was
accepted by the council, subject to a condition that the donor would submit plan of the roads and an agreement to
change the names of two of the street. However, the donation was not executed, which prompted Zuleta to write a
letter to the district engineer calling attention the approval of RA 920. The district engineer, on the other hand, did not
endorse the letter that inasmuch the feeder roads in question were private property at the time of passage and approval
of RA 920, the appropriation for the construction was illegal and therefore, void ab initio. Petitioner, prayed for RA 920
be declared null and void and the alleged deed of donation be declared unconstitutional. Lower court dismissed the case
and dissolved the writ of preliminary injunction.

Issue:
Whether or Not the deed of donation and the appropriation of funds stipulated in RA 920 are constitutional.

Held:
The ruling case law rules that the legislature is without power to appropriate public revenue for anything but public
purpose. The taxing power must be exercised for public purposes only and the money raised by taxation can be
expended only for public purposes and not for the advantage of private individuals.
In the case at bar, the legality of the appropriation of the feeder roads depend upon whether the said roads were public
or private property when the bill was passed by congress or when it became effective. The land which was owned by
Zulueta, the appropriation sought a private purpose and hence, null and void. The donation did not cure the nullity of
the appropriation; therefore a judicial nullification of a said donation need not precede the declaration of
unconstitutionality of the said appropriation.
The decision appealed from is reversed.
PUNSALAN VS. MUNICIPAL BOARD OF MANILA
95 PHIL 46; NO.L-4817; 26 MAY 1954

Facts:
Petitioners, who are professionals in the city, assail Ordinance No. 3398 together with the law authorizing it (Section 18
of the Revised Charter of the City of Manila). The ordinance imposes a municipal occupation tax on persons exercising
various professions in the city and penalizes non-payment of the same. The law authorizing said ordinance empowers
the Municipal Board of the city to impose a municipal occupation tax on persons engaged in various professions.
Petitioners, having already paid their occupation tax under section 201 of the National Internal Revenue Code, paid the
tax under protest as imposed by Ordinance No. 3398. The lower court declared the ordinance invalid and affirmed the
validity of the law authorizing it.

Issue:
Whether or Not the ordinance and law authorizing it constitute class legislation, and authorize what amounts to double
taxation.

Held:
The Legislature may, in its discretion, select what occupations shall be taxed, and in its discretion may tax all, or select
classes of occupation for taxation, and leave others untaxed. It is not for the courts to judge which cities or
municipalities should be empowered to impose occupation taxes aside from that imposed by the National Government.
That matter is within the domain of political departments. The argument against double taxation may not be invoked if
one tax is imposed by the state and the other is imposed by the city. It is widely recognized that there is nothing
inherently terrible in the requirement that taxes be exacted with respect to the same occupation by both the state and
the political subdivisions thereof. Judgment of the lower court is reversed with regards to the ordinance and affirmed as
to the law authorizing it.
LLADOC VS. COMMISSIONER OF INTERNAL REVENUE
14 SCRA 292; NO.L-19201; 16 JUN 1965

Facts:
Sometime in 1957, M.B. Estate Inc., of Bacolod City, donated 10,000.00 pesos in cash to Fr. Crispin Ruiz, the parish priest
of Victorias, Negros Occidental, and predecessor of Fr. Lladoc, for the construction of a new Catholic church in the
locality. The donated amount was spent for such purpose.

On March 3, 1958, the donor M.B. Estate filed the donor's gift tax return. Under date of April 29, 1960. Commissioner of
Internal Revenue issued an assessment for the donee's gift tax against the Catholic Parish of Victorias of which petitioner
was the parish priest.

Issue:
Whether or not the imposition of gift tax despite the fact the Fr. Lladoc was not the Parish priest at the time of donation,
Catholic Parish priest of Victorias did not have juridical personality as the constitutional exemption for religious purpose
is valid.

Held:
Yes, imposition of the gift tax was valid, under Section 22(3) Article VI of the Constitution contemplates exemption only
from payment of taxes assessed on such properties as Property taxes contra distinguished from Excise taxes The
imposition of the gift tax on the property used for religious purpose is not a violation of the Constitution. A gift tax is not
a property by way of gift inter vivos.

The head of the Diocese and not the parish priest is the real party in interest in the imposition of the donee's tax on the
property donated to the church for religious purpose.
ABRA VALLEY COLLEGE, INC vs Aquino
G.R. No. L-39086, June 15, 1988

FACTS:
On June 8, 1972 the properties of the Abra Valley Junior College, Inc. was sold at public auction for the satisfaction of the
unpaid real property taxes thereon and the same was sold to Paterno Millare who offered the highest bid of P6,000.00
and a Certificate of Sale in his favor was issued by the defendant Municipal Treasurer.

(a) that the school is recognized by the government and is offering Primary, High School and College Courses, and has a
school population of more than one thousand students all in all; (b) that it is located right in the heart of the town of
Bangued, a few meters from the plaza and about 120 meters from the Court of First Instance building; (c) that the
elementary pupils are housed in a two-storey building across the street; (d) that the high school and college students are
housed in the main building; (e) that the Director with his family is in the second floor of the main building; and (f) that
the annual gross income of the school reaches more than one hundred thousand pesos.

The only issue left for the Court to determine and as agreed by the parties, is whether or not the lot and building in
question are used exclusively for educational purposes.

ISSUE:
Whether or not the properties are exclusively for education purposes?

HELD:
Petitioner contends that the primary use of the lot and building for educational purposes, and not the incidental use
thereof, determines and exemption from property taxes under Section 22 (3), Article VI of the 1935 Constitution. Hence,
the seizure and sale of subject college lot and building, which are contrary thereto as well as to the provision of
Commonwealth Act No. 470, otherwise known as the Assessment Law, are without legal basis and therefore void.

On the other hand, private respondents maintain that the college lot and building in question which were subjected to
seizure and sale to answer for the unpaid tax are used: (1) for the educational purposes of the college; (2) as the
permanent residence of the President and Director thereof, Mr. Pedro V. Borgonia, and his family including the in-laws
and grandchildren; and (3) for commercial purposes because the ground floor of the college building is being used and
rented by a commercial establishment, the Northern Marketing Corporation

The phrase “exclusively used for educational purposes” was further clarified by this Court, thus““Moreover, the
exemption in favor of property used exclusively for charitable or educational purposes is ‘not limited to property actually
indispensable’ therefor, but extends to facilities which are incidental to and reasonably necessary for the
accomplishment of said purposes, such as in the case of hospitals, ‘a school for training nurses, a nurses’ home, property
use to provide housing facilities for interns, resident doctors, superintendents, and other members of the hospital staff,
and recreational facilities for student nurses, interns, and residents’ (84 CJS 6621), such as ‘athletic fields’ including ‘a
firm used for the inmates of the institution.’ ”

The exemption extends to facilities which are incidental to and reasonably necessary for the accomplishment of the
main purpose the lease of the first floor to the Northern Marketing Corporation cannot by any stretch of the imagination
be considered incidental to the purposes of education; Case at bar.— It must be stressed however, that while this Court
allows a more liberal and non-restrictive interpretation of the phrase “exclusively used for educational purposes” as
provided for in Article VI, Section 22, paragraph 3 of the 1935 Philippine Constitution, reasonable emphasis has always
been made that exemption extends to facilities which are incidental to and reasonably necessary for the
accomplishment of the main purposes. Otherwise stated, the use of the school building or lot for commercial purposes
is neither contemplated by law, nor by jurisprudence. Thus, while the use of the second floor of the main building in the
case at bar for residential purposes of the Director and his family, may find justification under the concept of incidental
use, which is complimentary to the main or primary pur-pose— educational, the lease of the first floor thereof to the
Northern Marketing Corporation cannot by any stretch of the imagination be considered incidental to the purposes of
education.

Trial Court correct in imposing the tax not because the second floor is being used by the Director and his family for
residential purposes but because the first floor is being used for commercial purposes.— Under the 1935 Constitution,
the trial court correctly arrived at the conclusion that the school building as well as the lot where it is built, should be
taxed, not because the second floor of the same is being used by the Director and his family for residential purposes, but
because the first floor thereof is being used for commercial purposes. However, since only a portion is used for purposes
of commerce, it is only fair that half of the assessed tax be returned to the school involved.
ARTICLE III – THE BILL OF RIGHTS

SECTION 1 DUE PROCESS OF LAW

Ichong vs Hernandez
G.R. No. L-7995 May 31, 1957

FACTS:
Republic Act No. 1180 is entitled "An Act to Regulate the Retail Business." In effect it nationalizes the retail trade
business. The main provisions of the Act are: (1) a prohibition against persons, not citizens of the Philippines, and against
associations, partnerships, or corporations the capital of which are not wholly owned by citizens of the Philippines, from
engaging directly or indirectly in the retail trade; (2) an exception from the above prohibition in favor of aliens actually
engaged in said business on May 15, 1954, who are allowed to continue to engaged therein, unless their licenses are
forfeited in accordance with the law, until their death or voluntary retirement in case of natural persons, and for ten
years after the approval of the Act or until the expiration of term in case of juridical persons; (3) an exception there from
in favor of citizens and juridical entities of the United States; (4) a provision for the forfeiture of licenses for violation of
the laws on nationalization, control weights and measures and labor and other laws relating to trade, commerce and
industry; (5) a prohibition against the establishment or opening by aliens actually engaged in the retail business of
additional stores or branches of retail business, (6) a provision requiring aliens actually engaged in the retail business to
present for registration with the proper authorities a verified statement concerning their businesses, giving, among
other matters, the nature of the business, their assets and liabilities and their offices and principal offices of judicial
entities; and (7) a provision allowing the heirs of aliens now engaged in the retail business who die, to continue such
business for a period of six months for purposes of liquidation.

Petitioner, for and in his own behalf and on behalf of other alien resident,s corporations and partnerships adversely
affected by the provisions of Republic Act. No. 1180, brought this action to obtain a judicial declaration that said Act is
unconstitutional, and to enjoin the Secretary of Finance and all other persons acting under him, particularly city and
municipal treasurers, from enforcing its provisions. Petitioner attacks the constitutionality of the Act, contending that it
denies to alien residents the equal protection of the laws and deprives of their liberty and property without due process
of law.

ISSUE:
Whether or not R.A. No. 1180 denies equal protection of laws and due process?

HELD:
The Court cited the following reason in upholding the constitutionality and validity of R.A. No. 1180 which does not
violate the equal protection of laws and due process.

We hold that the disputed law was enacted to remedy a real actual threat and danger to national economy posed by
alien dominance and control of the retail business and free citizens and country from dominance and control; that the
enactment clearly falls within the scope of the police power of the State, thru which and by which it protects its own
personality and insures its security and future.

The present dominance of the alien retailer, especially in the big centers of population, therefore, becomes a potential
source of danger on occasions of war or other calamity. We do not have here in this country isolated groups of harmless
aliens retailing goods among nationals; what we have are well organized and powerful groups that dominate the
distribution of goods and commodities in the communities and big centers of population. They owe no allegiance or
loyalty to the State, and the State cannot rely upon them in times of crisis or emergency. While the national holds his
life, his person and his property subject to the needs of his country, the alien may even become the potential enemy of
the State.

The law does not violate the equal protection clause of the Constitution because sufficient grounds exist for the
distinction between alien and citizen in the exercise of the occupation regulated. Aliens are under no special
constitutional protection which forbids a classification otherwise justified simply because the limitation of the class falls
along the lines of nationality. That would be requiring a higher degree of protection for aliens as a class than for similar
classes than for similar classes of American citizens. Broadly speaking, the difference in status between citizens and
aliens constitutes a basis for reasonable classification in the exercise of police power.

DUE PROCESS

The due process of law clause is not violated because the law is prospective in operation and recognizes the privilege of
aliens already engaged in the occupation and reasonably protects their privilege; that the wisdom and efficacy of the law
to carry out its objectives appear to us to be plainly evident — as a matter of fact it seems not only appropriate but
actually necessary — and that in any case such matter falls within the prerogative of the Legislature, with whose power
and discretion the Judicial department of the Government may not interfere.

The guaranty of due process demands only that the law shall not be unreasonable, arbitrary or capricious, and that the
means selected shall have a real and substantial relation to the subject sought to be attained.

So far as the requirement of due process is concerned and in the absence of other constitutional restriction a state is
free to adopt whatever economic policy may reasonably be deemed to promote public welfare, and to enforce that
policy by legislation adapted to its purpose. The courts are without authority either to declare such policy, or, when it is
declared by the legislature, to override it. If the laws passed are seen to have a reasonable relation to a proper
legislative purpose, and are neither arbitrary nor discriminatory, the requirements of due process are satisfied, and
judicial determination to that effect renders a court functus officio. . . .

To justify the state in thus interposing its authority in behalf of the public, it must appear, first, that the interests of the
public generally, as distinguished from those of a particular class, require such interference; and second, that the means
are reasonably necessary for the accomplishment of the purpose, and not unduly oppressive upon individuals. The real
question at issue, therefore, is not that posed by petitioner, which overlooks and ignores the facts and circumstances,
but this, Is the exclusion in the future of aliens from the retail trade unreasonable?; Arbitrary capricious, taking into
account the illegitimate and pernicious form and manner in which the aliens have heretofore engaged therein? As thus
correctly stated the answer is clear. The law in question is deemed absolutely necessary to bring about the desired
legislative objective, i.e., to free national economy from alien control and dominance. It is not necessarily unreasonable
because it affects private rights and privileges (11 Am. Jur. pp. 1080-1081.) The test of reasonableness of a law is the
appropriateness or adequacy under all circumstances of the means adopted to carry out its purpose into effect (Id.)
Judged by this test, disputed legislation, which is not merely reasonable but actually necessary, must be considered not
to have infringed the constitutional limitation of reasonableness.
PHIL. PHOSPHATE FERTILIZER CORP. VS. TORRES
231 SCRA 335; G.R. NO.98050; 17 MAR 1994

Facts:
Philphos Movement for Progress, Inc. (PMPI for brevity), filed with the Department of Labor and Employment a petition
for certification election among the supervisory employees of petitioner, alleging that as a supervisory union duly
registered with the Department of Labor and Employment it was seeking to represent the supervisory employees of
Philippine Phosphate Fertilizer Corporation. Mediator-Arbiter Rodolfo S. Milado issued an order directing the holding of
a certification election among the supervisory employees of petitioner, excluding therefrom the superintendents and
the professional and technical employees. However, the PMPI filed an amended petition with the Mediator-Arbiter
wherein it sought to represent not only the supervisory employees of petitioner but also its professional/technical and
confidential employees. The parties therein agreed to submit their respective position papers and to consider the
amended petition submitted for decision on the basis thereof and related documents. Mediator-Arbiter Milado issued
an order granting the petition and directing the holding of a certification election among the "supervisory, professional
(engineers, analysts, mechanics, accountants, nurses, midwives, etc.), technical, and confidential employees. PHILPHOS
appealed the order to the Secretary of Labor and Employment who rendered a decision through Undersecretary
Bienvenido Laguesma dismissing the appeal. PHILPHOS moved for reconsideration but the same was denied; hence, the
instant petition alleging denial of due process on the part of the DOLE to which the mediator-arbiter was under.

Issue:
Whether or Not there was denial of due process.

Held:
There was no denial of due process. The essence of due process is simply an opportunity to be heard or, as applied to
administrative proceedings, an opportunity to explain one's side or an opportunity to seek a reconsideration of the
action or ruling complained of petitioner PHILPHOS agreed to file its position paper with the Mediator-Arbiter and to
consider the case submitted for decision on the basis of the position papers filed by the parties, there was sufficient
compliance with the requirement of due process, as petitioner was afforded reasonable opportunity to present its side.
Moreover, petitioner could have, if it so desired, insisted on a hearing to confront and examine the witnesses of the
other party. But it did not; instead it opted to submit its position paper with the Mediator-Arbiter. Besides, petitioner
had all the opportunity to ventilate its arguments in its appeal to the Secretary of Labor
YNOT VS. IAC
148 SCRA 659; G.R. NO. 74457; 20 MAR 1987

Facts:
Executive Order No. 626-A prohibited the transportation of carabaos and carabeef from one province to another. The
carabaos of petitioner were confiscated for violation of Executive Order No 626-A while he was transporting them from
Masbate to Iloilo. Petitioner challenged the constitutionality of Executive Order No. 626-A. The government argued that
Executive Order No. 626-A was issued in the exercise of police power to conserve the carabaos that were still fit for farm
work or breeding.

Issue:
Whether or Not EO No. 626-A is a violation of Substantive Due Process.

Held:
The challenged measure is an invalid exercise of police power, because it is not reasonably necessary for the purpose of
the law and is unduly oppressive. It is difficult to see how prohibiting the transfer of carabaos from one province to
another can prevent their indiscriminate killing. Retaining the carabaos in one province will not prevent their slaughter
there. Prohibiting the transfer of carabeef, after the slaughter of the carabaos, will not prevent the slaughter either.
ALONTE VS. SAVELLANO
G.R. No. 131652, March 9, 1998

Facts:
Alonte was accused of raping JuvieLyn Punongbayan with accomplice Buenaventura Concepcion. It was alleged that
Concepcion befriended Juvie and had later lured her into Alonete’s house who was then the mayor of Biňan, Laguna.
The case was brought before RTC Biňan. The counsel and the prosecutor later moved for a change of venue due to
alleged intimidation. While the change of venue was pending, Juvie executed an affidavit of desistance. The prosecutor
continued on with the case and the change of venue was done notwithstanding opposition from Alonte. The case was
raffled to the Manila RTC under J Savellano. Savellano later found probable cause and had ordered the arrest of Alonte
and Concepcion. Thereafter, the prosecution presented Juvie and had attested the voluntariness of her desistance the
same being due to media pressure and that they would rather establish new life elsewhere. Case was then submitted for
decision and Savellano sentenced both accused to reclusion perpetua. Savellano commented that Alonte waived his
right to due process when he did not cross examine Juvie when clarificatory questions were raised about the details of
the rape and on the voluntariness of her desistance.

ISSUE:
Whether or not Alonte has been denied criminal due process.

HELD:
The SC ruled that Savellano should inhibit himself from further deciding on the case due to animosity between him and
the parties. There is no showing that Alonte waived his right. The standard of waiver requires that it “not only must be
voluntary, but must be knowing, intelligent, and done with sufficient awareness of the relevant circumstances and likely
consequences.” Mere silence of the holder of the right should not be so construed as a waiver of right, and the courts
must indulge every reasonable presumption against waiver. Savellano has not shown impartiality by repeatedly not
acting on numerous petitions filed by Alonte. The case is remanded to the lower court for retrial and the decision earlier
promulgated is nullified.
ANIAG VS. COMELEC
237 SCRA 194; G.R. NO. 104961; 7 OCT 1994

Facts:
In preparation for the synchronized national and local elections, the COMELEC issued Resolution No. 2323, “Gun Ban”,
promulgating rules and regulations on bearing, carrying and transporting of firearm or other deadly weapons on security
personnel or bodyguards, on bearing arms by members of security agencies or police organizations, and organization or
maintenance of reaction forces during the election period. COMELEC also issued Resolution No. 2327 providing for the
summary disqualification of candidates engaged in gunrunning, using and transporting of firearms, organizing special
strike forces, and establishing spot checkpoints. Pursuant to the “Gun Ban”, Mr. Serrapio Taccad, Sergeant at Arms of
the House of Representatives, wrote petitioner for the return of the two firearms issued to him by the House of
Representatives. Petitioner then instructed his driver, Arellano, to pick up the firearms from petitioner’s house and
return them to Congress. The PNP set up a checkpoint. When the car driven by Arellano approached the checkpoint, the
PNP searched the car and found the firearms. Arellano was apprehended and detained. He then explained the order of
petitioner. Petitioner also explained that Arellano was only complying with the firearms ban, and that he was not a
security officer or a bodyguard. Later, COMELEC issued Resolution No.92-0829 directing the filing of information against
petitioner and Arellano for violation of the Omnibus Election Code, and for petitioner to show cause why he should not
be disqualified from running for an elective position. Petitioner then questions the constitutionality of Resolution No.
2327. He argues that “gunrunning, using or transporting firearms or similar weapons” and other acts mentioned in the
resolution are not within the provisions of the Omnibus Election Code. Thus, according to petitioner, Resolution No.
2327 is unconstitutional. The issue on the disqualification of petitioner from running in the elections was rendered moot
when he lost his bid for a seat in Congress in the elections.

Issue:
Whether or Not petitioner can be validly prosecuted for instructing his driver to return the firearms issued to him on the
basis of the evidence gathered from the warrant less search of his car

Held:
A valid search must be authorized by a search warrant issued by an appropriate authority. However, a warrantless
search is not violative of the Constitution for as long as the vehicle is neither searched nor its occupants subjected to a
body search, and the inspection of the vehicle is merely limited to a visual search. In the case at bar, the guns were not
tucked in Arellano’s waist nor placed within his reach, as they were neatly packed in gun cases and placed inside a bag at
the back of the car. Given these circumstances, the PNP could not have thoroughly searched the car lawfully as well as
the package without violating the constitutional injunction. Absent any justifying circumstance specifically pointing to
the culpability of petitioner and Arellano, the search could not have been valid. Consequently, the firearms obtained
from the warrantless search cannot be admitted for any purpose in any proceeding. It was also shown in the facts that
the PNP had not informed the public of the purpose of setting up the checkpoint. Petitioner was also not among those
charged by the PNP with violation of the Omnibus Election Code. He was not informed by the City Prosecutor that he
was a respondent in the preliminary investigation. Such constituted a violation of his right to due process. Hence, it
cannot be contended that petitioner was fully given the opportunity to meet the accusation against him as he was not
informed that he was himself a respondent in the case. Thus, the warrantless search conducted by the PNP is declared
illegal and the firearms seized during the search cannot be used as evidence in any proceeding against the petitioner.
Resolution No. 92-0829 is unconstitutional, and therefore, set aside.
Philcomsat v. Alcuaz
GR 84818, 18 December 1989

Facts:
By virtue of Republic Act 5514, the Philippine Communications Satellite Corporation (PHILCOMSAT) was granted "a
franchise to establish, construct, maintain and operate in the Philippines, at such places as the grantee may select,
station or stations and associated equipment and facilities for international satellite communications," the authority to
"construct and operate such ground facilities as needed to deliver telecommunications services from the
communications satellite system and ground terminal or terminals." By designation of the Republic of the Philippines, it
is also the sole signatory for the Philippines in the Agreement and the Operating Agreement relating to the International
Telecommunications
Satellite Organization (INTELSAT), as well as in the Convention and the Operating Agreement of the
International Maritime Satellite Organization (INMARSAT), which two global commercial telecommunications satellite
corporations were collectively established by various states in line with the principles set forth in Resolution 1721 (XVI)
of the United Nations’s General Assembly. Since 1968, It has been leasing its satellite circuits to PLDT, Philippine Global
Communications, Eastern Telecom, Globe Mackay Cable and Radio Corp. ITT, and Capitol Wireless or their predecessors-
in-interest. The satellite services thus provided by PHILCOMSAT enable said international carriers to serve the public
with indispensable communication services, such as overseas telephone, telex, facsimile, telegrams, high speed data,
live television in full color, and television standard conversion from European to American or vice versa.
It was exempt from the jurisdiction of the then Public Service Commission, now National Telecommunications
Commission (NTC). However, pursuant to Executive Order (EO) 196 issued on 17 June 1987, it was placed under the
jurisdiction, control and regulation of NTC, including all its facilities and services and the fixing of rates. Implementing
said executive order, NTC required PHILCOMSAT to apply for the requisite certificate of public convenience and
necessity covering its facilities and the services it renders, as well as the corresponding authority to charge rates
therefor. On 9 September 1987, PHILCOMSAT tiled with NTC an application for authority to continue operating and
maintaining the same facilities it has been continuously operating and maintaining since 1967, to continue providing the
international satellite communications services it has likewise been providing since 1967, and to charge the current rates
applied for in rendering such services. Pending hearing, it also applied for a provisional authority so that it can continue
to operate and maintain the facilities, provide the services and charge therefor the aforesaid rates therein applied for.
On 16 September 1987, PHILCOMSAT was granted a provisional authority to continue operating its existing facilities, to
render the services it was then offering, and to charge the rates it was then charging.
This authority was valid for 6 months from the date of said order. When said provisional authority expired on 17 March
1988, it was extended for another 6 months, or up to 16 September 1988. Thereafter, the NTC further extended the
provisional authority of PHILCOMSAT for another 6 months, counted from 16 September 1988, but it directed
PHILCOMSAT to charge modified reduced rates through a reduction of 15% on the present authorized rates.
PHILCOMSAT assailed said order.

Issue:
Whether the NTC is not required to provide notice and hearing to PHILCOMSAT in its rate-fixing order, which fixed a
temporary rate pending final determination of PHILCOMSAT’s application.

Held:
The NTC, in the exercise of its rate-fixing power, is limited by the requirements of public safety, public interest,
reasonable feasibility and reasonable rates, which conjointly more than satisfy the requirements of a valid delegation of
legislative power. The NTC order violates procedural due process because it was issued motu proprio, without notice to
PHILCOMSAT and without the benefit of a hearing. Said order was based merely on an "initial evaluation," which is a
unilateral evaluation, but had PHILCOMSAT been given an opportunity to present its side before the order in question
was issued, the confiscatory nature of the rate reduction and the consequent deterioration of the public service could
have been shown and demonstrated to NTC. The order pertains exclusively to PHILCOMSAT and to no other. Reduction
of rates was made without affording PHILCOMSAT the benefit of an explanation as to what particular aspect or aspects
of the financial statements warranted a corresponding rate reduction. PHILCOMSAT was not even afforded the
opportunity to cross-examine the inspector who issued the report on which NTC based its questioned order. While the
NTC may fix a temporary rate pending final determination of the application of PHILCOMSAT, such ratefixing order,
temporary though it may be, is not exempt from the statutory procedural requirements of notice and hearing, as well as
the requirement of reasonableness. Assuming that such power is vested in NTC, it may not exercise the same in an
arbitrary and confiscatory manner. Categorizing such an order as temporary in nature does not perforce entail the
applicability of a different rule of statutory procedure than would otherwise be applied to any other order on the same
matter unless otherwise provided by the applicable law. NTC has no authority to make such order without first giving
PHILCOMSAT a hearing, whether the order be temporary or permanent, and it is immaterial whether the same is made
upon a complaint, a summary investigation, or upon the commission's own motion.
ANG TIBAY VS. COURT OF INDUSTRIAL RELATIONS (CIR)
69 PHIL 635; G.R. NO. 46496; 27 FEB 1940

Facts:
There was agreement between Ang Tibay and the National Labor Union, Inc (NLU). The NLU alleged that the supposed
lack of leather material claimed by Toribio Teodoro was but a scheme adopted to systematically discharge all the
members of the NLU, from work. And this averment is desired to be proved by the petitioner with the records of the
Bureau of Customs and Books of Accounts of native dealers in leather. That National Worker's Brotherhood Union of
Ang Tibay is a company or employer union dominated by Toribio Teodoro, which was alleged by the NLU as an illegal
one. The CIR, decided the case and elevated it to the Supreme Court, but a motion for new trial was raised by the NLU.
But the Ang Tibay filed a motion for opposing the said motion.

Issue:
Whether or Not, the motion for new trial is meritorious to be granted.

Held:
To begin with the issue before us is to realize the functions of the CIR. The CIR is a special court whose functions are
specifically stated in the law of its creation which is the Commonwealth Act No. 103). It is more an administrative board
than a part of the integrated judicial system of the nation. It is not intended to be a mere receptive organ of the
government. Unlike a court of justice which is essentially passive, acting only when its jurisdiction is invoked and
deciding only cases that are presented to it by the parties litigant, the function of the CIR, as will appear from perusal of
its organic law is more active, affirmative and dynamic. It not only exercises judicial or quasi-judicial functions in the
determination of disputes between employers and employees but its functions are far more comprehensive and
extensive. It has jurisdiction over the entire Philippines, to consider, investigate, decide, and settle any question, matter
controversy or disputes arising between, and/ or affecting employers and employees or laborers, and landlords and
tenants or farm-laborers, and regulates the relations between them, subject to, and in accordance with, the provisions
of CA 103.

As laid down in the case of Goseco v. CIR, the SC had the occasion to point out that the CIR is not narrowly constrained
by technical rules of procedure, and equity and substantial merits of the case, without regard to technicalities or legal
forms and shall not be bound by any technical rules of legal evidence but may inform its mind in such manner as it may
deem just and equitable.

The fact, however, that the CIR may be said to be free from rigidity of certain procedural requirements does not mean
that it can in justiciable cases coming before it, entirely ignore or disregard the fundamental and essential requirements
of due process in trials and investigations of an administrative character. There cardinal primary rights which must be
respected even in proceedings of this character:

(1) the right to a hearing, which includes the right to present one's cause and submit evidence in support thereof;
(2) The tribunal must consider the evidence presented;
(3) The decision must have something to support itself;
(4) The evidence must be substantial;
(5) The decision must be based on the evidence presented at the hearing; or at least contained in the record and
disclosed to the parties affected;
(6) The tribunal or body or any of its judges must act on its own independent consideration of the law and facts of
the controversy, and not simply accept the views of a subordinate;
(7) The Board or body should, in all controversial questions, render its decision in such manner that the parties to
the proceeding can know the various Issue involved, and the reason for the decision rendered.
The failure to grasp the fundamental issue involved is not entirely attributable to the parties adversely affected by the
result. Accordingly, the motion for a new trial should be, and the same is hereby granted, and the entire record of this
case shall be remanded to the CIR, with instruction that it reopen the case receive all such evidence as may be relevant,
and otherwise proceed in accordance with the requirements set forth. So ordered.
ATENEO DE MANILA UNIVERSITY VS. HON. JUDGE IGNACIO CAPULONG
222 SCRA 644; G.R. 99327; 27 MAY 1993

Facts:
Leonardo H. Villa, a first year law student of Petitioner University, died of serious physical injuries at Chinese General
Hospital after the initiation rites of Aquila Legis. Bienvenido Marquez was also hospitalized at the Capitol Medical Center
for acute renal failure occasioned by the serious physical injuries inflicted upon him on the same occasion. Petitioner
Dean Cynthia del Castillo created a Joint Administration-Faculty-Student Investigating Committee which was tasked to
investigate and submit a report within 72 hours on the circumstances surrounding the death of Lennie Villa. Said notice
also required respondent students to submit their written statements within twenty-four (24) hours from receipt.
Although respondent students received a copy of the written notice, they failed to file a reply. In the meantime, they
were placed on preventive suspension. The Joint Administration-Faculty-Student Investigating Committee, after
receiving the written statements and hearing the testimonies of several witness, found a prima facie case against
respondent students for violation of Rule 3 of the Law School Catalogue entitled "Discipline." Respondent students were
then required to file their written answers to the formal charge. Petitioner Dean created a Disciplinary Board to hear the
charges against respondent students. The Board found respondent students guilty of violating Rule No. 3 of the Ateneo
Law School Rules on Discipline which prohibits participation in hazing activities. However, in view of the lack of
unanimity among the members of the Board on the penalty of dismissal, the Board left the imposition of the penalty to
the University Administration. Accordingly, Fr. Bernas imposed the penalty of dismissal on all respondent students.
Respondent students filed with RTC Makati a TRO since they are currently enrolled. This was granted. A TRO was also
issued enjoining petitioners from dismissing the respondents. A day after the expiration of the temporary restraining
order, Dean del Castillo created a Special Board to investigate the charges of hazing against respondent students Abas
and Mendoza. This was requested to be stricken out by the respondents and argued that the creation of the Special
Board was totally unrelated to the original petition which alleged lack of due process. This was granted and
reinstatement of the students was ordered.

Issue:
Was there denial of due process against the respondent students.

Held:
There was no denial of due process, more particularly procedural due process. Dean of the Ateneo Law School, notified
and required respondent students to submit their written statement on the incident. Instead of filing a reply,
respondent students requested through their counsel, copies of the charges. The nature and cause of the accusation
were adequately spelled out in petitioners' notices. Present is the twin elements of notice and hearing.
Respondent students argue that petitioners are not in a position to file the instant petition under Rule 65 considering
that they failed to file a motion for reconsideration first before the trial court, thereby by passing the latter and the
Court of Appeals. It is accepted legal doctrine that an exception to the doctrine of exhaustion of remedies is when the
case involves a question of law, as in this case, where the issue is whether or not respondent students have been
afforded procedural due process prior to their dismissal from Petitioner University.
Minimum standards to be satisfied in the imposition of disciplinary sanctions in academic institutions, such as petitioner
university herein, thus:
(1) the students must be informed in writing of the nature and cause of any accusation against them;
(2) that they shall have the right to answer the charges against them with the assistance of counsel, if desired:
(3) they shall be informed of the evidence against them
(4) they shall have the right to adduce evidence in their own behalf; and
(5) the evidence must be duly considered by the investigating committee or official designated by the school
authorities to hear and decide the case.
First Class Cadet Aldrin Jeff Cudia v. The Superintendent of the Philippine Military Academy
GR Number 211362,February 24, 2014

Facts:
Petitioner, Cadet First Class Cudia, was a member of the Siklab Diwa Class of 2014 of the Philippine Military Academy. He
was supposed to graduate with honors as the class salutatorian, receive the Philippine Navy Saber as the top Navy Cadet
graduate and be commissioned as an ensign of the Navy.
Petitioner was issued a Delinquency Report (DR) because he was late for two minutes in his ENG 412 class, other cadets
were also reported late for 5 minutes. The DRs reached the Department of Tactical Officers and were logged and
transmitted to the Company of Tactical Officers (TCO) for explanation. Cudia incurred the penalty of 11 demerits and 13
touring hours.
Several days after, Cudia was reported to the Honor Committee (HC) per violation of the Honor Code. Lying that is giving
statements that perverts the truth in his written appeal stating that his 4th period class ended at 3:00 that made him
late for the succeeding class.
Cudia submitted his letter of explanation on the honor report. The HC constituted a team to conduct the preliminary
investigation on the violation, it recommended the case be formalized. Cudia pleaded not guilty. The result was 8-1
guilty verdict and upon the order of the Chairman, the HC reconvened in the chambers, after, the Presiding Officer
announced a 9-0 guilty verdict.
The HC denied Cudia’s appeal. The Headquarters Tactics Group (HTG) conducted a formal review and checking of
findings. Special orders were issued placing Cudia on indefinite leave of absence and pending approval of separation
from the Armed Forces of the Philippines. Cudia submitted a letter to the Office of the Commandant of Cadets
requesting his re-instatement. The matter was referred to Cadet Review and Appeals Board (CRAB) and it upheld the
decision.
Cudia wrote a letter to President Aquino but the President sustained the findings of the CRAB. CHR-CAR issued a
resolution finding probable cause for Human Rights Violations.

Issue:
1. Whether or not the PMA committed grave abuse of discretion in dismissing Cudia in utter disregard of his right
to due process and in holding that he violated the Honor Code through lying.
2. Whether or not the court can interfere with military affairs

Ruling:
1. No. The determination of whether the PMA cadet has rights to due process, education, and property should be
placed in the context of the Honor Code. All the administrative remedies were exhausted. A student of a military
academy must be prepared to subordinate his private interest for the proper functioning of the institution. The PMA
may impose disciplinary measures and punishments as it deems fit and consistent with the peculiar needs of the
institution. PMA has regulatory authority to administratively dismiss erring cadets. PMA has a right to invoke academic
freedom in the enforcement of the internal rules and regulations.
2. Yes. The court is part of the checks-and-balance machinery mandated by Article VIII of the Constitution. The court’s
mandate (according to Section 1, Article 8) is expanded that the duty of the courts is not only to “settle actual
controversies involving rights which are legally demandable and enforceable” but also “to determine whether or not
there has been a grave abuse of discretion on the part of any branch or instrumentality of the Government” even if the
latter does not exercise judicial, quasi-judicial, or ministerial functions. No one is above the law, including the military,
especially in violations of Constitutionally guaranteed rights.

Dispositive:
The petition is denied. The dismissal of Cudia from PMA is affirmed.
SECTION 1 EQUAL PROTECTION OF THE LAWS

PEOPLE VS. VERA


NOVEMBER 16, 1937

The People of the Philippine Islands and the Hong Kong & Shanghai Banking Corporation – petitioners
Jose O. Vera and Mariano Cu Unjieng – respondents

Facts:
1. Hon. Jose O. Vera, is the Judge ad interim of the seventh branch of the Court of First Instance of Manila, who heard
the application of Cu Unjieng for probation in criminal case 42649. The information in the said criminal case was
filed with the CFI on 15 October 1931, HSBC intervening in the case as private prosecutor. The CFI, on 8 January
1934, rendered a judgment of conviction sentencing Cu Unjieng to indeterminate penalty ranging from 4 years and 2
months of prision correccional to 8 years of prision mayor, to pay the costs and with reservation of civil action to the
offended party, HSBC.
2. Upon appeal, the court, on 26 March 1935, modified the sentence to an indeterminate penalty of from 5 years and 6
months of prision correccional to 7 years, 6 months and 27 days of prision mayor, but affirmed the judgment in all
other respects.
3. Cu Unjieng filed a motion for reconsideration and four successive motions for new trial which were denied on 17
December 1935, and final judgment was accordingly entered on 18 December 1935. Cu Unjieng thereupon sought to
have the case elevated on certiorari to the Supreme Court of the United States but the latter denied the petition for
certiorari in November, 1936.
4. The Supreme Court, on 24 November 1936, denied the petition subsequently filed by Cu Unjieng for leave to file a
second alternative motion for reconsideration or new trial and thereafter remanded the case to the court of origin
for execution of the judgment.
5. Cu Unjieng filed an application for probation on 27 November 1936, before the trial court, under the provisions of
Act 4221 of the defunct Philippine Legislature. Cu Unjieng states in his petition, inter alia, that he is innocent of the
crime of which he was convicted, that he has no criminal record and that he would observe good conduct in the
future.
6. Thereafter, the CFI of Manila, seventh branch, Judge Jose O. Vera presiding, set the petition for hearing on 5 April
1937. On 2 April 1937, the Fiscal of the City of Manila filed an opposition to the granting of probation to Cu Unjieng.

ISSUE/S:
a. Whether or not the constitutionality of Act No. 4221 has been properly raised in the proceedings OR Whether
the People of the Philippines, through the Solicitor General and Fiscal of the City of Manila, is a proper party in
present case
b. Whether or not the said Act is constitutional

Wherein the constitutionality of said act is challenged on these three grounds


1. The said Act encroaches upon the pardoning power of the Executive
2. That it constitutes an undue delegation of legislative power
3. That it denies the equal protection of the laws

HELD:
a. YES. The people of the Philippines, represented by the Solicitor-General and the Fiscal of the City of Manila, is such a
proper party in the present proceedings. It goes without saying that if Act No. 4221 really violates the Constitution,
the People of the Philippines has a substantial interest in having it set aside. Hence, the well-settled rule that the
state can challenge the validity of its own laws.
b. NO. Said act is unconstitutional

Grounds:
1. No, the Act does not conflict with the pardoning power of the Executive. The pardoning power, in respect to those
serving their probationary sentences remains as full and complete as if the Probation Law – pardoning is different from
suspension
2. Yes, the Act constitutes an undue delegation of legislative power. Section 11 of the Act says that the provincial
boards of the various provinces are to determine for themselves, whether the Probation Law shall apply to their
provinces or not at all. But law states that the power of suspending laws shall be exercised by solely the legislature.
3. Yes, the act denies the equal protection of laws. The resultant inequality may be said to flow from the previously
mentioned unwarranted delegation of legislative power. One province may adopt the law, another might not. This
means that a person coming within the purview of law would be liable to enjoy the benefits of probation in one
province, while another person similarly situated in another province would be denied those benefits. –and if a law has
the effect of denying the equal protection of the law, it is unconstitutional.
Ichong vs Hernandez
G.R. No. L-7995 May 31, 1957

FACTS:
Republic Act No. 1180 is entitled "An Act to Regulate the Retail Business." In effect it nationalizes the retail trade
business. The main provisions of the Act are: (1) a prohibition against persons, not citizens of the Philippines, and against
associations, partnerships, or corporations the capital of which are not wholly owned by citizens of the Philippines, from
engaging directly or indirectly in the retail trade; (2) an exception from the above prohibition in favor of aliens actually
engaged in said business on May 15, 1954, who are allowed to continue to engaged therein, unless their licenses are
forfeited in accordance with the law, until their death or voluntary retirement in case of natural persons, and for ten
years after the approval of the Act or until the expiration of term in case of juridical persons; (3) an exception there from
in favor of citizens and juridical entities of the United States; (4) a provision for the forfeiture of licenses for violation of
the laws on nationalization, control weights and measures and labor and other laws relating to trade, commerce and
industry; (5) a prohibition against the establishment or opening by aliens actually engaged in the retail business of
additional stores or branches of retail business, (6) a provision requiring aliens actually engaged in the retail business to
present for registration with the proper authorities a verified statement concerning their businesses, giving, among
other matters, the nature of the business, their assets and liabilities and their offices and principal offices of judicial
entities; and (7) a provision allowing the heirs of aliens now engaged in the retail business who die, to continue such
business for a period of six months for purposes of liquidation.

Petitioner, for and in his own behalf and on behalf of other alien resident,s corporations and partnerships adversely
affected by the provisions of Republic Act. No. 1180, brought this action to obtain a judicial declaration that said Act is
unconstitutional, and to enjoin the Secretary of Finance and all other persons acting under him, particularly city and
municipal treasurers, from enforcing its provisions. Petitioner attacks the constitutionality of the Act, contending that it
denies to alien residents the equal protection of the laws and deprives of their liberty and property without due process
of law.

ISSUE:
Whether or not R.A. No. 1180 denies equal protection of laws and due process?

HELD:
The Court cited the following reason in upholding the constitutionality and validity of R.A. No. 1180 which does not
violate the equal protection of laws and due process.
We hold that the disputed law was enacted to remedy a real actual threat and danger to national economy posed by
alien dominance and control of the retail business and free citizens and country from dominance and control; that the
enactment clearly falls within the scope of the police power of the State, thru which and by which it protects its own
personality and insures its security and future.

The present dominance of the alien retailer, especially in the big centers of population, therefore, becomes a potential
source of danger on occasions of war or other calamity. We do not have here in this country isolated groups of harmless
aliens retailing goods among nationals; what we have are well organized and powerful groups that dominate the
distribution of goods and commodities in the communities and big centers of population. They owe no allegiance or
loyalty to the State, and the State cannot rely upon them in times of crisis or emergency. While the national holds his
life, his person and his property subject to the needs of his country, the alien may even become the potential enemy of
the State.

The law does not violate the equal protection clause of the Constitution because sufficient grounds exist for the
distinction between alien and citizen in the exercise of the occupation regulated. Aliens are under no special
constitutional protection which forbids a classification otherwise justified simply because the limitation of the class falls
along the lines of nationality. That would be requiring a higher degree of protection for aliens as a class than for similar
classes than for similar classes of American citizens. Broadly speaking, the difference in status between citizens and
aliens constitutes a basis for reasonable classification in the exercise of police power.

DUE PROCESS

The due process of law clause is not violated because the law is prospective in operation and recognizes the privilege of
aliens already engaged in the occupation and reasonably protects their privilege; that the wisdom and efficacy of the law
to carry out its objectives appear to us to be plainly evident — as a matter of fact it seems not only appropriate but
actually necessary — and that in any case such matter falls within the prerogative of the Legislature, with whose power
and discretion the Judicial department of the Government may not interfere.

The guaranty of due process demands only that the law shall not be unreasonable, arbitrary or capricious, and that the
means selected shall have a real and substantial relation to the subject sought to be attained.

So far as the requirement of due process is concerned and in the absence of other constitutional restriction a state is
free to adopt whatever economic policy may reasonably be deemed to promote public welfare, and to enforce that
policy by legislation adapted to its purpose. The courts are without authority either to declare such policy, or, when it is
declared by the legislature, to override it. If the laws passed are seen to have a reasonable relation to a proper
legislative purpose, and are neither arbitrary nor discriminatory, the requirements of due process are satisfied, and
judicial determination to that effect renders a court functus officio. . . .

To justify the state in thus interposing its authority in behalf of the public, it must appear, first, that the interests of the
public generally, as distinguished from those of a particular class, require such interference; and second, that the means
are reasonably necessary for the accomplishment of the purpose, and not unduly oppressive upon individuals. The real
question at issue, therefore, is not that posed by petitioner, which overlooks and ignores the facts and circumstances,
but this, Is the exclusion in the future of aliens from the retail trade unreasonable?; Arbitrary capricious, taking into
account the illegitimate and pernicious form and manner in which the aliens have heretofore engaged therein? As thus
correctly stated the answer is clear. The law in question is deemed absolutely necessary to bring about the desired
legislative objective, i.e., to free national economy from alien control and dominance. It is not necessarily unreasonable
because it affects private rights and privileges (11 Am. Jur. pp. 1080-1081.) The test of reasonableness of a law is the
appropriateness or adequacy under all circumstances of the means adopted to carry out its purpose into effect (Id.)
Judged by this test, disputed legislation, which is not merely reasonable but actually necessary, must be considered not
to have infringed the constitutional limitation of reasonableness.
VILLEGAS VS. HIU CHIONG
86 SCRA 270; NO.L-29646; 10 NOV 1978

Facts:
The controverted Ordinance no. 6537 was passed by the Municipal Board of Manila on February 22, 1968 and signed by
Mayor Villegas. It is an ordinance making it unlawful for any person not a citizen of the Philippines to be employed in any
place of employment or to be engaged in any kind of trade business or occupation within the city of Manila without
securing an employment permit from the Mayor of Manila and for other purposes.

Hiu Chiong Tsai Pao Ho, who was employed in Manila filed a petition praying for the writ of preliminary injunction and
restraining order to stop the enforcement of said ordinance.

Issue:
Whether or Not Ordinance no.6537 violates the due process and equal protection clauses of the Constitution.

Held:
It is a revenue measure. The city ordinance which imposes a fee of 50.00 pesos to enable aliens generally to be
employed in the city of Manila is not only for the purpose of regulation.

While it is true that the first part which requires the alien to secure an employment permit from the Mayor involves the
exercise of discretion and judgment in processing and approval or disapproval of application is regulatory in character,
the second part which requires the payment of a sum of 50.00 pesos is not a regulatory but a revenue measure.

Ordinance no. 6537 is void and unconstitutional. This is tantamount to denial of the basic human right of the people in
the Philippines to engaged in a means of livelihood. While it is true that the Philippines as a state is not obliged to admit
aliens within it's territory, once an alien is admitted he cannot be deprived of life without due process of law. This
guarantee includes the means of livelihood. Also it does not lay down any standard to guide the City Mayor in the
issuance or denial of an alien employment permit fee.
DUMLAO VS. COMELEC
95 SCRA 392; L-52245; 22 JAN 1980

Facts:
Petitioner Dumlao questions the constitutionality of Sec. 4 of Batas Pambansa Blg 52 as discriminatory and contrary to
equal protection and due process guarantees of the Constitution. Sec. 4 provides that any retired elective provicial or
municipal official who has received payments of retirement benefits and shall have been 65 years of age at the
commencement of the term of office to which he seeks to be elected, shall not be qualified to run for the same elective
local office from which he has retired. According to Dumlao, the provision amounts to class legislation. Petitioners Igot
and Salapantan Jr. also assail the validity of Sec. 4 of Batas Pambansa Blg 52, which states that any person who has
committed any act of disloyalty to the State, including those amounting to subversion, insurrection, rebellion, or other
similar crimes, shall not be qualified for any of the offices covered by the act, or to participate in any partisan activity
therein: provided that a judgment of conviction of those crimes shall be conclusive evidence of such fact and the filing of
charges for the commission of such crimes before a civil court or military tribunal after preliminary investigation shall be
prima facie evidence of such fact.

Issue:
Whether or not the aforementioned statutory provisions violate the Constitution and thus, should be declared null and
void

Held:
In regards to the unconstitutionality of the provisions, Sec. 4 of BP Blg 52 remains constitutional and valid. The
constitutional guarantee of equal protection of the laws is subject to rational classification. One class can be treated
differently from another class. In this case, employees 65 years of age are classified differently from younger employees.
The purpose of the provision is to satisfy the “need for new blood” in the workplace. In regards to the second paragraph
of Sec. 4, it should be declared null and void for being violative of the constitutional presumption of innocence
guaranteed to an accused. “Explicit is the constitutional provision that, in all criminal prosecutions, the accused shall be
presumed innocent until the contrary is proved, and shall enjoy the right to be heard by himself and counsel (Article IV,
section 19, 1973 Constitution). An accusation, according to the fundamental law, is not synonymous with guilt. The
challenged proviso contravenes the constitutional presumption of innocence, as a candidate is disqualified from running
for public office on the ground alone that charges have been filed against him before a civil or military tribunal. It
condemns before one is fully heard. In ultimate effect, except as to the degree of proof, no distinction is made between
a person convicted of acts of dislotalty and one against whom charges have been filed for such acts, as both of them
would be ineligible to run for public office. A person disqualified to run for public office on the ground that charges have
been filed against him is virtually placed in the same category as a person already convicted of a crime with the penalty
of arresto, which carries with it the accessory penalty of suspension of the right to hold office during the term of the
sentence (Art. 44, Revised Penal Code).”

And although the filing of charges is considered as but prima facie evidence, and therefore, may be rebutted, yet. there
is "clear and present danger" that because of the proximity of the elections, time constraints will prevent one charged
with acts of disloyalty from offering contrary proof to overcome the prima facie evidence against him.
Additionally, it is best that evidence pro and con of acts of disloyalty be aired before the Courts rather than before an
administrative body such as the COMELEC. A highly possible conflict of findings between two government bodies, to the
extreme detriment of a person charged, will thereby be avoided. Furthermore, a legislative/administrative
determination of guilt should not be allowed to be substituted for a judicial determination.
Being infected with constitutional infirmity, a partial declaration of nullity of only that objectionable portion is
mandated. It is separable from the first portion of the second paragraph of section 4 of Batas Pambansa Big. 52 which
can stand by itself.
Wherefore, the first paragraph of section 4 of Batas pambansa Bilang 52 is hereby declared valid and that portion of the
second paragraph of section 4 of Batas Pambansa Bilang 52 is hereby declared null and void, for being violative of the
constitutional presumption of innocence guaranteed to an accused.
PASEI [Philippine Association of Service Exporters Inc] v. Drilon
GR L-81958, 30 June 1988

Facts:
The Philippine Association of Service Exporters, Inc. (PASEI) is a firm "engaged principally in the recruitment of Filipino
workers, male and female, for overseas placement." It challenged the Constitutional validity of DOLE’s Department
Order 1 (series of 1988), in the character of "Guidelines Governing the Temporary Suspension of Deployment of Filipino
Domestic and Household Workers," in a petition for certiorari and prohibition. The measure is assailed (1) for
"discrimination against males or females;" that it "does not apply to all Filipino workers but only to domestic helpers and
females with similar skills;" (2) for being violative of the right to travel, and (3) for being an invalid exercise of the
lawmaking power, police power being legislative, and not executive, in character. PASEI also invoked Section 3 of Article
XIII of the Constitution providing for worker participation "in policy and decision-making processes affecting their rights
and benefits as may be provided by law as Department Order No. 1, as contended, was passed in the absence of prior
consultations. It also claimed that it violated the Charter's non-impairment clause, in addition to the "great and
irreparable injury" that PASEI members face should the Order be further enforced. On 25 May 1988, the Solicitor
General, on behalf of the Secretary of Labor and Administrator of the POEA, filed a Comment informing the Court that
on 8 March 1988, the Labor Secretary lifted the deployment ban in the states of Iraq, Jordan, Qatar, Canada, Hongkong,
United States, Italy, Norway, Austria, and Switzerland. In submitting the validity of the challenged "guidelines," the
Solicitor General invokes the police power of the Philippine State.

Issue:
Whether Department Order 1 unduly discriminates against women.

Held:
Department Order 1 applies only to "female contract workers," but it does not thereby make an undue discrimination
between the sexes. ‘Equality before the law" under the Constitution does not import a perfect identity of rights among
all men and women. It admits of classifications, provided that (1) such classifications rest on substantial distinctions; (2)
they are germane to the purposes of the law; (3) they are not confined to existing conditions; and (4) they apply equally
to all members of the same class. The classification made — the preference for female workers — rests on substantial
distinctions. The sordid tales of maltreatment suffered by migrant Filipina workers, even rape and various forms of
torture, confirmed by testimonies of returning workers, are compelling motives for urgent Government action. As
precisely the caretaker of Constitutional rights, the Court is called upon to protect victims of exploitation. In fulfilling
that duty, the Court sustains the Government's efforts. There is no evidence that, except perhaps for isolated instances,
Filipino men abroad have been afflicted with an identical predicament. Discrimination in this case is justified.
Further, the impugned guidelines are applicable to all female domestic overseas workers, not all Filipina
workers. Had the ban been given universal applicability, then it would have been unreasonable and arbitrary, due to the
fact that not all of them are similarly circumstanced. What the Constitution prohibits is the singling out of a select
person or group of persons within an existing class, to the prejudice of such a person or group or resulting in an unfair
advantage to another person or group of persons. Where the classification is based on such distinctions that make a real
difference as infancy, sex, and stage of civilization of minority groups, the better rule is to recognize its validity only if the
young, the women, and the cultural minorities are singled out for favorable treatment.
International School Alliance of Educators (ISAE) vs. Quisumbing
GR 128845, 1 June 2000

Facts:
The International School, Inc., pursuant to Presidential Decree 732, is a domestic educational institution established
primarily for dependents of foreign diplomatic personnel and other temporary residents. To enable the School to
continue carrying out its educational program and improve its standard of instruction, Section 2(c) of the same decree
authorizes the School to employ its own teaching and management personnel selected by it either locally or abroad,
from Philippine or other nationalities, such personnel being exempt from otherwise applicable laws and regulations
attending their employment, except laws that have been or will be enacted for the protection of employees.
Accordingly, the School hires both foreign and local teachers as members of its faculty, classifying the same into two: (1)
foreign-hires and (2) local-hires. The School employs four tests to determine whether a faculty member should be
classified as a foreign-hire or a local hire, i.e. (a) What is one's domicile? (b) Where is one's home economy? (c) To which
country does one owe economic allegiance? (d) Was the individual hired abroad specifically to work in the School and
was the School responsible for bringing that individual to the Philippines? The School grants foreign-hires certain
benefits not accorded local-hires. These include housing, transportation, shipping costs, taxes, and home leave travel
allowance. Foreign-hires are also paid a salary rate 25% more than local-hires.
The School justifies the difference on two "significant economic disadvantages" foreign-hires have to endure, namely: (a)
the "dislocation factor" and (b) limited tenure. The compensation scheme is simply the School's adaptive measure to
remain competitive on an international level in terms of attracting competent professionals in the field of international
education. The compensation package given to local-hires has been shown to apply to all, regardless of race. There are
foreigners who have been hired locally and who are paid equally as Filipino local hires. When negotiations for a new
collective bargaining agreement were held on June 1995, the International School Alliance of Educators (ISAE), "a
legitimate labor union and the collective bargaining representative of all faculty members" of the School, contested the
difference in salary rates between foreign and local-hires. This issue, as well as the question of whether foreign-hires
should be included in the appropriate bargaining unit, eventually caused a deadlock between the parties. On 7
September 1995, ISAE filed a notice of strike. The failure of the National Conciliation and Mediation Board to bring the
parties to a compromise prompted the Department of Labor and Employment (DOLE) to assume jurisdiction over the
dispute. On 10 June 1996, the DOLE Acting Secretary, Crescenciano B. Trajano, issued an Order resolving the parity and
representation issues in favor of the School. Then DOLE Secretary Leonardo A. Quisumbing subsequently denied ISAE's
motion for reconsideration in an Order dated 19 March 1997. ISAE sought relief from the Supreme Court.

Issue:
Whether the School unduly discriminated against the local-hires.

Held:
That public policy abhors inequality and discrimination is beyond contention. Our Constitution and laws reflect the
policy against these evils. The Constitution in the Article on Social Justice and Human Rights exhorts Congress to "give
highest priority to the enactment of measures that protect and enhance the right of all people to human dignity, reduce
social, economic, and political inequalities." The very broad Article 19 of the Civil Code requires every person, "in the
exercise of his rights and in the performance of his duties, [to] act with justice, give everyone his due, and observe
honesty and good faith. International law, which springs from general principles of law, likewise proscribes
discrimination. The Universal Declaration of Human Rights, the International Covenant on Economic, Social, and Cultural
Rights, the International Convention on the Elimination of All Forms of Racial Discrimination, the Convention against
Discrimination in Education, the Convention (No. 111) Concerning Discrimination in Respect of Employment and
Occupation 16 — all embody the general principle against discrimination, the very antithesis of fairness and justice. The
Philippines, through its Constitution, has incorporated this principle as part of its national laws. In the workplace, where
the relations between capital and labor are often skewed in favor of capital, inequality and discrimination by the
employer are all the more reprehensible. If an employer accords employees the same position and rank, the
presumption is that these employees perform equal work. This presumption is borne by logic and human experience. If
the employer pays one employee less than the rest, it is not for that employee to explain why he receives less or why
the others receive more. That would be adding insult to injury. The employer has discriminated against that employee; it
is for the employer to explain why the employee is treated unfairly. Herein, the International School has failed to
discharge this burden. There is no evidence here that foreign-hires perform 25% more efficiently or effectively than the
local-hires. Both groups have similar functions and responsibilities, which they perform under similar working
conditions. The School cannot invoke the need to entice foreign-hires to leave their domicile to rationalize the
distinction in salary rates without violating the principle of equal work for equal pay. The point-of-hire classification
employed by respondent School to justify the distinction in the salary rates of foreign-hires and local hires to be an
invalid classification. There is no reasonable distinction between the services rendered by foreign-hires and localhires.
The practice of the School of according higher salaries to foreign-hires contravenes public policy.
Garcia vs. J. Drilon and Garcia, G. R. No. 179267, 25 June 2013
Nature of the Case: Petition for Review of Republic Act (R.A.) 9262

Facts:
Private respondent Rosalie filed a petition before the RTC of Bacolod City a Temporary Protection Order against her
husband, Jesus, pursuant to R.A. 9262, entitled “An Act Defining Violence Against Women and Their Children, Providing
for Protective Measures for Victims, Prescribing Penalties Therefor, and for Other Purposes.” She claimed to be a victim
of physical, emotional, psychological and economic violence, being threatened of deprivation of custody of her children
and of financial support and also a victim of marital infidelity on the part of petitioner.
The TPO was granted but the petitioner failed to faithfully comply with the conditions set forth by the said TPO, private-
respondent filed another application for the issuance of a TPO ex parte. The trial court issued a modified TPO and
extended the same when petitioner failed to comment on why the TPO should not be modified. After the given time
allowance to answer, the petitioner no longer submitted the required comment as it would be an “axercise in futility.”
Petitioner filed before the CA a petition for prohibition with prayer for injunction and TRO on, questioning the
constitutionality of the RA 9262 for violating the due process and equal protection clauses, and the validity of the
modified TPO for being “an unwanted product of an invalid law.”
The CA issued a TRO on the enforcement of the TPO but however, denied the petition for failure to raise the issue of
constitutionality in his pleadings before the trial court and the petition for prohibition to annul protection orders issued
by the trial court constituted collateral attack on said law.
Petitioner filed a motion for reconsideration but was denied. Thus, this petition is filed.

Issues:
1. WON the CA erred in dismissing the petition on the theory that the issue of constitutionality was not raised at the
earliest opportunity and that the petition constitutes a collateral attack on the validity of the law.
2. WON the CA committed serious error in failing to conclude that RA 9262 is discriminatory, unjust and violative of the
equal protection clause.
3. WON the CA committed grave mistake in not finding that RA 9262 runs counter to the due process clause of the
Constitution
4. WON the CA erred in not finding that the law does violence to the policy of the state to protect the family as a basic
social institution
5. WON the CA seriously erredin declaring RA 9262 as invalid and unconstitutional because it allows an undue
delegation of judicial power to Brgy. Officials.

Decision:
1. Petitioner contends that the RTC has limited authority and jurisdiction, inadequate to tackle the complex issue
of constitutionality. Family Courts have authority and jurisdiction to consider the constitutionality of a statute.
The question of constitutionality must be raised at the earliest possible time so that if not raised in the
pleadings, it may not be raised in the trial and if not raised in the trial court, it may not be considered in appeal.
2. RA 9262 does not violate the guaranty of equal protection of the laws. Equal protection simply requires that all
persons or things similarly situated should be treated alike, both as to rights conferred and responsibilities
imposed. In Victoriano v. Elizalde Rope Workerkers’ Union, the Court ruled that all that is required of a valid
classification is that it be reasonable, which means that the classification should be based on substantial
distinctions which make for real differences; that it must be germane to the purpose of the law; not limited to
existing conditions only; and apply equally to each member of the class. Therefore, RA9262 is based on a valid
classification and did not violate the equal protection clause by favouring women over men as victims of
violence and abuse to whom the Senate extends its protection.
3. RA 9262 is not violative of the due process clause of the Constitution. The essence of due process is in the
reasonable opportunity to be heard and submit any evidence one may have in support of one’s defense. The
grant of the TPO exparte cannot be impugned as violative of the right to due process.
4. The non-referral of a VAWC case to a mediator is justified. Petitioner’s contention that by not allowing
mediation, the law violated the policy of the State to protect and strengthen the family as a basic autonomous
social institution cannot be sustained. In a memorandum of the Court, it ruled that the court shall not refer the
case or any issue therof to a mediator. This is so because violence is not a subject for compromise.
5. There is no undue delegation of judicial power to Barangay officials. Judicial power includes the duty of the
courts of justice to settle actual controversies involving rights which are legally demandable and enforceable and
to determine whether or not there has been a grave abuse of discretion amounting to lack or excess of
jurisdiction on any part of any branch of the Government while executive power is the power to enforce and
administer the laws. The preliminary investigation conducted by the prosecutor is an executive, not a judicial,
function. The same holds true with the issuance of BPO. Assistance by Brgy. Officials and other law
enforcement agencies is consistent with their duty executive function.
The petition for review on certiorari is denied for lack of merit.
Ishmael Himagan vs People of the Philippines & Judge Hilario Mapayo
on November 6, 2010
“Equal Protection” – Suspension of PNP Members Charged with Grave Felonies

FACTS:
Himagan is a policeman assigned in Camp Catititgan, Davao City. He was charged for the murder of Benjamin Machitar Jr
and for the attempted murder of Benjamin’s younger brother, Barnabe. Pursuant to Sec 47 of RA 6975, Himagan was
placed into suspension pending the murder case. The law provides that “Upon the filing of a complaint or information
sufficient in form and substance against a member of the PNP for grave felonies where the penalty imposed by law is six
(6) years and one (1) day or more, the court shall immediately suspend the accused from office until the case is
terminated. Such case shall be subject to continuous trial and shall be terminated within ninety (90) days from
arraignment of the accused. Himagan assailed the suspension averring that Sec 42 of PD 807 of the Civil Service Decree,
that his suspension should be limited to ninety (90) days. He claims that an imposition of preventive suspension of over
90 days is contrary to the Civil Service Law and would be a violation of his constitutional right to equal protection of
laws.

ISSUE:
Whether or not Sec 47, RA 6975 violates equal protection guaranteed by the Constitution.

HELD:
The language of the first sentence of Sec 47 of RA 6975 is clear, plain and free from ambiguity. It gives no other meaning
than that the suspension from office of the member of the PNP charged with grave offense where the penalty is six
years and one day or more shall last until the termination of the case. The suspension cannot be lifted before the
termination of the case. The second sentence of the same Section providing that the trial must be terminated within
ninety (90) days from arraignment does not qualify or limit the first sentence. The two can stand independently of each
other. The first refers to the period of suspension. The second deals with the time from within which the trial should be
finished.
The reason why members of the PNP are treated differently from the other classes of persons charged criminally or
administratively insofar as the application of the rule on preventive suspension is concerned is that policemen carry
weapons and the badge of the law which can be used to harass or intimidate witnesses against them, as succinctly
brought out in the legislative discussions.
If a suspended policeman criminally charged with a serious offense is reinstated to his post while his case is pending, his
victim and the witnesses against him are obviously exposed to constant threat and thus easily cowed to silence by the
mere fact that the accused is in uniform and armed. the imposition of preventive suspension for over 90 days under Sec
47 of RA 6975 does not violate the suspended policeman’s constitutional right to equal protection of the laws.

Suppose the trial is not terminated within ninety days from arraignment, should the suspension of accused be lifted?
The answer is certainly no. While the law uses the mandatory word “shall” before the phrase “be terminated within
ninety (90) days”, there is nothing in RA 6975 that suggests that the preventive suspension of the accused will be lifted if
the trial is not terminated within that period. Nonetheless, the Judge who fails to decide the case within the period
without justifiable reason may be subject to administrative sanctions and, in appropriate cases where the facts so
warrant, to criminal or civil liability. If the trial is unreasonably delayed without fault of the accused such that he is
deprived of his right to a speedy trial, he is not without a remedy. He may ask for the dismissal of the case. Should the
court refuse to dismiss the case, the accused can compel its dismissal by certiorari, prohibition or mandamus, or secure
his liberty by habeas corpus.
QUINTO vs COMELEC
G.R. No. 189698, February 22, 2010

FACTS:
This is a motion for reconsideration filed by the COMELEC questioning an earlier decision of theSupreme Court declaring
as unconstitutional the second proviso in the third paragraph of Section 13 of Republic Act No. 9369, Section 66 of the
Omnibus Election Code and Section 4(a)of COMELEC Resolution No. 8678, mainly on the ground that they violate the
equal protectionclause of the Constitution and suffer from overbreadth.
The assailed Decision thus paved the way for public appointive officials to continue dischargingthe powers, prerogatives
and functions of their office notwithstanding their entry into thepolitical arena.
Section 4 (a) of COMELEC Resolution No. 8678 provides that, “
Any person holding a publicappointive office or position including active members of the Armed Forces of the
Philippines,and other officers and employees in government-owned or controlled corporations, shall beconsidered ipso
facto resigned from his office upon the filing of his certificate of candidacy.”
Petitioners of the above-entitled case are appointive officials who intend to be elected in thepreviously held 2010
elections and who felt aggrieved by the issuance of the questionedresolution.

ISSUE:
Whether or not 3rd Sec. 13 of RA 9369, Sec. 66 of the Omnibus Election Code, and Section 4 (a)of COMELEC Resolution
No. 8678 are unconstitutional.

HELD:
NO.
These provisions satisfy the requisites of the equal protection test, especially the secondrequirement that it must be
germane to the purposes of the law.
It was emphasized that the purpose of the law is to defer to the sovereign will of the people byletting elective officials
serve until the end of the terms for which they were electednotwithstanding the filing of their certificates of candidacy.
On the contrary, the automatic resignation rule was imposed upon appointive officials because
unlike elected politicians, “appointive officials, as officers and employees in the civil service, are
strictly prohibited from engaging in any partisan political activity or from taking part in any
election, except to vote” (Sec. 55 of the Administrative Code of 1987).
It is settled that the equal protection clause does not demand absolute equality; it merelyrequires that all persons shall
be treated alike, under like circumstances and conditions both asto privileges conferred and liabilities enforced. The test
used is reasonableness which requires that:
1. The classification rests on substantial distinctions;
2. It is germane to the purposes of the law;
3. It is not limited to existing conditions only; and
4. It applies equally to all members of the same class.
Substantial distinctions clearly exist between elective officials and appointive officials. Theformer occupy their office by
virtue of the mandate of the electorate. They are elected to anoffice for a definite term and may be removed therefrom
only upon stringent conditions. On theother hand, appointive officials hold their office by virtue of their designation
thereto by anappointing authority. Some appointive officials hold their office in a permanent capacity and areentitled to
security of tenure while others serve at the pleasure of the appointing authority
Louis “Barok” C. Biraogo, petitioner
Vs.
The Philippine Truth Commission of 2010, respondent
GR No. 129935

----------------------

GR No. 193036

Rep. Edcel C. Lagman, Rep. Rodolfo B. Albano, Jr., Rep. Simeon A. Datumanong, and Rep. Orlando B. Fua, Sr., petitioners
Vs.
Executive Secretary Paquito N. Ochoa, Jr., and Department of Budget and Management Secretary Florencio B. Abad,
repondents

Facts:
The genesis of the foregoing cases can be traced to the events prior to the historic May 2010 elections, when then
Senator Benigno Simeon Aquino III declared his staunch condemnation of graft and corruption with his slogan, "Kung
walang corrupt, walang mahirap." The Filipino people, convinced of his sincerity and of his ability to carry out this noble
objective, catapulted the good senator to the presidency.

Thus, at the dawn of his administration, the President on July 30, 2010, signed Executive Order No. 1 establishing the
Philippine Truth Commission of 2010 (Truth Commission).

The first case is G.R. No. 192935, a special civil action for prohibition instituted by petitioner Louis Biraogo (Biraogo) in
his capacity as a citizen and taxpayer. Biraogo assails Executive Order No. 1 for being violative of the legislative power of
Congress under Section 1, Article VI of the Constitution as it usurps the constitutional authority of the legislature to
create a public office and to appropriate funds therefor.

The second case, G.R. No. 193036, is a special civil action for certiorari and prohibition filed by petitioners Edcel C.
Lagman, Rodolfo B. Albano Jr., Simeon A. Datumanong, and Orlando B. Fua, Sr. (petitioners-legislators) as incumbent
members of the House of Representatives.

Issue/s:
1. Whether or not the petitioners have the legal standing to file their respective petitions and question Executive
Order No. 1;
2. Whether or not Executive Order No. 1 violates the principle of separation of powers by usurping the powers of
Congress to create and to appropriate funds for public offices, agencies and commissions;
3. Whether or not Executive Order No. 1 supplants the powers of the Ombudsman and the DOJ;
4. Whether or not Executive Order No. 1 violates the equal protection clause; and
5. Whether or not petitioners are entitled to injunctive relief.

Ruling:

Legal Standing of the Petitioners

The Court finds reason in Biraogo’s assertion that the petition covers matters of transcendental importance to justify the
exercise of jurisdiction by the Court. There are constitutional issues in the petition which deserve the attention of this
Court in view of their seriousness, novelty and weight as precedents. Where the issues are of transcendental and
paramount importance not only to the public but also to the Bench and the Bar, they should be resolved for the
guidance of all. The Court takes cognizance of the petition not due to overwhelming political undertones that clothe the
issue in the eyes of the public, but because the Court stands firm in its oath to perform its constitutional duty to settle
legal controversies with overreaching significance to society.

Power of the President to Create the Truth Commission

The Chief Executive’s power to create the Ad hoc Investigating Committee cannot be doubted. Having been
constitutionally granted full control of the Executive Department, to which respondents belong, the President has the
obligation to ensure that all executive officials and employees faithfully comply with the law. With AO 298 as mandate,
the legality of the investigation is sustained. Such validity is not affected by the fact that the investigating team and the
PCAGC had the same composition, or that the former used the offices and facilities of the latter in conducting the
inquiry.

Power of the Truth Commission to Investigate

The distinction between the power to investigate and the power to adjudicate was delineated by the Court in Cariño v.
Commission on Human Rights.59 Thus:

The legal meaning of "investigate" is essentially the same: "(t)o follow up step by step by patient inquiry or observation.
To trace or track; to search into; to examine and inquire into with care and accuracy; to find out by careful inquisition;
examination; the taking of evidence; a legal inquiry;" "to inquire; to make an investigation," "investigation" being in turn
described as "(a)n administrative function, the exercise of which ordinarily does not require a hearing. 2 Am J2d Adm L
Sec. 257; x x an inquiry, judicial or otherwise, for the discovery and collection of facts concerning a certain matter or
matters."

In the legal sense, "adjudicate" means: "To settle in the exercise of judicial authority. To determine finally. Synonymous
with adjudge in its strictest sense;" and "adjudge" means: "To pass on judicially, to decide, settle or decree, or to
sentence or condemn. x x. Implies a judicial determination of a fact, and the entry of a judgment."

Finally, nowhere in Executive Order No. 1 can it be inferred that the findings of the PTC are to be accorded
conclusiveness. Much like its predecessors, the Davide Commission, the Feliciano Commission and the Zenarosa
Commission, its findings would, at best, be recommendatory in nature. And being so, the Ombudsman and the DOJ have
a wider degree of latitude to decide whether or not to reject the recommendation. These offices, therefore, are not
deprived of their mandated duties but will instead be aided by the reports of the PTC for possible indictments for
violations of graft laws.

Violation of the Equal Protection Clause

The petitioners assail Executive Order No. 1 because it is violative of this constitutional safeguard. They contend that it
does not apply equally to all members of the same class such that the intent of singling out the "previous
administration" as its sole object makes the PTC an "adventure in partisan hostility." Thus, in order to be accorded with
validity, the commission must also cover reports of graft and corruption in virtually all administrations previous to that
of former President Arroyo.
The equal protection clause is aimed at all official state actions, not just those of the legislature. Its inhibitions cover all
the departments of the government including the political and executive departments, and extend to all actions of a
state denying equal protection of the laws, through whatever agency or whatever guise is taken.

Applying these precepts to this case, Executive Order No. 1 should be struck down as violative of the equal protection
clause. The clear mandate of the envisioned truth commission is to investigate and find out the truth "concerning the
reported cases of graft and corruption during the previous administration" only. The intent to single out the previous
administration is plain, patent and manifest. Mention of it has been made in at least three portions of the questioned
executive order.

The issue that seems to take center stage at present is - whether or not the Supreme Court, in the exercise of its
constitutionally mandated power of Judicial Review with respect to recent initiatives of the legislature and the executive
department, is exercising undue interference. Is the Highest Tribunal, which is expected to be the protector of the
Constitution, itself guilty of violating fundamental tenets like the doctrine of separation of powers? Time and again, this
issue has been addressed by the Court, but it seems that the present political situation calls for it to once again explain
the legal basis of its action lest it continually be accused of being a hindrance to the nation’s thrust to progress.

WHEREFORE, the petitions are GRANTED. Executive Order No. 1 is hereby declared UNCONSTITUTIONAL insofar as it is
violative of the equal protection clause of the Constitution.

As also prayed for, the respondents are hereby ordered to cease and desist from carrying out the provisions of Executive
Order No. 1.
CENTRAL BANK (NOW BANGKO SENTRAL NG PILIPINAS) EMPLOYEES ASSOCIATION, INC., PETITIONER, vs. BANGKO
SENTRAL NG PILIPINAS AND THE EXECUTIVE SECRETARY, RESPONDENTS.
G.R. NO. 148208, DECEMBER 15, 2004

FACTS:
On July 3, 1993, R.A. No. 7653 (the New Central Bank Act) took effect. It abolished the old Central Bank of the
Philippines, and created a new BSP.
On June 8, 2001, almost eight years after the effectivity of R.A. No. 7653, petitioner Central Bank (now BSP)
Employees Association, Inc., filed a petition for prohibition against BSP and the Executive Secretary of the Office of the
President, to restrain respondents from further implementing the last proviso in Section 15(c), Article II of R.A. No. 7653,
on the ground that it is unconstitutional.

Article II, Section 15(c) of R.A. No. 7653 provides:


Section 15, Exercise of Authority -In the exercise of its authority, the Monetary Board shall:
(c) Establish a human resource management system which shall govern the selection, hiring, appointment, transfer,
promotion, or dismissal of all personnel. Such system shall aim to establish professionalism and excellence at all levels of
the Bangko Sentral in accordance with sound principles of management.

A compensation structure, based on job evaluation studies and wage surveys and subject to the Board’s approval,
shall be instituted as an integral component of the Bangko Sentral’s human resource development program: Provided,
That the Monetary Board shall make its own system conform as closely as possible with the principles provided for
under Republic Act No. 6758 [Salary Standardization Act]. Provided, however, that compensation and wage structure of
employees whose positions fall under salary grade 19 and below shall be in accordance with the rates prescribed under
Republic Act No. 6758. The thrust of petitioner’s challenge is that the above proviso makes an unconstitutional cut
between two classes of employees in the BSP, viz: (1) the BSP officers or those exempted from the coverage of the
Salary Standardization Law (SSL) (exempt class); and (2) the rank-and-file (Salary Grade [SG] 19 and below), or those not
exempted from the coverage of the SSL (non-exempt class). It is contended that this classification is “a classic case of
class legislation,” allegedly not based on substantial distinctions which make real differences, but solely on the SG of the
BSP personnel’s position.

Petitioner also claims that it is not germane to the purposes of Section 15(c), Article II of R.A. No. 7653, the most
important of which is to establish professionalism and excellence at all levels in the BSP. Petitioner offers the following
sub-set of arguments:
a. the legislative history of R.A. No. 7653 shows that the questioned proviso does not appear in the original
and amended versions of House Bill No. 7037, nor in the original version of Senate Bill No. 1235;
b. subjecting the compensation of the BSP rank-and-file employees to the rate prescribed by the SSL actually
defeats the purpose of the law of establishing professionalism and excellence eat all levels in the BSP;
c. the assailed proviso was the product of amendments introduced during the deliberation of Senate Bill No.
1235, without showing its relevance to the objectives of the law, and even admitted by one senator as
discriminatory against low-salaried employees of the BSP;
d. GSIS, LBP, DBP and SSS personnel are all exempted from the coverage of the SSL; thus within the class of
rank-and-file personnel of government financial institutions (GFIs), the BSP rank-and-file are also
discriminated upon; and
e. the assailed proviso has caused the demoralization among the BSP rank-and-file and resulted in the gross
disparity between their compensation and that of the BSP officers’.

In sum, petitioner posits that the classification is not reasonable but arbitrary and capricious, and violates the equal
protection clause of the Constitution. Petitioner also stresses: (a) that R.A. No. 7653 has a separability clause, which will
allow the declaration of the unconstitutionality of the proviso in question without affecting the other provisions; and (b)
the urgency and propriety of the petition, as some 2,994 BSP rank-and-file employees have been prejudiced since 1994
when the proviso was implemented. Petitioner concludes that: (1) since the inequitable proviso has no force and effect
of law, respondents’ implementation of such amounts to lack of jurisdiction; and (2) it has no appeal nor any other plain,
speedy and adequate remedy in the ordinary course except through this petition for prohibition, which this Court should
take cognizance of, considering the transcendental importance of the legal issue involved.

Respondent BSP, in its comment, contends that the provision does not violate the equal protection clause and can
stand the constitutional test, provided it is construed in harmony with other provisions of the same law, such as “fiscal
and administrative autonomy of BSP,” and the mandate of the Monetary Board to “establish professionalism and
excellence at all levels in accordance with sound principles of management.”

The Solicitor General, on behalf of respondent Executive Secretary, also defends the validity of the provision. Quite
simplistically, he argues that the classification is based on actual and real differentiation, even as it adheres to the
enunciated policy of R.A. No. 7653 to establish professionalism and excellence within the BSP subject to prevailing laws
and policies of the national government.

ISSUE:
Thus, the sole - albeit significant - issue to be resolved in this case is whether the last paragraph of Section 15(c),
Article II of R.A. No. 7653, runs afoul of the constitutional mandate that "No person shall be . . . denied the equal
protection of the laws."

RULING:
A. UNDER THE PRESENT STANDARDS OF EQUAL PROTECTION, SECTION 15(c), ARTICLE II OF R.A. NO. 7653 IS VALID.

Jurisprudential standards for equal protection challenges indubitably show that the classification created by the
questioned proviso, on its face and in its operation, bears no constitutional infirmities.

It is settled in constitutional law that the "equal protection" clause does not prevent the Legislature from establishing
classes of individuals or objects upon which different rules shall operate - so long as the classification is not
unreasonable.

B. THE ENACTMENT, HOWEVER, OF SUBSEQUENT LAWS - EXEMPTING ALL OTHER RANK-AND-FILE EMPLOYEES OF GFIs
FROM THE SSL - RENDERS THE CONTINUED APPLICATION OF THE CHALLENGED PROVISION A VIOLATION OF THE EQUAL
PROTECTION CLAUSE.

While R.A. No. 7653 started as a valid measure well within the legislature’s power, we hold that the enactment of
subsequent laws exempting all rank-and-file employees of other GFIs leeched all validity out of the challenged proviso.

The constitutionality of a statute cannot, in every instance, be determined by a mere comparison of its provisions with
applicable provisions of the Constitution, since the statute may be constitutionally valid as applied to one set of facts
and invalid in its application to another.

A statute valid at one time may become void at another time because of altered circumstances. Thus, if a statute in its
practical operation becomes arbitrary or confiscatory, its validity, even though affirmed by a former adjudication, is
open to inquiry and investigation in the light of changed conditions.
The foregoing provisions impregnably institutionalize in this jurisdiction the long honored legal truism of "equal pay
for equal work." Persons who work with substantially equal qualifications, skill, effort and responsibility, under similar
conditions, should be paid similar salaries.

Congress retains its wide discretion in providing for a valid classification, and its policies should be accorded
recognition and respect by the courts of justice except when they run afoul of the Constitution. The deference stops
where the classification violates a fundamental right, or prejudices persons accorded special protection by the
Constitution. When these violations arise, this Court must discharge its primary role as the vanguard of constitutional
guaranties, and require a stricter and more exacting adherence to constitutional limitations. Rational basis should not
suffice.

Furthermore, concerns have been raised as to the propriety of a ruling voiding the challenged provision. It has been
proffered that the remedy of petitioner is not with this Court, but with Congress, which alone has the power to erase
any inequity perpetrated by R.A. No. 7653. Indeed, a bill proposing the exemption of the BSP rank-and-file from the SSL
has supposedly been filed.

Under most circumstances, the Court will exercise judicial restraint in deciding questions of constitutionality,
recognizing the broad discretion given to Congress in exercising its legislative power. Judicial scrutiny would be based on
the “rational basis” test, and the legislative discretion would be given deferential treatment.

But if the challenge to the statute is premised on the denial of a fundamental right or the perpetuation of prejudice
against persons favored by the Constitution with special protection, judicial scrutiny ought to be more strict. A weak and
watered down view would call for the abdication of this Court’s solemn duty to strike down any law repugnant to the
Constitution and the rights it enshrines. This is true whether the actor committing the unconstitutional act is a private
person or the government itself or one of its instrumentalities. Oppressive acts will be struck down regardless of the
character or nature of the actor.
Accordingly, when the grant of power is qualified, conditional or subject to limitations, the issue on whether or not the
prescribed qualifications or conditions have been met, or the limitations respected, is justifiable or non-political, the
crux of the problem being one of legality or validity of the contested act, not its wisdom. Otherwise, said qualifications,
conditions or limitations - particularly those prescribed or imposed by the Constitution - would be set at naught. What is
more, the judicial inquiry into such issue and the settlement thereof are the main functions of courts of justice under the
Presidential form of government adopted in our 1935 Constitution, and the system of checks and balances, one of its
basic predicates. As a consequence, we have neither the authority nor the discretion to decline passing upon said issue,
but are under the ineluctable obligation - made particularly more exacting and peremptory by our oath, as members of
the highest Court of the land, to support and defend the Constitution - to settle it.

In the case at bar, the challenged proviso operates on the basis of the salary grade or officer-employee status. It is
akin to a distinction based on economic class and status, with the higher grades as recipients of a benefit specifically
withheld from the lower grades. Officers of the BSP now receive higher compensation packages that are competitive
with the industry, while the poorer, low-salaried employees are limited to the rates prescribed by the SSL. The
implications are quite disturbing: BSP rank-and-file employees are paid the strictly regimented rates of the SSL while
employees higher in rank - possessing higher and better education and opportunities for career advancement - are given
higher compensation packages to entice them to stay. Considering that majority, if not all, the rank-and-file employees
consist of people whose status and rank in life are less and limited, especially in terms of job marketability, it is they -
and not the officers - who have the real economic and financial need for the adjustment This is in accord with the policy
of the Constitution "to free the people from poverty, provide adequate social services, extend to them a decent
standard of living, and improve the quality of life for all.” Any act of Congress that runs counter to this constitutional
desideratum deserves strict scrutiny by this Court before it can pass muster.
To be sure, the BSP rank-and-file employees merit greater concern from this Court. They represent the more impotent
rank-and-file government employees who, unlike employees in the private sector, have no specific right to organize as a
collective bargaining unit and negotiate for better terms and conditions of employment, nor the power to hold a strike
to protest unfair labor practices. These BSP rank-and-file employees represent the politically powerless and they should
not be compelled to seek a political solution to their unequal and iniquitous treatment. Indeed, they have waited for
many years for the legislature to act. They cannot be asked to wait some more for discrimination cannot be given any
waiting time. Unless the equal protection clause of the Constitution is a mere platitude, it is the Court’s duty to save
them from reasonless discrimination.

IN VIEW WHEREOF, we hold that the continued operation and implementation of the last proviso of Section 15(c),
Article II of Republic Act No. 7653 is unconstitutional.
ALMONTE et. al. VS VASQUEZ
G.R. No. 95367 May 23, 1995

Facts
The case is a petition for certiorari, prohibition, and mandamus to annul the subpoena duces tecum and orders issued by
respondent Ombudsman, requiring petitioners Neria Rogado and Elisa Rivera, as chief accountant and record custodian
of the Economic Intelligence and Investigation Bureau (EIIB) to produce “all documents relating to Personal Services
Funds for the year 1988 and all evidence, such as vouchers (salary) for the whole plantilla of EIIB for 1988” and to enjoin
him from enforcing his orders.
An anonymous and unsigned letter purportedly written by an employee of the EIIB, was sent to the Secretary of Finance,
with copies furnished to several government offices, including the Office of the Ombudsman.
In the letter were allegations as to the misuse of funds from the savings of unfulfilled plantilla positions, among other
forms of corruption and abuse of power.
As a response to the letter-complaint, petitioner Almonte denied allegations. Petitioner Perez also denied the issue for
the savings realized from the implementation of E.O. No. 127, since the DBM only allotted for the remaining 947
personnel, and that the disbursement of funds for the plantilla positions for overt and covert personnel had been
cleared by COA.
Jose F. Sano, the Graft Investigation Officer of the Ombudsman’s office found their responses unsatisfactory; therefore
he asked for authority to conduct an investigation. Anticipating the grant of his request, he issued a subpoena to
petitioners, compelling them to submit their counter-affidavits and the affidavits of their witnesses, as well as subpoena
duces tecum to the chief of the EIIB’s Accounting Division, ordering him to bring “all documents relating to Personal
Service Funds for the year 1988 and all evidence, such as vouchers (salary) for the whole plantilla of EIIB for 1988.”
Petitioners then moved to quash the subpoena (which was granted by the Ombudsman since no affidavit was filed
against petitioners) and the subpoena duces tecum, which was denied, since it was directed to the Chief Accountant,
petitioner Nerio Rogado. In addition the Ombudsman ordered the Chief of the Records a Section of the EIIB, petitioner
Elisa Rivera, to produce before the investigator "all documents relating to Personnel Service Funds, for the year 1988,
and all documents, salary vouchers for the whole plantilla of the EIIB for 1988, within ten (10) days from receipt hereof."
Petitioners filed for a motion of reconsideration, which was denied.

Issue(s)
1. Whether or not an unsigned and unverified letter complained is an “appropriate case” within the concept of the
Constitution
2. Whether or not the documents in question are classified, and therefore beyond the reach of public respondent’s
subpoena duces tecum.

Discussion
The petition is DISMISSED, but it is directed that the inspection of subpoenaed documents be made personally in camera
by the Ombudsman, and with all the safeguards outlined in the decision.
True, the court recognizes the privilege based on state secrets. However, in the case at bar, there have been no claims
that military or diplomatic secrets will be disclosed by the production of records pertaining to the personnel of the EIIB.
Nor is there a law or regulation which considers personnel records of the EIIB as classified information. On the contrary,
COA Circular No. 88-293 states that “The only item of expenditure which should be treated as strictly confidential
because it falls under the category of classified information is that relating to purchase of information and payment of
rewards.”
And even if the subpoenaed documents are treated as presumptively privileged, the decision would only justify ordering
the inspection in camera, and not their nonproduction.
Further, documents in question are public documents and as petitioner claims, the disbursements by the EIIB of funds
for personal service has already been cleared by COA, then there should be no reason why they should object to the
examination of the documents by the respondent Ombudsman.
As to the issue whether or not an unsigned and unverified letter is an “appropriate case”, it is expressly provided for in
the Constitution that “the Ombudsman and his Deputies, as protectors of the people, shall act promptly on complaints
filed in any form or manner against public officials or employees of the Government, or any subdivision, agency, or
instrumentality thereof, including government-owned or controlled corporations and shall in appropriate cases, notify
the complainants of the actions taken and the result thereof.

Ruling
WHEREFORE, the petition is DISMISSED, but it is directed that the inspection of subpoenaed documents be made
personally in camera by the Ombudsman, and with all the safeguards outlined in this decision.
ORMOC SUGAR COMPANY VS. TREASURER OF ORMOC CITY
22 SCRA 603; L-23794; 17 FEB 1968

Facts:
On January 29, 1964, the Municipal Board of Ormoc City passed Ordinance No. 4, Series of 1964, imposing "on any and
all productions of centrifugal sugar milled at the Ormoc Sugar Company, Inc., in Ormoc City a municipal tax equivalent to
one per centum (1%) per export sale to the United States of America and other foreign countries." Payments for said tax
were made, under protest, by Ormoc Sugar Company, Inc. on March 20, 1964 for P7, 087.50 and on April 20, 1964 for
P5, 000, or a total of P12, 087.50.

On June 1, 1964, Ormoc Sugar Company, Inc. filed before the Court of First Instance of Leyte, with service of a copy upon
the Solicitor General, a complaint against the City of Ormoc as well as its Treasurer, Municipal Board and Mayor, alleging
that the afore-stated ordinance is unconstitutional for being violative of the equal protection clause (Sec. 1[1], Art. III,
Constitution) and the rule of uniformity of taxation (Sec. 22[1]), Art. VI, Constitution).

Answering, the defendants asserted that the tax ordinance was within defendant city's power to enact under the Local
Autonomy Act and that the same did not violate the afore-cited constitutional limitations. After pre-trial and submission
of the case on memoranda, the Court of First Instance, on August 6, 1964, rendered a decision that upheld the
constitutionality of the ordinance and declared the taxing power of defendant chartered city broadened by the Local
Autonomy Act to include all other forms of taxes, licenses or fees not excluded in its charter.

Issue:
Whether or Not the ordinance is unconstitutional for being violative of the equal protection clause under Sec. 1[1], Art.
III, Constitution.
Whether or not it was violative of the rule of uniformity of taxation under the Bill of Rights, Sec. 22[1], Art. VI,
Constitution.

Held:
The Constitution in the bill of rights provides: ". . . nor shall any person be denied the equal protection of the laws." (Sec.
1 [1], Art. III) In Felwa vs. Salas, We ruled that the equal protection clause applies only to persons or things identically
situated and does not bar a reasonable classification of the subject of legislation, and a classification is reasonable where
(1) it is based on substantial distinctions which make real differences; (2) these are germane to the purpose of the law;
(3) the classification applies not only to present conditions but also to future conditions which are substantially identical
to those of the present; (4) the classification applies only to those who belong to the same class.

A perusal of the requisites instantly shows that the questioned ordinance does not meet them, for it taxes only
centrifugal sugar produced and exported by the Ormoc Sugar Company, Inc. and none other. At the time of the taxing
ordinance's enactment, Ormoc Sugar Company, Inc., it is true, was the only sugar central in the city of Ormoc. Still, the
classification, to be reasonable, should be in terms applicable to future conditions as well. The taxing ordinance should
not be singular and exclusive as to exclude any subsequently established sugar central, of the same class as plaintiff, for
the coverage of the tax. As it is now, even if later a similar company is set up, it cannot be subject to the tax because the
ordinance expressly points only to Ormoc City Sugar Company, Inc. as the entity to be levied upon.

Appellant, however, is not entitled to interest; on the refund because the taxes were not arbitrarily collected (Collector
of Internal Revenue v. Binalbagan). 6 At the time of collection, the ordinance provided a sufficient basis to preclude
arbitrariness, the same being then presumed constitutional until declared otherwise.
Wherefore, the decision appealed from is hereby reversed, the challenged ordinance is declared unconstitutional and
the defendants-appellees are hereby ordered to refund the P12,087.50 plaintiff-appellant paid under protest. No costs.
So ordered.
SERRANO v. GALLANT MARITIME SERVICES INC. & MARLOWE NAVIGATION CO., INC.
G.R. No. 167614. March 24, 2009

Facts:
Petitioner was hired by Gallant Maritime Services, Inc. and Marlow Navigation Co., Ltd. (respondents) under a POEA-
approved Contract of Employment. On March 19, 1998, the date of his departure, petitioner was constrained to accept a
downgraded employment contract for the position of Second Officer with a monthly salary of US$1,000.00, upon the
assurance and representation of respondents that he would be made Chief Officer by the end of April. However,
respondents did not deliver on their promise to make petitioner Chief Officer. Hence, petitioner refused to stay on as
Second Officer and was repatriated to the Philippines on May.

Petitioner's employment contract was for a period of 12 months or from March 19, 1998 up to March 19, 1999, but at
the time of his repatriation on May 26, 1998, he had served only two (2) months and seven (7) days of his contract,
leaving an unexpired portion of nine (9) months and twenty-three (23) days.

Petitioner filed with the Labor Arbiter (LA) a Complaint against respondents for constructive dismissal and for payment
of his money claims. LA rendered the dismissal of petitioner illegal and awarding him monetary benefits. Respondents
appealed to the NLRC to question the finding of the LA. Likewise, petitioner also appealed to the NLRC on the sole issue
that the LA erred in not applying the ruling of the Court in Triple Integrated Services, Inc. v. National Labor Relations
Commission that in case of illegal dismissal, OFWs are entitled to their salaries for the unexpired portion of their
contracts.
Petitioner also appealed to the NLRC on the sole issue that the LA erred in not applying the ruling of the Court in
Triple Integrated Services, Inc. v. National Labor Relations Commission that in case of illegal dismissal, OFWs are entitled
to their salaries for the unexpired portion of their contracts. Petitioner filed a Motion for Partial Reconsideration; he
questioned the constitutionality of the subject clause. Petitioner filed a Petition for Certiorari with the CA, reiterating the
constitutional challenge against the subject clause. CA affirmed the NLRC ruling on the reduction of the applicable salary
rate; however, the CA skirted the constitutional issue raised by petitioner.

The last clause in the 5th paragraph of Section 10, Republic Act (R.A.) No. 8042, to wit:
Sec. 10. Money Claims. - x x x In case of termination of overseas employment without just, valid or authorized cause as
defined by law or contract, the workers shall be entitled to the full reimbursement of his placement fee with interest of
twelve percent (12%) per annum, plus his salaries for the unexpired portion of his employment contract or for three (3)
months for every year of the unexpired term, whichever is less.

Applying the subject clause, the NLRC and the CA computed the lump-sum salary of petitioner at the monthly rate of
US$1,400.00 covering the period of three months out of the unexpired portion of nine months and 23 days of his
employment contract or a total of US$4,200.00.

Impugning the constitutionality of the subject clause, petitioner contends that, in addition to the US$4,200.00 awarded
by the NLRC and the CA, he is entitled to US$21,182.23 more or a total of US$25,382.23, equivalent to his salaries for
the entire nine months and 23 days left of his employment contract, computed at the monthly rate of US$2,590.00

Issue:
1.) Is petitioner entitled to his monetary claim which is the lump-sum salary for the entire unexpired portion of his 12-
month employment contract, and not just for a period of three months?
2.) Should petitioner’s overtime and leave pay form part of the salary basis in the computation of his monetary award,
because these are fixed benefits that have been stipulated into his contract?
Held:
1.) Yes. Petitioner is awarded his salaries for the entire unexpired portion of his employment contract consisting of nine
months and 23 days computed at the rate of US$1,400.00 per month. The subject clause “or for three months for every
year of the unexpired term, whichever is less” in the 5th paragraph of Section 10 of Republic Act No. 8042 is declared
unconstitutional.

In sum, prior to R.A. No. 8042, OFWs and local workers with fixed-term employment who were illegally discharged were
treated alike in terms of the computation of their money claims: they were uniformly entitled to their salaries for the
entire unexpired portions of their contracts. But with the enactment of R.A. No. 8042, specifically the adoption of the
subject clause, illegally dismissed OFWs with an unexpired portion of one year or more in their employment contract
have since been differently treated in that their money claims are subject to a 3-month cap, whereas no such limitation
is imposed on local workers with fixed-term employment.

The Court concludes that the subject clause contains a suspect classification in that, in the computation of the monetary
benefits of fixed-term employees who are illegally discharged, it imposes a 3-month cap on the claim of OFWs with an
unexpired portion of one year or more in their contracts, but none on the claims of other OFWs or local workers with
fixed-term employment. The subject clause singles out one classification of OFWs and burdens it with a peculiar
disadvantage.

The Court further holds that the subject clause violates petitioner's right to substantive due process, for it deprives him
of property, consisting of monetary benefits, without any existing valid governmental purpose. The subject clause being
unconstitutional, petitioner is entitled to his salaries for the entire unexpired period of nine months and 23 days of his
employment contract, pursuant to law and jurisprudence prior to the enactment of R.A. No. 8042.

2.) No. The word salaries in Section 10(5) does not include overtime and leave pay. For seafarers like petitioner, DOLE
Department Order No. 33, series 1996, provides a Standard Employment Contract of Seafarers, in which salary is
understood as the basic wage, exclusive of overtime, leave pay and other bonuses; whereas overtime pay is
compensation for all work “performed” in excess of the regular eight hours, and holiday pay is compensation for any
work “performed” on designated rest days and holidays.

By the foregoing definition alone, there is no basis for the automatic inclusion of overtime and holiday pay in the
computation of petitioner's monetary award; unless there is evidence that he performed work during those periods.
LITO CORPUZ vs. PEOPLE OF THE PHILIPPINES
G.R. No. 180016, April 29, 2014 PERALTA, J.:

FACTS:
1. Accused Corpuz received from complainant Tangcoy pieces of jewelry with an obligation to sell the same and remit
the proceeds of the sale or to return the same if not sold, after the expiration of 30 days.
2. The period expired without Corpuz remitting anything to Tangcoy.
3. When Corpuz and Tangcoy met, Corpuz promised that he will pay, but to no avail.
4. Tangcoy filed a case for estafa with abuse of confidence against Corpuz.
5. Corpuz argued as follows:
a. The proof submitted by Tangcoy (receipt) is inadmissible for being a mere photocopy.
b. The information was defective because the date when the jewelry should be returned and the date when crime
occurred is different from the one testified to by Tangcoy.
c. Fourth element of estafa or demand is not proved.
d. Sole testimony of Tangcoy is not sufficient for conviction

ISSUES and RULING


Can the court admit as evidence a photocopy of document without violating the best evidence rule (only original
documents, as a general rule, is admissible as evidence)?
Yes. The established doctrine is that when a party failed to interpose a timely objection to evidence at the time they
were offered in evidence, such objection shall be considered as waived.
Here, Corpuz never objected to the admissibility of the said evidence at the time it was identified, marked and testified
upon in court by Tangcoy. Corpuz also failed to raise an objection in his Comment to the prosecution’s formal offer of
evidence and even admitted having signed the said receipt.

Is the date of occurrence of time material in estafa cases with abuse of confidence?
No. It is true that the gravamen of the crime of estafa with abuse of confidence under Article 315, paragraph 1,
subparagraph (b) of the RPC is the appropriation or conversion of money or property received to the prejudice of the
owner and that the time of occurrence is not a material ingredient of the crime. Hence, the exclusion of the period and
the wrong date of the occurrence of the crime, as reflected in the Information, do not make the latter fatally defective.

Further, the following satisfies the sufficiency of information:


1. The designation of the offense by the statute;
2. The acts or omissions complained of as constituting the offense;
3. The name of the offended party; and
4. The approximate time of the commission of the offense, and the place wherein the offense was committed.

The 4th element is satisfied. Even though the information indicates that the time of offense was committed “on or about
the 5th of July 1991,” such is not fatal to the prosecution’s cause considering that Section 11 of the same Rule requires a
statement of the precise time only when the same is a material ingredient of the offense.

What is the form of demand required in estafa with abuse of confidence?


Note first that the elements of estafa with abuse of confidence are as follows:
(a) that money, goods or other personal property is received by the offender in trust, or on commission,
or for administration, or under any other obligation involving the duty to make delivery of, or to
return the same;
(b) that there be misappropriation or conversion of such money or property by the offender or denial
on his part of such receipt;
(c) that such misappropriation or conversion or denial is to the prejudice of another; and
(d) that there is a demand made by the offended party on the offender.

No specific type of proof is required to show that there was demand. Demand need not even be formal; it may be
verbal. The specific word “demand” need not even be used to show that it has indeed been made upon the person
charged, since even a mere query as to the whereabouts of the money [in this case, property], would be tantamount to
a demand.

In Tubb v. People, where the complainant merely verbally inquired about the money entrusted to the accused, the
query was tantamount to a demand.

May a sole witness be considered credible?


Yes. Note first that settled is the rule that in assessing the credibility of witnesses, SC gives great respect to the
evaluation of the trial court for it had the unique opportunity to observe the demeanor of witnesses and their
deportment on the witness stand, an opportunity denied the appellate courts, which merely rely on the records of the
case.
The assessment by the trial court is even conclusive and binding if not tainted with arbitrariness or oversight of some
fact or circumstance of weight and influence, especially when such finding is affirmed by the CA. Truth is established not
by the number of witnesses, but by the quality of their testimonies, for in determining the value and credibility of
evidence, the witnesses are to be weighed not numbered.
SECTION 2 SEARCHES AND SEIZURES

People vs. Andre Marti


GR 81561, 18 January 1991

Facts:
On 14 August 1987, Andre Marti and his common-law wife, Shirley Reyes, went to the booth of the
Manila Packing and Export Forwarders in the Pistang Pilipino Complex, Ermita, Manila, carrying with them 4 gift-
wrapped packages. Anita Reyes (the proprietress and no relation to Shirley Reyes) attended to them. Marti informed
Anita Reyes that he was sending the packages to a friend in Zurich, Switzerland. Marti filled up the contract necessary
for the transaction, writing therein his name, passport number, the date of shipment and the name and address of the
consignee, namely, "WALTER FIERZ, Mattacketr II, 8052 Zurich, Switzerland." Anita Reyes did not inspect the packages as
Marti refused, who assured the former that the packages simply contained books, cigars, and gloves and were gifts to
his friend in Zurich. In view of Marti's representation, the 4 packages were then placed inside a brown corrugated box,
with styro-foam placed at the bottom and on top of the packages, and sealed with masking tape. Before delivery of
Marti's box to the Bureau of Customs and/or Bureau of Posts, Mr. Job Reyes (proprietor) and husband of Anita (Reyes),
following standard operating procedure, opened the boxes for final inspection, where a peculiar odor emitted
therefrom. Job pulled out a cellophane wrapper protruding from the opening of one of the gloves, and took several
grams of the contents thereof. Job Reyes forthwith prepared a letter reporting the shipment to the NBI and requesting a
laboratory examination of the samples he extracted from the cellophane wrapper. At the Narcotics Section of the
National Bureau of Investigation (NBI), the box containing Marti's packages was opened, yielding dried marijuana leaves,
or cake-like (bricks) dried marijuana leaves. The NBI agents made an inventory and took charge of the box and of the
contents thereof, after signing a "Receipt" acknowledging custody of the said effects. Thereupon, the NBI agents tried to
locate Marti but to no avail, inasmuch as the latter's stated address was the Manila Central Post Office. Thereafter, an
Information was filed against Marti for violation of RA 6425, otherwise known as the Dangerous Drugs Act. After trial,
the Special Criminal Court of Manila (Regional Trial Court, Branch XLIX) rendered the decision, convicting Marti of
violation of Section 21 (b), Article IV in relation to Section 4, Article 11 and Section 2 (e)(i), Article 1 of Republic Act
6425, as amended, otherwise known as the Dangerous Drugs Act. Marti appealed.

Issue:
Whether an act of a private individual, allegedly in violation of the accused's constitutional rights, be invoked against the
State.

Held:
In the absence of governmental interference, the liberties guaranteed by the Constitution cannot be invoked against the
State. The contraband herein, having come into possession of the Government without the latter transgressing the
accused's rights against unreasonable search and seizure, the Court sees no cogent reason why the same should not be
admitted against him in the prosecution of the offense charged. The mere presence of the NBI agents did not convert
the reasonable search effected by Reyes into a warrantless search and seizure proscribed by the Constitution. Merely to
observe and look at that which is in plain sight is not a search. Having observed that which is open, where no trespass
has been committed in aid thereof, is not search. Where the contraband articles are identified without a trespass on the
part of the arresting officer, there is not the search that is prohibited by the constitution. The constitutional proscription
against unlawful searches and seizures therefore applies as a restraint directed only against the government and its
agencies tasked with the enforcement of the law. Thus, it could only be invoked against the State to whom the restraint
against arbitrary and unreasonable exercise of power is imposed. If the search is made upon the request of law
enforcers, a warrant must generally be first secured if it is to pass the test of constitutionality. However, if the search is
made at the behest or initiative of the proprietor of a private establishment for its own and private purposes, as in the
case at bar, and without the intervention of police authorities, the right against unreasonable search and seizure cannot
be invoked for only the act of private individual, not the law enforcers, is involved. In sum, the protection against
unreasonable searches and seizures cannot be extended to acts committed by private individuals so as to bring it within
the ambit of alleged unlawful intrusion by the government.
Stonehill vs. Diokno
GR L-19550, 19 June 1967

Facts:
Upon application of the officers of the government, Special Prosecutors Pedro D. Cenzon, Efren I. Plana and Manuel
Villareal Jr. and Assistant Fiscal Manases G. Reyes; Judge Amado Roan (Municipal Court of Manila), Judge Roman
Cansino (Municipal Court of Manila), Judge Hermogenes Caluag (Court of First
Instance of Rizal-Quezon City Branch), and Judge Damian Jimenez (Municipal Court of Quezon City) issued, on different
dates, a total of 42 search warrants against Harry S. Stonehill, Robert P. Brooks, HJohn J. Brooks, and Karl Beck, and/or
the corporations of which they were officers, directed to any peace officer, to search the said persons and/or the
premises of their offices, warehouses and/or residences, and to seize and take possession of the following personal
property to wit: "Books of accounts, financial records, vouchers, correspondence, receipts, ledgers, journals, portfolios,
credit journals, typewriters, and other documents and/or papers showing all business transactions including
disbursements receipts, balance sheets and profit and loss statements and Bobbins (cigarette wrappers)" as "the subject
of the offense; stolen or embezzled andproceeds or fruits of the offense," or "used or intended to be used as the means
of committing the offense," which is described in the applications adverted to above as "violation of Central Bank Laws,
Tariff and Customs Laws, Internal Revenue (Code) and the Revised Penal Code." Alleging that the search warrants are
null and void, as contravening the Constitution and the Rules of Court, Stonehill, et. al. filed with the Supreme Court the
original action for certiorari, prohibition, mandamus and injunction. On 22 March 1962, the Supreme Court issued the
writ of preliminary injunction prayed for in the petition. However, by resolution dated 29 June 1962, the writ was
partially lifted or dissolved, insofar as the papers, documents and things seized from the offices of the corporations are
concerned; but, the injunction was maintained as regards the papers, documents and things found and seized in the
residences of Stonehill, et. al.

Issue:
Whether Stonehill, et. al. can assail the legality of the contested warrants that allowed seizure of documents, papers and
other effects in the corporate offices, and other places besides their residences.

Held:
Stonehill, et. al. maintained that the search warrants are in the nature of general warrants and that, accordingly, the
seizures effected upon the authority thereof are null and void. No warrant shall issue but upon probable cause, to be
determined by the judge in the manner set forth in said provision; and the warrant shall particularly describe the things
to be seized. None of these requirements has been complied with in the contested warrants. The grave violation of the
Constitution made in the application for the contested search warrants was compounded by the description therein
made of the effects to be searched for and seized. The warrants authorized the search for and seizure of records
pertaining to all business transactions of Stonehill, et. al., regardless of whether the transactions were legal or illegal.
The warrants sanctioned the seizure of all records of the corporate officers and the corporations, whatever their nature,
thus openly contravening the explicit command of our Bill of Rights — that the things to be seized be particularly
described — as well as tending to defeat its major objective: the elimination of general warrants. However, the
documents, papers, and things seized under the alleged authority of the warrants in question may be split into (2) major
groups, namely: (a) those found and seized in the offices of the corporations and (b) those found seized in the
residences of Stonehill, et. al. As regards the first group, Stonehill, et. al. have no cause of action to assail the legality of
the contested warrants and of the seizures made in pursuance thereof, for the simple reason that said corporations
have their respective personalities, separate and distinct from the personality of Stonehill, et.al., regardless of the
amount of shares of stock or of the interest of each of them in said corporations, and
whatever the offices they hold therein may be. Indeed, it is well settled that the legality of a seizure can be contested
only by the party whose rights have been impaired thereby, and that the objection to an unlawful search and seizure is
purely personal and cannot be availed of by third parties. Consequently, Stonehill, et. al. may not validly object to the
use in evidence against them of the documents, papers and things seized from the offices and premises of the
corporations adverted to above, since the right to object to the admission of said papers in evidence belongs exclusively
to the corporations, to whom the seized effects belong, and may not be invoked by the corporate officers in proceedings
against them in their individual capacity. With respect to the documents, papers and things seized in the residences of
Stonehill, et. al., the 29 June 1962 Resolution of the Supreme Court, denying the lifting of the writ of preliminary
injunction previously issued by the Court on the documents, papers and things seized in the residences, in effect,
restrained the prosecutors from using them in evidence against Stonehill, et. al. Thus, the Court held that the warrants
for the search of 3 residences are null and void; that the searches and seizures therein made are illegal; that the writ of
preliminary injunction heretofore issued, in connection with the documents, papers and other effects thus seized in said
residences is made permanent, that the writs prayed for are granted, insofar as the documents, papers and other effects
so seized in the residences are concerned; and that the petition herein is dismissed and the writs prayed for denied, as
regards the documents, papers and other effects seized in the 29 places, offices and other
premises.
SOLIVEN VS. MAKASIAR 167 SCRA 393;
G.R. NO. 82585; 14 NOV 1988

Facts:
In these consolidated cases, three principal issues were raised: (1) whether or not petitioners were denied due process
when information for libel were filed against them although the finding of the existence of a prima facie case was still
under review by the Secretary of Justice and, subsequently, by the President; and (2) whether or not the constitutional
rights of Beltran were violated when respondent RTC judge issued a warrant for his arrest without personally examining
the complainant and the witnesses, if any, to determine probable cause. Subsequent events have rendered the first
issue moot and academic. On March 30, 1988, the Secretary of Justice denied petitioners' motion for reconsideration
and upheld the resolution of the Undersecretary of Justice sustaining the City Fiscal's finding of a prima facie case
against petitioners. A second motion for reconsideration filed by petitioner Beltran was denied by the Secretary of
Justice on April 7, 1988. On appeal, the President, through the Executive Secretary, affirmed the resolution of the
Secretary of Justice on May 2, 1988. The motion for reconsideration was denied by the Executive Secretary on May 16,
1988. With these developments, petitioners' contention that they have been denied the administrative remedies
available under the law has lost factual support.

Issues:
(1) Whether or Not petitioners were denied due process when information for libel were filed against them although the
finding of the existence of a prima facie case was still under review by the Secretary of Justice and, subsequently, by the
President.
(2) Whether or Not the constitutional rights of Beltran were violated when respondent RTC judge issued a warrant for
his arrest without personally examining the complainant and the witnesses, if any, to determine probable cause

Held:
With respect to petitioner Beltran, the allegation of denial of due process of law in the preliminary investigation is
negated by the fact that instead of submitting his counter- affidavits, he filed a "Motion to Declare Proceedings Closed,"
in effect waiving his right to refute the complaint by filing counter-affidavits. Due process of law does not require that
the respondent in a criminal case actually file his counter-affidavits before the preliminary investigation is deemed
completed. All that is required is that the respondent be given the opportunity to submit counter-affidavits if he is so
minded.

The second issue, raised by petitioner Beltran, calls for an interpretation of the constitutional provision on the issuance
of warrants of arrest. The pertinent provision reads:

Art. III, Sec. 2. The right of the people to be secure in their persons, houses, papers and effects against unreasonable
searches and seizures of whatever nature and for any purpose shall be inviolable, and no search warrant or warrant of
arrest shall issue except upon probable cause to be determined personally by the judge after examination under oath or
affirmation of the complainant and the witnesses he may produce, and particularly describing the place to be searched
and the persons or things to be seized.

The addition of the word "personally" after the word "determined" and the deletion of the grant of authority by the
1973 Constitution to issue warrants to "other responsible officers as may be authorized by law," has apparently
convinced petitioner Beltran that the Constitution now requires the judge to personally examine the complainant and
his witnesses in his determination of probable cause for the issuance of warrants of arrest. This is not an accurate
interpretation.
What the Constitution underscores is the exclusive and personal responsibility of the issuing judge to satisfy himself of
the existence of probable cause. In satisfying himself of the existence of probable cause for the issuance of a warrant of
arrest, the judge is not required to personally examine the complainant and his witnesses. Following established
doctrine and procedure, he shall: (1) personally evaluate the report and the supporting documents submitted by the
fiscal regarding the existence of probable cause and, on the basis thereof, issue a warrant of arrest; or (2) if on the basis
thereof he finds no probable cause, he may disregard the fiscal's report and require the submission of supporting
affidavits of witnesses to aid him in arriving at a conclusion as to the existence of probable cause.

Sound policy dictates this procedure, otherwise judges would be unduly laden with the preliminary examination and
investigation of criminal complaints instead of concentrating on hearing and deciding cases filed before their courts. It
has not been shown that respondent judge has deviated from the prescribed procedure. Thus, with regard to the
issuance of the warrants of arrest, a finding of grave abuse of discretion amounting to lack or excess of jurisdiction
cannot be sustained. The petitions fail to establish that public respondents, through their separate acts, gravely abused
their discretion as to amount to lack of jurisdiction. Hence, the writs of certiorari and prohibition prayed for cannot
issue.

WHEREFORE, finding no grave abuse of discretion amounting to excess or lack of jurisdiction on the part of the public
respondents, the Court Resolved to DISMISS the petitions in G. R. Nos. 82585, 82827 and 83979. The Order to maintain
the status quo contained in the Resolution of the Court en banc dated April 7, 1988 and reiterated in the Resolution
dated April 26, 1988 is LIFTED.
SILVA VS. PRESIDING JUDGE
203 SCRA 140; G.R. No. 81756; 21 Oct 1991

Facts:
Sgt. Villamor, chief of the PC Narcom Detachment in Dumaguete City filed an "application for search warrant" and
"Deposition of witness" against petitioner Nicomedes Silva and Martin Silva. Judge Nickarter Ontal, then the presiding
judge of RTC of Dumaguete issued Search Warrant No.1 pursuant to the said applications for violation of RA 6425
Dangerous Drugs ACT of 1972. Such warrant states that there is a probable cause to believe that Mr. Tama Silva has the
possession and control of marijuana dried leaves, cigarette and joint. The warrant authorizes Sgt. Villamor to make an
immediate search at any time of the room of Mr. Tama Silva at the residence of his father Comedes Silva and to open
aparadors, lockers, cabinets, cartons and containers to look for said illegal drugs. In the course of the search, the officers
seized money belonging to Antonieta Silva in the amount of P1,231.40. Petitioner filed a motion to quash Search
Warrant No.1 on the ground that 1) it was issued on the sole basis of mimeographed 2) the judge failed to personally
examine the complainant and witness by searching questions and answers.

Issue:
Whether or Not Search Warrant No.1 is invalid. WON the officers abused their authority in seizing the money of
Antonieta Silva.

Held:
Search Warrant No. 1 is invalid due to the failure of the judge to examine the witness in the form of searching questions
and answers. The questions asked were leading as they are answerable by mere yes or no. Such questions are not
sufficiently searching to establish probable cause. The questions were already mimeographed and all the witness had to
do was fill in their answers on the blanks provided. Judge Ontal is guilty of grave abuse of discretion when he rejected
the motion of Antonieta Silva seeking the return of her money.

The officers who implemented the search warrant clearly abused their authority when they seized the money of
Antonieta Silva. The warrant did not indicate the seizure of money but only for marijuana leaves, cigarettes..etc. Search
Warrant No. 1 is declared null and void.

*** Sec 4 Rule 126 Rules of Court


Examination of the complainant, record -the judge before issuing the warrant, personally examine in the form of
searching questions and answers, in writing and under oath the complainant and any witness he may produce the facts
personally known to them and attach to the record their sworn statements together with their affidavits.
MORANO VS. VIVO
20 SCRA 562; G.R. L-22196; 30 JUN 1967

Facts:
Chan Sau Wah, a Chinese citizen born in Fukien, China arrived in the Philippines on November 1961 to visit her cousin,
Samuel Lee Malaps. She left China and her children by a first marriage: Fu Tse Haw and Fu Yan Kai both minors, in the
care of neighbors in Fukien, China. Chan Sau wah arrived in the Philippines with Fu Yan Fun, her minor son also by the
first marriage. Chan Sau Wah and her minor son Fu Yan Fun were permitted only into the Philippines under a temporary
visitor's visa for two months and after they posted a cash bond of 4,000 pesos. On January 1962, Chan Sau Wah married
Esteban Morano, a native-born Filipino citizen. Born to this union on September 1962 was Esteban Morano, Jr. To
prolong their stay in the Philippines, Chan Sau Wah and Fu Yan Fun obtained several extensions. The last extension
expired on September 10, 1962. In a letter dated August 31, 1962, the Commissioner of Immigration ordered Chan Sau
Wah and her son, Fu Yan Fun, to leave the country on or before September 10, 1962 with a warning that upon failure so
to do, he will issue a warrant for their arrest and will cause the confiscation of their bond.

Issue:
Whether or Not the issuance of the warrant of arrest is unconstitutional.

Held:
Chan Sau Wah entered the Philippines on a tourist-temporary visitor's visa. She is a non-immigrant. Under Section 13
just quoted, she may therefore be admitted if she were a qualified and desirable alien and subject to the provisions of
the last paragraph of Section 9. Therefore, first, she must depart voluntarily to some foreign country; second, she must
procure from the appropriate consul the proper visa; and third, she must thereafter undergo examination by the officials
of the Bureau of Immigration at the port of entry for determination of her admissibility in accordance with the
requirements of the immigration Act. This Court in a number of cases has ruled, and consistently too, that an alien
admitted as a temporary visitor cannot change his or her status without first departing from the country and complying
with the requirements of Section 9 of the Immigration Act. The gravamen of petitioners' argument is that Chan Sau Wah
has, since her entry, married in Manila a native-born Filipino, Esteban Morano. It will not particularly help analysis for
petitioners to appeal to family solidarity in an effort to thwart her deportation. Chan Sau Wah, seemingly is not one who
has a high regard for such solidarity. Proof: She left two of her children by the first marriage, both minors, in the care of
neighbors in Fukien, China.Then, the wording of the statute heretofore adverted to is a forbidding obstacle which will
prevent this Court from writing into the law an additional provision that marriage of a temporary alien visitor to a
Filipino would ipso facto make her a permanent resident in his country. This is a field closed to judicial action. No
breadth of discretion is allowed. We cannot insulate her from the State's power of deportation. it would be an easy
matter for an alien woman to enter the Philippines as a temporary visitor, go through a mock marriage, but actually live
with another man as husband and wife, and thereby skirt the provisions of our immigration law. Also, a woman of
undesirable character may enter this country, ply a pernicious trade, marry a Filipino, and again throw overboard
Sections 9 and 13 of the Act. Such a flanking movement, we are confident, is impermissible.Recently we confirmed the
rule that an alien wife of a Filipino may not stay permanently without first departing from the Philippines. Reason:
Discourage entry under false pretenses.
HARVEY V. DEFENSOR-SANTIAGO
162 SCRA 840; G.R. NO. 82544; 28 JUN 1988

Facts:
This is a petition for Habeas Corpus. Petitioners are the following: American nationals Andrew Harvey, 52 and Jonh
Sherman 72. Dutch Citizen Adriaan Van Den Elshout, 58. All reside at Pagsanjan Laguna respondent Commissioner
Miriam Defensor Santiago issued Mission Orders to the Commission of Immigration and Deportation (CID) to
apprehended petitioners at their residences. The “Operation Report” read that Andrew Harvey was found together
with two young boys. Richard Sherman was found with two naked boys inside his room. While Van Den Elshout in the
“after Mission Report” read that two children of ages 14 and 16 has been under his care and subjects confirmed being
live-in for sometime now.

Seized during the petitioner’s apprehension were rolls of photo negatives and photos of suspected child prostitutes
shown in scandalous poses as well as boys and girls engaged in sex. Posters and other literature advertising the child
prostitutes were also found.

Petitioners were among the 22 suspected alien pedophiles. They were apprehended 17 February1988 after close
surveillance for 3 month of the CID in Pagsanjan, Laguna. 17 of the arrested aliens opted for self-deportation. One
released for lack of evidence, another charged not for pedophile but working with NO VISA, the 3 petitioners chose to
face deportation proceedings. On 4 March1988, deportation proceedings were instituted against aliens for being
undesirable aliens under Sec.69 of Revised Administrative Code.

Warrants of Arrest were issued 7March1988 against petitioners for violation of Sec37, 45 and 46 of Immigration Act and
sec69 of Revised Administrative Code. Trial by the Board of Special Inquiry III commenced the same date. Petition for
bail was filed 11March 1988 but was not granted by the Commissioner of Immigration. 4 April1988 Petitioners filed a
petition for Writ of Habeas Corpus. The court heard the case on oral argument on 20 April 1988.

Issue:
Whether or Not the Commissioner has the power to arrest and detain petitioners pending determination of existence of
probable cause.

Whether or Not there was unreasonable searches and seizures by CID agents.

Whether or Not the writ of Habeas Corpus may be granted to petitioners.

Held:
While pedophilia is not a crime under the Revised Penal Code, it violates the declared policy of the state to promote and
protect the physical, moral, spiritual and social well being of the youth. The arrest of petitioners was based on the
probable cause determined after close surveillance of 3 months. The existence of probable cause justified the arrest
and seizure of articles linked to the offense. The articles were seized as an incident to a lawful arrest; therefore the
articles are admissible evidences (Rule 126, Section12 of Rules on Criminal Procedure).

The rule that search and seizures must be supported by a valid warrant of arrest is not an absolute rule. There are at
least three exceptions to this rule. 1.) Search is incidental to the arrest. 2.) Search in a moving vehicle. 3.) Seizure of
evidence in plain view. In view of the foregoing, the search done was incidental to the arrest.

The filing of the petitioners for bail is considered as a waiver of any irregularity attending their arrest and estops them
from questioning its validity. Furthermore, the deportation charges and the hearing presently conducted by the Board
of Special Inquiry made their detention legal. It is a fundamental rule that habeas corpus will not be granted when
confinement is or has become legal, although such confinement was illegal at the beginning.

The deportation charges instituted by the Commissioner of Immigration are in accordance with Sec37 (a) of the
Philippine Immigration Act of 1940 in relation to sec69 of the Revised Administrative code. Section 37 (a) provides that
aliens shall be arrested and deported upon warrant of the Commissioner of Immigration and Deportation after a
determination by the Board of Commissioners of the existence of a ground for deportation against them. Deportation
proceedings are administrative in character and never construed as a punishment but a preventive measure. Therefore,
it need not be conducted strictly in accordance with ordinary Court proceedings. What is essential is that there should
be a specific charge against the alien intended to be arrested and deported. A fair hearing must also be conducted with
assistance of a counsel if desired.

Lastly, the power to deport aliens is an act of the State and done under the authority of the sovereign power. It a police
measure against the undesirable aliens whose continued presence in the country is found to be injurious to the public
good and tranquility of the people.
Alvarez vs. Court of First Instance of Tayabas
GR 45358, 29 January 1937

Facts:
On 3 June 1936, the chief of the secret service of the Anti-Usury Board, of the Department of Justice, presented to Judge
Eduardo Gutierrez David then presiding over the Court of First Instance of Tayabas, an affidavit alleging that according to
reliable information, Narciso Alvarez kept in his house in Infanta, Tayabas, books, documents, receipts, lists, chits and
other papers used by him in connection with his activities as a moneylender, charging usurious rates of interest in
violation of the law. In his oath at the end of the affidavit, the chief of the secret service stated that his answers to the
questions were correct to the best of his knowledge and belief. He did not swear to the truth of his statements upon his
own knowledge of the facts but upon the information received by him from a reliable person. Upon the affidavit the
judge, on said date, issued the warrant which is the subject matter of the petition, ordering the search of the Alvarez's
house at any time of the day or night, the seizure of the books and documents and the immediate delivery thereof to
him to be disposed of in accordance with the law. With said warrant, several agents of the Anti-Usury Board entered
Alvarez's store and residence at 7:00 p.m. of 4 June 1936, and seized and took possession of the following articles:
internal revenue licenses for the years 1933 to 1936, 1 ledger, 2 journals, 2 cashbooks, 9 order books, 4 notebooks, 4
check stubs, 2 memorandums, 3 bankbooks, 2 contracts, 4 stubs, 48 stubs of purchases of copra, 2 inventories, 2
bundles of bills of lading, 1 bundle of credit receipts, 1 bundle of stubs of purchases of copra, 2 packages of
correspondence, 1 receipt book belonging to Luis Fernandez, 14 bundles of invoices and other papers, many documents
and loan contracts with security and promissory notes, 504 chits, promissory notes and stubs of used checks of the
Hongkong & Shanghai Banking Corporation (HSBC). The search for and seizure of said articles were made with the
opposition of Alvarez who stated his protest below the inventories on the ground that the agents seized even the
originals of the documents. As the articles had not been brought immediately to the judge who issued the search
warrant, Alvarez, through his attorney, filed a motion on 8 June 1936, praying that the agent Emilio L. Siongco, or any
other agent, be ordered immediately to deposit all the seized articles in the office of the clerk of court and that said
agent be declared guilty of contempt for having disobeyed the order of the court. On said date the court issued an order
directing Siongco to deposit all the articles seized within 24 hours from the receipt of notice thereof and giving him a
period of 5 days within which to show cause why he should not be punished for contempt of court. On 10 June, Attorney
Arsenio Rodriguez, representing the Anti-Usury Board, filed a motion praying that the order of the 8th of said month be
set aside and that the Anti-Usury Board be authorized to retain the articles seized for a period of 30 days for the
necessary investigation. On June 25, the court issued an order requiring agent Siongco forthwith to file the search
warrant and the affidavit in the court, together with the proceedings taken by him, and to present an inventory duly
verified by oath of all the articles seized. On July 2, the attorney for the petitioner filed a petition alleging that the search
warrant issued was illegal and that it had not yet been returned to date together with the proceedings taken in
connection therewith, and praying that said warrant be cancelled, that an order be issued directing the return of all the
articles seized to Alvarez, that the agent who seized them be declared guilty of contempt of court, and that charges be
filed against him for abuse of authority. On September 10, the court issued an order holding: that the search warrant
was obtained and issued in accordance with the law, that it had been duly complied with and, consequently, should not
be cancelled, and that agent Siongco did not commit any contempt of court and must, therefore, be exonerated, and
ordering the chief of the Anti-Usury Board in Manila to show cause, if any, within the unextendible period of 2 days from
the date of notice of said order, why all the articles seized appearing in the inventory should not be returned to Alvarez.
The assistant chief of the Anti-Usury Board of the Department of Justice filed a motion praying, for the reasons stated
therein, that the articles seized be ordered retained for the purpose of conducting an investigation of the violation of
the Anti-Usury Law committed by Alvarez. On October 10, said official again filed another motion alleging that he
needed 60 days to examine the documents and papers seized, which are designated on pages 1 to 4 of the inventory by
Nos. 5, 10, 16, 23, 25-27, 30-31 , 34, 36-43 and 45, and praying that he be granted said period of 60 days. In an order of
October 16, the court granted him the period of 60 days to investigate said 19 documents. Alvarez, herein, asks that the
search warrant as well as the order authorizing the agents of the Anti-Usury Board to retain the articles seized, be
declared illegal and set aside, and prays that all the articles in question be returned to him.

Issue:
Whether the search warrant issued by the court is illegal because it has been based upon the affidavit of
agent Almeda in whose oath he declared that he had no personal knowledge of the facts which were to serve as a basis
for the issuance of the warrant but that he had knowledge thereof through mere information secured from a person
whom he considered reliable, and that it is illegal as it was not supported by other affidavits aside from that made by the
applicant.

Held:
Section 1, paragraph 3, of Article III of the Constitution and Section 97 of General Orders 58 require that there be not
only probable cause before the issuance of a search warrant but that the search warrant must be based upon an
application supported by oath of the applicant and the witnesses he may produce. In its broadest sense, an oath
includes any form of attestation by which a party signifies that he is bound in conscience to perform an act faithfully and
truthfully; and it is sometimes defined as an outward pledge given by the person taking it that his attestation or promise
is made under an immediate sense of his responsibility to God. The oath required must refer to the truth of the facts
within the personal knowledge of the petitioner or his witnesses, because the purpose thereof is to convince the
committing magistrate, not the individual making the affidavit and seeking the issuance of the warrant, of the existence
of probable cause. The true test of sufficiency of an affidavit to warrant issuance of a search warrant is whether it has
been drawn in such a manner that perjury could be charged thereon and affiant be held liable for damages caused. The
affidavit, which served as the exclusive basis of the search warrant, is insufficient and fatally defective by reason of the
manner in which the oath was made, and therefore, the search warrant and the subsequent seizure of the books,
documents and other papers are illegal. Further, it is the practice in this jurisdiction to attach the affidavit of at least the
applicant or complainant to the application. It is admitted that the judge who issued the search warrant in this case,
relied exclusively upon the affidavit made by agent Almeda and that he did not require nor take the deposition of any
other witness. Neither the Constitution nor General Orders 58 provides that it is of imperative necessity to take the
depositions of the witnesses to be presented by the applicant or complainant in addition to the affidavit of the latter.
The purpose of both in requiring the presentation of depositions is nothing more than to satisfy the committing
magistrate of the existence of probable cause. Therefore, if the affidavit of the applicant or complainant is sufficient, the
judge may dispense with that of other witnesses. Inasmuch as the affidavit of the agent was insufficient because his
knowledge of the facts was not personal but merely hearsay, it is the duty of the judge to require the affidavit of one or
more witnesses for the purpose of determining the existence of probable cause to warrant the issuance of the search
warrant. When the affidavit of the applicant or complainant contains sufficient facts within his personal and direct
knowledge, it is sufficient if the judge is satisfied that there exists probable cause; when the applicant's knowledge of
the facts is mere hearsay, the affidavit of one or more witnesses having a personal knowledge of the facts is necessary.
Thus the warrant issued is likewise illegal because it was based only on the affidavit of the agent who had no personal
knowledge of the facts.
Mata vs. Bayona
GR 50720, 26 March 1984

Facts:
Soriano Mata was accused under Presidential Decree (PD) 810, as amended by PD 1306, the
information against him alleging that Soriano Mata offered, took and arranged bets on the Jai Alai game by "selling
illegal tickets known as 'Masiao tickets' without any authority from the Philippine Jai Alai &
Amusement Corporation or from the government authorities concerned." Mata claimed that during the
hearing of the case, he discovered that nowhere from the records of the said case could be found the search warrant
and other pertinent papers connected to the issuance of the same, so that he had to inquire from the City Fiscal its
whereabouts, and to which inquiry Judge Josephine K. Bayona, presiding Jufe of the City Court of Ormoc replied, "it is
with the court". The Judge then handed the records to the Fiscal who attached them to the records. This led Mata to file
a motion to quash and annul the search warrant and for the return of the articles seized, citing and invoking, among
others, Section 4 of Rule 126 of the Revised Rules of Court. The motion was denied by the Judge on 1 March 1979,
stating that the court has made a thorough investigation and examination under oath of Bernardo U. Goles and
Reynaldo T. Mayote, members of the Intelligence Section of 352nd PC Co./Police District II INP; that in fact the court
made a certification to that effect; and that the fact that documents relating to the search warrant were not attached
immediately to the record of the criminal case is of no moment, considering that the rule does not specify when these
documents are to be attached to the records. Mata's motion for reconsideration of the aforesaid order having been
denied, he came to the Supreme Court, with the petition for certiorari, praying, among others, that the Court declare
the search warrant to be invalid for its alleged failure to comply with the requisites of the Constitution and the Rules of
Court, and that all the articles confiscated under such warrant as inadmissible as evidence in the case, or in any
proceedings on the matter.

Issue:
Whether the judge must before issuing the warrant personally examine on oath or affirmation the complainant and any
witnesses he may produce and take their depositions in writing, and attach them to the record, in addition to any
affidavits presented to him.

Held:
Under the Constitution "no search warrant shall issue but upon probable cause to be determined by the Judge or such
other responsible officer as may be authorized by law after examination under oath or affirmation of the complainant
and the witnesses he may produce". More emphatic and detailed is the implementing rule of the constitutional
injunction, The Rules provide that the judge must before issuing the warrant personally examine on oath or affirmation
the complainant and any witnesses he may produce and take their depositions in writing, and attach them to the record,
in addition to any affidavits presented to him. Mere affidavits of the complainant and his witnesses are thus not
sufficient. The examining Judge has to take depositions in writing of the complainant and the witnesses he may produce
and to attach them to the record. Such written deposition is necessary in order that the Judge may be able to properly
determine the existence or nonexistence of the probable cause, to hold liable for perjury the person giving it if it will be
found later that his declarations are false. We, therefore, hold that the search warrant is tainted with illegality by the
failure of the Judge to conform with the essential requisites of taking the depositions in writing and attaching them to
the record, rendering the search warrant invalid.
PEOPLE VS. DEL ROSARIO
234 SCRA 246; G.R. NO. 109633; 20 JUL 1994

Facts:
Accused was charged and convicted by the trial court of illegal possession of firearms and illegal possession and sale of
drugs, particularly methamphetamine or shabu. After the issuance of the search warrant, which authorized the search
and seizure of an undetermined quantity of methamphetamine and its paraphernalia’s, an entrapment was planned that
led to the arrest of del Rosario and to the seizure of the shabu, its paraphernalia’s and of a .22 caliber pistol with 3 live
ammunition.

Issue:
Whether or Not the seizure of the firearms was proper.

Held:
No. Sec 2 art. III of the constitution specifically provides that a search warrant must particularly describe the things to be
seized. In herein case, the only objects to be seized that the warrant determined was the methamphetamine and the
paraphernalia’s therein. The seizure of the firearms was unconstitutional.

Wherefore the decision is reversed and the accused is acquitted.


In RE: Umil, Umil vs. Ramos
GR 81567, 9 July 1990
also Roque vs. de Villa [GR 84581-82],
In RE: Anonuevo. Anonuevo vs. Ramos [GR 84583-84], In RE: Ocaya. Ocaya vs. Aguirre [GR
83162], In RE: Espiritu. Espiritu vs. Lim [GR 85727], and In RE: Nazareno. Nazareno vs.
Station Commander of Muntinlupa Police Station [GR 86332]

Facts: [GR 81567]


On 1 February 1988, the Regional Intelligence Operations Unit of the Capital Command (RIOU-CAPCOM) received
confidential information about a member of the NPA Sparrow Unit (liquidation squad) being treated for a gunshot
wound at the St. Agnes Hospital in Roosevelt Avenue, Quezon City. Upon verification, it was found that the wounded
person, who was listed in the hospital records as Ronnie Javelon, is actually Rolando Dural, a member of the NPA
liquidation squad, responsible for the killing of 2 CAPCOM soldiers the day before, or on 31 January 1988, in Macanining
Street, Bagong Barrio, Caloocan City. In view of this verification, Dural was transferred to the Regional Medical Services
of the CAPCOM, for security reasons. While confined thereat, or on 4 February 1988, Dural was positively identified by
eyewitnesses as the gunman who went on top of the hood of the CAPCOM mobile patrol car, and fired at the 2 CAPCOM
soldiers seated inside the car identified as T/Sgt. Carlos Pabon and CIC Renato Manligot. As a consequence of this
positive identification, Dural was referred to the Caloocan City Fiscal who conducted an inquest and thereafter filed with
the Regional Trial Court of Caloocan City an information charging Rolando Dural alias Ronnie Javelon with the crime of
"Double Murder with Assault Upon Agents of Persons in Authority." (Criminal Case C-30112; no bail recommended). On
15 February 1988, the information was amended to include, as defendant, Bernardo Itucal, Jr. who, at the filing of the
original information, was still unidentified.
Meanwhile, on 6 February 1988, a petition for habeas corpus was filed with the Supreme Court on behalf of Roberto
Umil, Rolando Dural, and Renato Villanueva. The Court issued the writ of habeas corpus on 9 February 1988 and Fidel V.
Ramos, Maj. Gen. Renato de Villa, Brig. Gen. Ramon Montano, and Brig. Gen. Alexander Aguirre filed a Return of the
Writ on 12 February 1988. Thereafter, the parties were heard on 15 February 1988. On 26 February 1988, however, Umil
and Villanueva posted bail before the Regional Trial Court of Pasay City where charges for violation of the Anti-
Subversion Act had been filed against them, and they were accordingly released.

Issue:
Whether Dural can be validly arrested without any warrant of arrest for the crime of rebellion.

Held:
Dural, it clearly appears that he was not arrested while in the act of shooting the 2 CAPCOM soldiers nor was he arrested
just after the commission of the said offense for his arrest came a day after the said shooting incident. Seemingly, his
arrest without warrant is unjustified. However, Dural was arrested for being a member of the New Peoples Army (NPA),
an outlawed subversive organization. Subversion being a continuing offense, the arrest of Rolando Dural without
warrant is justified as it can be said that he was committing an offense when arrested. The crimes of rebellion,
subversion, conspiracy or proposal to commit such crimes, and crimes or offenses committed in furtherance thereof or
in connection therewith constitute direct assaults against the State and are in the nature of continuing crimes. The
arrest of persons involved in the rebellion whether as its fighting armed elements, or for committing non-violent acts
but in furtherance of the rebellion, is more an act of capturing them in the course of an armed conflict, to quell the
rebellion, than for the purpose of immediately prosecuting them in court for a statutory offense. The arrest, therefore,
need not follow the usual procedure in the prosecution of offenses which requires the determination by a judge of the
existence of probable cause before the issuance of a judicial warrant of arrest and the granting of bail if the offense is
bailable. Obviously, the absence of a judicial warrant is no legal impediment to arresting or capturing persons
committing overt acts of violence against government forces, or any other milder acts but equally in pursuance of the
rebellious movement. The arrest or capture is thus impelled by the exigencies of the situation that involves the very
survival of society and its government and duly constituted authorities.
UMIL VS. RAMOS
187 SCRA 311; G.R. NO. 81567; 3 OCT 1991

Facts:
On 1 February 1988, military agents were dispatched to the St. Agnes Hospital, Roosevelt Avenue, Quezon City, to verify
a confidential information which was received by their office, about a "sparrow man" (NPA member) who had been
admitted to the said hospital with a gunshot wound. That the wounded man in the said hospital was among the five (5)
male "sparrows" who murdered two (2) Capcom mobile patrols the day before, or on 31 January 1988 at about 12:00
o'clock noon, before a road hump along Macanining St., Bagong Barrio, Caloocan City. The wounded man's name was
listed by the hospital management as "Ronnie Javellon," twenty-two (22) years old of Block 10, Lot 4, South City Homes,
Biñan, Laguna however it was disclosed later that the true name of the wounded man was Rolando Dural. In view of this
verification, Rolando Dural was transferred to the Regional Medical Servicesof the CAPCOM, for security reasons. While
confined thereat, he was positively identified by the eyewitnesses as the one who murdered the 2 CAPCOM mobile
patrols.

Issue:
Whether or Not Rolando was lawfully arrested.

Held:
Rolando Dural was arrested for being a member of the NPA, an outlawed subversive organization. Subversion being a
continuing offense, the arrest without warrant is justified as it can be said that he was committing as offense when
arrested. The crimes rebellion, subversion, conspiracy or proposal to commit such crimes, and crimes or offenses
committed in furtherance therefore in connection therewith constitute direct assaults against the state and are in the
nature of continuing crimes.
198 People vs. Sucro
GR 93239, 18 March 1991

Facts:
On 21 March 1989, Pat. Roy Fulgencio, a member of the INP, Kalibo, Aklan, was instructed by P/Lt.
Vicente Seraspi, Jr. (Station Commander of the INP Kalibo, Aklan) to monitor the activities of Edison Sucro, because of
information gathered by Seraspi that Sucro was selling marijuana. As planned, at about 5:00 P.M. on said date, Pat.
Fulgencio positioned himself under the house of a certain Arlie Regalado at C. Quimpo Street. Adjacent to the house of
Regalado, about 2 meters away, was a chapel. Thereafter, Pat. Fulgencio saw Sucro enter the chapel, taking something
which turned out later to be marijuana from the compartment of a cart found inside the chapel, and then return to the
street where he handed the same to a buyer, Aldie Borromeo. After a while Sucro went back to the chapel and again
came out with marijuana which he gave to a group of persons. It was at this instance that Pat. Fulgencio radioed P/Lt.
Seraspi and reported the activity going on P/Lt. Seraspi instructed Pat. Fulgencio to continue monitoring developments.
At about 6:30 P.M., Pat. Fulgencio again called up Seraspi to report that a third buyer later identified as Ronnie
Macabante, was transacting with Sucro. At that point, the team of P/Lt Seraspi proceeded to the area and while the
police officers were at the Youth Hostel at Maagma St., Pat. Fulgencio told P/Lt. Seraspi to intercept Macabante and
Sucro. P/ Lt. Seraspi and his team caught up with Macabante at the crossing of Mabini and Maagma Sts. In front of the
Aklan Medical Center. Upon seeing the police, Macabante threw something to the ground which turned out to be a tea
bag of marijuana. When confronted, Macabante readily admitted that he bought the same from Sucro in front of the
chapel. The police team was able to overtake and arrest Sucro at the corner of C. Quimpo and Veterans Sts. The police
recovered 19 sticks and 4 teabags of marijuana from the cart inside
the chapel and another teabag from Macabante. The teabags of marijuana were sent to the PC-INP Crime Laboratory
Service, at Camp Delgado, Iloilo City for analysis. The specimens were all found positive of marijuana. Sucro was charged
with violation of Section 4, Article II of the Dangerous Drugs Act. Upon arraignment, Sucro, assisted by counsel, entered
a plea of "not guilty" to the offense charged. Trial ensued and a judgment of conviction was rendered, finding Sucro
guilty of the sale of prohibited drug and sentencing him to suffer the penalty of life imprisonment, and pay a fine of
P20,000, and costs. Sucro appealed.

Issue:
Whether the arrest without warrant of the accused is lawful and consequently, whether the evidence resulting from
such arrest is admissible.

Held:
Section 5, Rule 113 of the Rules on Criminal Procedure provides for the instances where arrest without warrant is
considered lawful. The rule states that "A peace officer or private person may, without warrant, arrest a person: (a)
When in his presence, the person to be arrested has committed, is actually committing, or is attempting to commit an
offense; (b) When an offense has in fact just been committed, and he has personal knowledge of facts indicating that
the person to be arrested has committed it;" An offense is committed in the presence or within the view of an officer,
within the meaning of the rule authorizing an arrest without a warrant, when the officer sees the offense, although at a
distance, or hears the disturbances created thereby and proceeds at once to the scene thereof. The failure of the police
officers to secure a warrant stems from the fact that their knowledge acquired from the surveillance was insufficient to
fulfill the requirements for the issuance of a search warrant. What is paramount is that probable cause existed. Still, that
searches and seizures must be supported by a valid warrant is not an absolute rule. Among the exceptions granted by
law is a search incidental to a lawful arrest under Sec. 12, Rule 126 of the Rules on Criminal Procedure, which provides
that a person lawfully arrested may be searched for dangerous weapons or anything which may be used as proof of the
commission of an offense, without a search warrant. Herein, police officers have personal knowledge of the actual
commission of the crime when it had earlier conducted surveillance activities of the accused. Under the circumstances
(monitoring of transactions) there existed probable cause for the arresting officers, to arrest Sucro who was in fact
selling marijuana and to seize the contraband. Thus, as there is nothing unlawful about the arrest considering its
compliance with the requirements of a warrantless arrest; ergo, the fruits obtained from such lawful arrest are
admissible in evidence.
People vs. Rodrigueza
GR 95902, 4 February 1992

Facts:
[Prosecution] At around 5:00 p.m. of 1 July 1987, CIC Ciriaco Taduran was in their headquarters at the Office of the
Narcotics Regional Unit at Camp Bagong Ibalon, Legaspi City, together with S/Sgt. Elpidio Molinawe, CIC Leonardo B.
Galutan and their commanding officer, Major Crisostomo M. Zeidem, when a confidential informer arrived and told
them that there was an ongoing illegal traffic of prohibited drugs in Tagas, Daraga, Albay. Major Zeidem formed a team
to conduct a buybust operation, which team was given P200.00 in different denominations to buy marijuana. These bills
were treated with ultraviolet powder at the Philippine Constabulary Crime Laboratory (PCCL). Sgt. Molinawe gave the
money to Taduran who acted as the poseur buyer. He was told to look for a certain Don, the alleged seller of prohibited
drugs. Taduran went to Tagas alone and, while along the road, he met Samuel Segovia. He asked Segovia where he could
find Don and where he could buy marijuana. Segovia left for a while and when he returned, he was accompanied by a
man who was later on introduced to him as Don Rodrigueza. After agreeing on the price of P200.00 for 100 grams of
marijuana, Don halted a passing tricycle driven by Antonio Lonceras. He boarded it and left Taduran and Segovia. When
he came back, Don gave Taduran "a certain object wrapped in a plastic" which was later identified as marijuana, and
received payment therefor. Thereafter, Taduran returned to the headquarters and made a report regarding his said
purchase of marijuana. Based on that information, Major Zeidem ordered a team to conduct an operation to apprehend
the suspects. In the evening of the same date, CIC Galutan and S/Sgt. Molinawe proceeded to Regidor Street, Daraga,
Albay and arrested Rodrigueza, Antonio Lonceras and Samuel Segovia. The constables were not, however, armed with a
warrant of arrest when they apprehended the three accused. The arrests were brought to the headquarters for
investigation. Thereafter, agents of the Narcotics Command (NARCOM) conducted a raid in the house of Jovencio
Rodrigueza, Don's father. Taduran did not go with them. During the raid, they were able to confiscate dried marijuana
leaves and a plastic syringe, among others. The search, however, was not authorized by any search warrant. The next
day, Jovencio Rodrigueza was released from detention but Don Rodrigueza was detained. [Defense] Don Rodrigueza, on
the other hand, claimed that on said date he was in the house of his aunt in San Roque, Legaspi City. He stayed there
overnight and did not leave the place until the next day when his brother arrived and told him that their father was
taken by some military men the preceding night. Rodrigueza went to Camp Bagong Ibalon and arrived there at around
8:00 a.m. of 2 July 1987. When he arrived, he was asked if he knew anything about the marijuana incident, to which
question he answered in the negative. Like Segovia, he was made to hold a P10.00 bill and was brought to the crime
laboratory for examination. From that time on, he was not allowed to go home and was detained inside the camp. He
was also tortured in order to make him admit his complicity in the alleged sale of marijuana.
On 10 July 1987, Don Rodrigueza, Samuel Segovia and Antonio Lonceras, for possession of 100 grams of marijuana
leaves and for selling, in a buy-bust operation, said 100 grams of dried marijuana leaves for a consideration of P200.00.
During the arraignment, all the accused pleaded not guilty to the charge against them. The Regional Trial Court of
Legaspi City, Branch 10, found Don Rodrigueza guilty beyond reasonable doubt of violating Section 4, Article II of the
Dangerous Drugs Act of 1972 (Republic Act 6425, as amended) and sentenced him to suffer the penalty of life
imprisonment and to pay a fine of P20,000.00 and costs. The court, however, acquitted Segovia and Lonceres.
Rodrigueza appealed.

Issue:
Whether the time of Don Rodrigueza’s arrest is material in determining his culpability in the crime charged.

Held:
As provided in the present Constitution, a search, to be valid, must generally be authorized by a search warrant duly
issued by the proper government authority. True, in some instances, the Court has allowed government authorities to
conduct searches and seizures even without a search warrant. Thus, when the owner of the premises waives his right
against such incursion; when the search is incidental to a lawful arrest; when it is made on vessels and aircraft for
violation of customs laws; when it is made on automobiles for the purpose of preventing violations of smuggling or
immigration laws; when it involves prohibited articles in plain view; or in cases of inspection of buildings and other
premises for the enforcement of fire, sanitary and building regulations, a search may be validly made even without a
search warrant. Herein, however, the raid conducted by the NARCOM agents in the house of Jovencio Rodrigueza was
not authorized by any search warrant. It does not appear, either, that the situation falls under any of the
aforementioned cases. Hence, Rodrigueza's right against unreasonable search and seizure was clearly violated. The
NARCOM agents could not have justified their act by invoking the urgency and necessity of the situation because the
testimonies of the prosecution witnesses reveal that the place had already been put under surveillance for quite some
time. Had it been their intention to conduct the raid, then they should, because they easily could, have first secured a
search warrant during that time. Further, the inconsistencies made by prosecution witnesses give more credibility to the
testimony of Don Rodrigueza. While it is true that Rodrigueza's defense amounts to an alibi, and as such is the weakest
defense in a criminal prosecution, there are, nonetheless, some evidentiary aspects pointing to the truth in his
testimony. Firstly, the Joint Affidavit of Arrest corroborates his testimony that he was not among those who were
arrested on the night of 1 July 1987. His co-accused Segovia also testified that Rodrigueza was not with them when they
were apprehended by the NARCOM agents. Hence, Rodrigueza is acquitted of the crime charged, due to the failure of
the prosecution to establish its cause.
Go vs. Court of Appeals
GR 101837, 11 February 1992

Facts:
On 2 July 1991, Eldon Maguan was driving his car along Wilson St., San Juan, Metro Manila, heading towards P. Guevarra
St. Rolito Go y Tambunting entered Wilson St., where it is a one-way street and started traveling in the opposite or
"wrong" direction. At the corner of Wilson and J. Abad Santos Sts., Go's and Maguan's cars nearly bumped each other.
Go alighted from his car, walked over and shot Maguan inside his car. Go then boarded his car and left the scene. A
security guard at a nearby restaurant was able to take down Go's car plate number. The police arrived shortly thereafter
at the scene of the shooting and there retrieved an empty shell and one round of live ammunition for a 9mm caliber
pistol. Verification at the Land Transportation Office showed that the car was registered to one Elsa Ang Go. The
following day, the police returned to the scene of the shooting to find out where the suspect had come from; they were
informed that Go had dined at Cravings Bake Shop shortly before the shooting. The police obtained a facsimile or
impression of the credit card used by Go from the cashier of the bake shop. The security guard of the bake shop was
shown a picture of Go and he positively identified him as the same person who had shot Maguan. Having established
that the assailant was probably Go, the police launched a manhunt for Go. On 8 July 1991, Go presented himself before
the San Juan Police Station to verify news reports that he was being hunted by the police; he was accompanied by two
(2) lawyers. The police forthwith detained him. An eyewitness to the shooting, who was at the police station at that
time, positively identified Go as the gunman. That same day, the police promptly filed a complaint for frustrated
homicide against Go with the Office of the Provincial Prosecutor of Rizal. First Assistant Provincial Prosecutor Dennis
Villa Ignacio ("Prosecutor") informed Go, in the Presence of his lawyers. that he could avail himself of his right to
preliminary investigation but that he must first sign a waiver of the provisions of Article 125 of the Revised Penal Code.
Go refused to execute any such waiver. On 9 July 1991, while the complaint was still with the Prosecutor, and before an
information could be filed in court, the victim, Eldon Maguan, died of his gunshot wound(s). Accordingly, on 11 July
1991, the Prosecutor, instead of filing an information for frustrated homicide, filed an information for murder before the
Regional Trial Court. No bail was recommended. At the bottom of the information, the Prosecutor certified that no
preliminary investigation had been conducted because the accused did not execute and sign a waiver of the provisions
of Article 125 of the Revised Penal Code. In the afternoon of 11 July 1991, Go's counsel filed with the prosecutor an
omnibus motion for immediate release and proper preliminary investigation, alleging that the warrantless arrest of Go
was unlawful and that no preliminary investigation had been conducted before the information was filed. On 12 July
1991, Go filed an urgent ex-parte motion for special raffle in order to expedite action on the Prosecutor's bail
recommendation. The case was raffled to the sala of Judge Benjamin V. Pelayo (Branch 168, RTC of Pasig City), who, on
the same date, approved the cash bond posted by Go and ordered his release. Go was in fact released that same day. On
16 July 1991, the Prosecutor filed with the Regional Trial Court a motion for leave to conduct preliminary investigation
and prayed that in the meantime all proceedings in the court be suspended. On the said date, the trial court issued an
Order 9 granting leave to conduct preliminary investigation and cancelling the arraignment set for 15
August 1991 until after the prosecution shall have concluded its preliminary investigation. On 17 July 1991, however, the
Judge motu proprio issued an Order, (1) recalling the 12 July 1991 Order which granted bail: petitioner was given 48
hours from receipt of the Order to surrender himself: (2) recalling and cancelling the 16 July 1991 Order which granted
leave to the Prosecutor to conduct preliminary investigation: (3) treating Go's omnibus motion for immediate release
and preliminary investigation dated 11 July 1991 as a petition for bail and set for hearing on 23 July 1991. On 19 July
1991, Go filed a petition for certiorari, prohibition and mandamus before the Supreme Court assailing the 17 July 1991
Order. Go also moved for suspension of all proceedings in the case pending resolution by the Supreme Court of his
petition: this motion was, however, denied by Judge Pelayo. On 23 July 1991, Go surrendered to the police. By a
Resolution dated 24 July 1991, the Supreme Court remanded the petition for certiorari, prohibition and mandamus to
the Court of Appeals. On 16 August 1991, Judge Pelayo issued an order in open court setting Go's arraignment on 23
August 1991. On 19 August 1991, Go filed with the Court of Appeals a motion to restrain his arraignment. On 23 August
1991, Judge Pelayo issued a Commitment Order directing the Provincial Warden of Rizal to admit Go into his custody at
the Rizal Provincial Jail. On the same date, Go was arraigned. In view, however, of his refusal to enter a plea, the trial
court entered for him a plea of not guilty. The trial court then set the criminal case for continuous hearings on 19, 24
and 26 September; on 2, 3, 11 and 17 October; and on 7, 8, 14, 15, 21 and 22 November 1991. On 27 August 1991. Go
filed a petition for habeas corpus in the Court of Appeals. On 30 August 1991, the Court of Appeals issued the writ of
habeas corpus. The petition for certiorari, prohibition and mandamus, on the one hand, and the petition for habeas
corpus, upon the other, were subsequently consolidated in the Court of Appeals. The Court of Appeals, on 2 September
1991, issued a resolution denying Go's motion to restrain his arraignment on the ground that motion had become moot
and academic. On 19 September 1991, trial of the criminal case commenced. On 23 September 1991, the Court of
Appeals rendered a consolidated decision dismissing the 2 petitions on the grounds that Go's warrantless arrest was
valid and Go's act of posting bail constituted waiver of any irregularity attending his arrest, among others. On 3 October
1991, the prosecution presented three (3) more witnesses at the trial. Go's Counsel also filed a "Withdrawal of
Appearance" with the trial court, with Go's conformity. On 4 October 1991, Go filed the present petition for Review on
Certiorari. On 14 October 1991, the Court issued a Resolution directing Judge Pelayo to held in abeyance the hearing of
the criminal case below until further orders from the Supreme Court.

Issue:
Whether Go was arrested legally without warrant for the killing of Maguan, and is thus not entitled to be released
pending the conduct of a preliminary investigation.

Held:
Go's warrantless "arrest" or detention does not fall within the terms of Section 5 of Rule 113 of the 1985 Rules on
Criminal Procedure which provides that "A peace officer or a private person may, without a warrant, arrest a person: (a)
When, in his presence, the person to be created has committed, is actually committing, or is attempting to commit an
offense; (b) When an offense has in fact just been ommitted, and he has personal knowledge of facts indicating that the
person to be arrested has committed it; and (c) When the person to be arrested is a prisoner who has escaped from a
penal establishment or place where he is serving final judgment or temporarily confined while his case is pending or has
escaped while being transferred from one confinement to another. In cases falling under paragraphs (a) and (b) hereof,
the person arrested without a warrant shall be forthwith delivered to the nearest police station or jail, and he shall be
proceeded against in accordance with Rule 112, Section 7." Go's "arrest" took place 6 days after the shooting of
Maguan. The "arresting" officers obviously were not present, within the meaning of Section 5(a), at the time Go had
allegedly shot Maguan. Neither could the "arrest" effected 6 days after the shooting be reasonably regarded as effected
"when [the shooting had] in fact just been committed" within the meaning of Section 5 (b). Moreover, none of the
"arresting" officers had any "personal knowledge" of facts indicating that Go was the gunman who had shot Maguan.
The information upon which the police acted had been derived from statements made by alleged eyewitnesses to the
shooting -- one stated that Go was the gunman another was able to take down the alleged gunman's car's plate number
which turned out to be registered in Go's wife's name. That information did not, however, constitute "personal
knowledge." It is thus clear to the Court that there was no lawful warrantless arrest of Go within the meaning of Section
5 of Rule 113. It is clear too that Section 7 of Rule 112 is also not applicable. Indeed, Go was not arrested at all. When he
walked into the
San Juan Police Station, accompanied by two (2) lawyers, he in fact placed himself at the disposal of the police
authorities. He did not state that he was "surrendering" himself, in all probability to avoid the implication he was
admitting that he had slain Eldon Maguan or that he was otherwise guilty of a crime.
When the police filed a complaint for frustrated homicide with the Prosecutor, the latter should have immediately
scheduled a preliminary investigation to determine whether there was probable cause for charging Go in court for the
killing of Eldon Maguan. Instead, as noted earlier, the Prosecutor proceeded under the erroneous supposition that
Section 7 of Rule 112 was applicable and required Go to waive the provisions of Article 125 of the Revised Penal Code as
a condition for carrying out a preliminary investigation. This was substantive error, for Go was entitled to a preliminary
investigation and that right should have been accorded him without any conditions. Moreover, since Go had not been
arrested; with or without a warrant, he was also entitled to be released forthwith subject only to his appearing at the
preliminary investigation.
POSADAS VS. COURT OF APPEALS
188 SCRA 288; G.R. NO. 89139; 2 AUG 1990

Facts:
Members of the Integrated National Police (INP) of the Davao Metrodiscom assigned with the Intelligence Task Force,
Pat. Ursicio Ungab and Pat. Umbra Umpar conducted surveillance along Magallanes Street, Davao City. While in the
vicinity of Rizal Memorial Colleges they spotted petitioner carrying a "buri" bag and they noticed him to be acting
suspiciously. They approached the petitioner and identified themselves as members of the INP. Petitioner attempted to
flee but his attempt to get away was unsuccessful. They then checked the "buri" bag of the petitioner where they found
one (1) caliber .38 Smith & Wesson revolver with Serial No. 770196, two (2) rounds of live ammunition for a .38 caliber
gun, a smoke (tear gas) grenade, and two (2) live ammunitions for a .22 caliber gun. They brought the petitioner to the
police station for further investigation. In the course of the same, the petitioner was asked to show the necessary license
or authority to possess firearms and ammunitions found in his possession but he failed to do so. He was then taken to
the Davao Metrodiscom office and the prohibited articles recovered from him were indorsed to M/Sgt. Didoy the officer
then on duty. He was prosecuted for illegal possession of firearms and ammunitions in the Regional Trial Court of Davao
City.

Issue:
Whether or Not the warantless search is valid.

Held:
In justifying the warrantless search of the buri bag then carried by the petitioner, argues that under Section 12, Rule 136
of the Rules of Court a person lawfully arrested may be searched for dangerous weapons or anything used as proof of a
commission of an offense without a search warrant. It is further alleged that the arrest without a warrant of the
petitioner was lawful under the circumstances.

in the case at bar, there is no question that, indeed, it is reasonable considering that it was effected on the basis of a
probable cause. The probable cause is that when the petitioner acted suspiciously and attempted to flee with the buri
bag there was a probable cause that he was concealing something illegal in the bag and it was the right and duty of the
police officers to inspect the same.

It is too much indeed to require the police officers to search the bag in the possession of the petitioner only after they
shall have obtained a search warrant for the purpose. Such an exercise may prove to be useless, futile and much too
late.

Clearly, the search in the case at bar can be sustained under the exceptions heretofore discussed, and hence, the
constitutional guarantee against unreasonable searches and seizures has not been violated.
PEOPLE V. MENGOTE
210 SCRA 174; G.R. NO. 87059; 22 JUN 1992

Facts:
The Western Police District received a telephone call from an informer that there were three suspicious looking persons
at the corner of Juan Luna and North Bay Boulevard in Tondo, Manila. A surveillance team of plainclothesmen was
forthwith dispatched to the place. The patrolmen saw two men looking from side to side, one of whom holding his
abdomen. They approached the persons and identified themselves as policemen, whereupon the two tried to run but
unable to escape because the other lawmen surrounded them. The suspects were then searched. One of them the
accused-appellant was found with a .38 caliber with live ammunitions in it, while his companion had a fan knife. The
weapons were taken from them and they were turned over to the police headquarters for investigation. An information
was filed before the RTC convicting the accused of illegal possession of firearm arm. A witness testified that the weapon
was among the articles stolen at his shop, which he reported to the police including the revolver. For his part, Mengote
made no effort to prove that he owned the fire arm or that he was licensed to possess it but instead, he claimed that the
weapon was planted on him at the time of his arrest. He was convicted for violation of P.D.1866 and was sentenced to
reclusion perpetua. In his appeal he pleads that the weapon was not admissible as evidence against him because it had
been illegally seized and therefore the fruit of a poisonous tree.

Issue:
Whether or not the warrantless search and arrest was illegal.

Held:
An evidence obtained as a result of an illegal search and seizure inadmissible in any proceeding for any purpose as
provided by Art. III sec 32 of the Constitution. Rule 113 sec.5 of the Rules of Court, provides arrest without warrant
lawful when: (a) the person to be arrested has committed, is actually committing, or is attempting to commit an offense,
(b) when the offense in fact has just been committed, and he has personal knowledge of the facts indicating the person
arrested has committed it and (c) the person to be arrested has escaped from a penal establishment or a place where
he is serving final judgment or temporarily confined while his case is pending, or has escaped while being transferred
from one confinement to another.

These requirements have not been established in the case at bar. At the time of the arrest in question, the accused
appellant was merely looking from side to side and holding his abdomen, according to the arresting officers themselves.
There was apparently no offense that has just been committed or was being actually committed or at least being
attempt by Mengote in their presence. Moreover a person may not be stopped and frisked in a broad daylight or on a
busy street on unexplained suspicion.

Judgment is reversed and set aside. Accused-appellant is acquitted.


Malacat vs. Court of Appeals
GR 123595, 12 December 1997

Facts:
On 27 August 1990, at about 6:30 p.m., allegedly in response to bomb threats reported seven days
earlier, Rodolfo Yu of the Western Police District, Metropolitan Police Force of the Integrated National
Police, Police Station No. 3, Quiapo, Manila, was on foot patrol with three other police officers (all of them in uniform)
along Quezon Boulevard, Quiapo, Manila, near the Mercury Drug store at Plaza Miranda. They chanced upon two groups
of Muslim-looking men, with each group, comprised of three to four men, posted at opposite sides of the corner of
Quezon Boulevard near the Mercury Drug Store. These men were acting suspiciously with "their eyes moving very fast."
Yu and his companions positioned themselves at strategic points and observed both groups for about 30 minutes. The
police officers then approached one group of men, who then fled in different directions. As the policemen gave chase,
Yu caught up with and apprehended Sammy Malacat y Mandar (who Yu recognized, inasmuch as allegedly the previous
Saturday, 25 August 1990, likewise at Plaza Miranda, Yu saw Malacat and 2 others attempt to detonate a grenade).
Upon searching Malacat, Yu found a fragmentation grenade tucked inside the latter's "front waist line." Yu's companion,
police officer Rogelio Malibiran, apprehended Abdul Casan from whom a .38 caliber revolver was recovered. Malacat
and Casan were then brought to Police Station 3 where Yu placed an "X" mark at the bottom of the grenade and
thereafter gave it to his commander. Yu did not issue any receipt for the grenade he allegedly recovered from Malacat.
On 30 August 1990, Malacat was charged with violating Section 3 of Presidential Decree 1866. At arraignment on 9
October 1990, petitioner, assisted by counsel de officio, entered a plea of not guilty. Malacat denied the charges and
explained that he only recently arrived in Manila. However, several other police officers mauled him, hitting him with
benches and guns. Petitioner was once again searched, but nothing was found on him. He saw the grenade only in court
when it was presented. In its decision dated 10 February 1994 but promulgated on 15 February 1994, the trial court
ruled that the warrantless search and seizure of Malacat was akin to a "stop and frisk," where a "warrant and seizure can
be effected without necessarily being preceded by an arrest" and "whose object is either to maintain the status quo
momentarily while the police officer seeks to obtain more information"; and that the seizure of the grenade from
Malacat was incidental to a lawful arrest. The trial court thus found Malacat guilty of the crime of illegal possession of
explosives under Section 3 of PD 1866, and sentenced him to suffer the penalty of not less than 17 years, 4 months and
1 day of Reclusion Temporal, as minimum, and not more than 30 years of Reclusion Perpetua, as maximum. On 18
February 1994, Malacat filed a notice of appeal indicating that he was appealing to the Supreme Court. However, the
record of the case was forwarded to the Court of Appeals (CA-GR CR 15988). In its decision of 24 January 1996, the Court
of Appeals affirmed the trial court. Manalili filed a petition for review with the Supreme Court.

Issue:
Whether the search made on Malacat is valid, pursuant to the exception of “stop and frisk.”

Held:
The general rule as regards arrests, searches and seizures is that a warrant is needed in order to validly effect the same.
The Constitutional prohibition against unreasonable arrests, searches and seizures refers to those effected without a
validly issued warrant, subject to certain exceptions. As regards valid warrantless arrests, these are found in Section 5,
Rule 113 of the Rules of Court. A warrantless arrest under the circumstances contemplated under Section 5(a) has been
denominated as one "in flagrante delicto," while that under Section 5(b) has been described as a "hot pursuit" arrest.
Turning to valid warrantless searches, they are limited to the following: (1) customs searches; (2) search of moving
vehicles; (3) seizure of evidence in plain view; (4) consent searches; (5) a search incidental to a lawful arrest; and (6) a
"stop and frisk." The concepts of a "stop-and-frisk" and of a search incidental to a lawful arrest must not be confused.
These two types of warrantless searches differ in terms of the requisite quantum of proof before they may be validly
effected and in their allowable scope. In a search incidental to a lawful arrest, as the precedent arrest determines the
validity of the incidental search. Here, there could have been no valid in flagrante delicto or hot pursuit arrest preceding
the search in light of the lack of personal knowledge on the part of Yu, the arresting officer, or an overt physical act, on
the part of Malacat, indicating that a crime had just been committed, was being committed or was going to be
committed. Plainly, the search conducted on Malacat could not have been one incidental to a lawful arrest. On the other
hand, while probable cause is not required to conduct a "stop and frisk," it nevertheless holds that mere suspicion or a
hunch will not validate a "stop and frisk." A genuine reason must exist, in light of the police officer's experience and
surrounding conditions, to warrant the belief that the person detained has weapons concealed about him. Finally, a
"stop-and-frisk" serves a two-fold interest: (1) the general interest of effective crime prevention and detection, which
underlies the recognition that a police officer may, under appropriate circumstances and in an appropriate manner,
approach a person for purposes of investigating possible criminal behavior even without probable cause; and (2) the
more pressing interest of safety and self-preservation which permit the police officer to take steps to assure himself that
the person with whom he deals is not armed with a deadly weapon that could unexpectedly and fatally be used against
the police officer. Here, there are at least three (3) reasons why the "stop-and-frisk" was invalid: First, there is grave
doubts as to Yu's claim that Malacat was a member of the group which attempted to bomb Plaza Miranda 2 days earlier.
This claim is neither supported by any police report or record nor corroborated by any other police officer who allegedly
chased that group. Second, there was nothing in Malacat's behavior or conduct which could have reasonably elicited
even mere suspicion other than that his eyes were "moving very fast" — an observation which leaves us incredulous
since Yu and his teammates were nowhere near Malacat and it was already 6:30 p.m., thus presumably dusk. Malacat
and his companions were merely standing at the corner and were not creating any commotion or trouble. Third, there
was at all no ground, probable or otherwise, to believe that Malacat was armed with a deadly weapon. None was visible
to Yu, for as he admitted, the alleged grenade was "discovered" "inside the front waistline" of Malacat, and from all
indications as to the distance between Yu and Malacat, any telltale bulge, assuming that Malacat was indeed hiding a
grenade, could not have been visible to Yu. What is unequivocal then are blatant violations of Malacat's rights solemnly
guaranteed in Sections 2 and 12(1) of Article III of the Constitution.
PEOPLE VS. AMMINUDIN
163 SCRA 402; G.R. L-74869; 6 Jul 1988

Facts:
Idel Aminnudin, accused-appellant was arrested on June 25, 1984, shortly after disembarking from the M/V Wilcon 9 at
about 8:30 in the evening, in Iloilo City. The PC officers who were in fact waiting for him because of a tip from one their
informers simply accosted him, inspected his bag and finding what looked liked marijuana leaves took him to their
headquarters for investigation. The two bundles of suspect articles were confiscated from him and later taken to the NBI
laboratory for examination. It was found to contain three kilos of what were later analyzed as marijuana leaves by an
NBI forensic examiner. An information for violation of the Dangerous Drugs Act was filed against him. Later, the
information was amended to include Farida Ali y Hassen, who had also been arrested with him that same evening and
likewise investigated. Both were arraigned and pleaded not guilty. Subsequently, the fiscal filed a motion to dismiss the
charge against Ali on the basis of a sworn statement of the arresting officers absolving her after a 'thorough
investigation." The motion was granted, and trial proceeded only against the accused-appellant, who was eventually
convicted . In his defense, Aminnudin disclaimed the marijuana, averring that all he had in his bag was his clothing
consisting of a jacket, two shirts and two pairs of pants. He alleged that he was arbitrarily arrested and immediately
handcuffed. His bag was confiscated without a search warrant. At the PC headquarters, he was manhandled to force him
to admit he was carrying the marijuana, the investigator hitting him with a piece of wood in the chest and arms even as
he parried the blows while he was still handcuffed. He insisted he did not even know what marijuana looked like and
that his business was selling watches and sometimes cigarettes. However the RTC rejected his allegations. Saying that he
only has two watches during that time and that he did not sufficiently proved the injuries allegedly sustained.

Issue:
Whether or not search of defendant’s bag is legal.

Held:
The search was illegal. Defendant was not caught in flagrante delicto, which could allow warrantless arrest or search. At
the moment of his arrest, he was not committing a crime. Nor was he about to do so or had just done so. To all
appearances, he was like any of the other passengers innocently disembarking from the vessel. The said marijuana
therefore could not be appreciated as evidence against the defendant, and furthermore he is acquitted of the crime as
charged.
People v. Malmstedt
GR 91107, 19 June 1991

Facts:
Mikael Malmstedt, a Swedish national, entered the Philippines for the 3rd time in December 1988 as a tourist. He had
visited the country sometime in 1982 and 1985. In the evening of 7 May 1989, Malmstedt left for Baguio City. Upon his
arrival thereat in the morning of the following day, he took a bus to Sagada and stayed in that place for 2 days. On 11
May 1989, Capt. Alen Vasco of NARCOM, stationed at Camp Dangwa, ordered his men to set up a temporary checkpoint
at Kilometer 14, Acop, Tublay, Mountain Province, for the purpose of checking all vehicles coming from the Cordillera
Region. The order to establish a checkpoint in the said area was prompted by persistent reports that vehicles coming
from Sagada were transporting marijuana and other prohibited drugs. Moreover, information was received by the
Commanding Officer of NARCOM, that same morning, that a Caucasian coming from Sagada had in his possession
prohibited drugs. At about 1:30 pm, the bus where Malmstedt was riding was stopped. Sgt. Fider and CIC Galutan
boarded the bus and announced that they were members of the NARCOM and that they would conduct an inspection.
During the inspection, CIC Galutan noticed a bulge on Malmstedt's waist. Suspecting the bulge on Malmstedt's waist to
be a gun, the officer asked for Malmstedt's passport and other identification papers. When Malmstedt failed to comply,
the officer required him to bring out whatever it was that was bulging on his waist, which was a pouch bag. When
Malmstedt opened the same bag, as ordered, the officer noticed 4 suspicious-looking objects wrapped in brown packing
tape, which turned out to contain hashish, a derivative of marijuana, when opened. Malmstedt stopped to get 2
travelling bags from the luggage carrier, each containing a teddy bear, when he was invited outside the bus for
questioning. It was observed that there were also bulges inside the teddy bears which did not feel like foam stuffing.
Malmstedt was then brought to the headquarters of the NARCOM at Camp Dangwa for further investigation. At the
investigation room, the officers opened the teddy bears and they were found to also contain hashish. Representative
samples were taken from the hashish found among the personal effects of Malmstedt and the same were brought to the
PC Crime Laboratory for chemical analysis, which established the objects examined as hashish. Malmstedt claimed that
the hashish was planted by the NARCOM officers in his pouch bag and that the 2 travelling bags were not owned by him,
but were merely entrusted to him by an Australian couple whom he met in Sagada. He further claimed that the
Australian couple intended to take the same bus with him but because there were no more seats available in said bus,
they decided to take the next ride and asked Malmstedt to take charge of the bags, and that they would meet each
other at the Dangwa Station. An information was filed against Malmstedt for violation of the Dangerous Drugs Act.
During the arraignment, Malmstedt entered a plea of "not guilty." After trial and on 12 October 1989, the trial court
found Malmstedt guilty beyond reasonable doubt for violation of Section 4, Article II of RA 6425 and sentenced him to
life imprisonment and to pay a fine of P20,000. Malmstedt sought reversal of the decision of the trial court.

Issue:
Whether the personal effects of Malmstedt may be searched without an issued warrant.

Held:
The Constitution guarantees the right of the people to be secure in their persons, houses, papers and effects against
unreasonable searches and seizures. However, where the search is made pursuant to a lawful arrest, there is no need to
obtain a search warrant. A lawful arrest without a warrant may be made by a peace officer or a private person under the
following circumstances. Section 5 provides that “a peace officer or a private person may, without a warrant, arrest a
person (a) When, in his presence, the person to be arrested has committed, is actually committing, or is attempting to
commit an offense; (b) When an offense has in fact just been committed, and he has personal knowledge of facts
indicating that the person to be arrested has committed it; and (c) When the person to be arrested is a prisoner who has
escaped from a penal establishment or place where he is serving final judgment or temporarily confined while his case is
pending, or has escaped while being transferred from one confinement to another. In cases falling under paragraphs (a)
and (b) hereof, the person arrested without a warrant shall be forthwith delivered to the nearest police station or jail,
and he shall be proceeded against in accordance with Rule 112, Section 7." Herein, Malmstedt was caught in flagrante
delicto, when he was transporting prohibited drugs. Thus, the search made upon his personal effects falls squarely under
paragraph (1) of the foregoing provisions of law, which allow a warrantless search incident to a lawful arrest.
Espano vs. Court of Appeals
GR 120431, 1 April 1998

Facts:
On 14 July 1991, at about 12:30 a.m., Pat. Romeo Pagilagan and other police officers, namely, Pat. Wilfredo Aquilino,
Simplicio Rivera, and Erlindo Lumboy of the Western Police District (WPD), Narcotics
Division went to Zamora and Pandacan Streets, Manila to confirm reports of drug pushing in the area. They saw Rodolfo
Espano selling "something" to another person. After the alleged buyer left, they approached Espano, identified
themselves as policemen, and frisked him. The search yielded two plastic cellophane tea bags of marijuana . When asked
if he had more marijuana, he replied that there was more in his house. The policemen went to his residence where they
found ten more cellophane tea bags of marijuana. Espano was brought to the police headquarters where he was
charged with possession of prohibited drugs. On 24 July 1991, Espano posted bail and the trial court issued his order of
release on 29 July 1991. On 14 August 1992, the trial court rendered a decision, convicting Espano of the crime charged.
Espano appealed the decision to the Court of Appeals. The appellate court, however, on 15 January 1995 affirmed the
decision of the trial court in toto. Espano filed a petition for review with the Supreme Court.

Issue:
Whether the search of Espano’s home after his arrest does not violate against his right against unreasonable search and
seizure.

Held:
Espano's arrest falls squarely under Rule 113 Section 5(a) of the Rules of Court. He was caught in flagranti as a result of a
buy-bust operation conducted by police officers on the basis of information received regarding the illegal trade of drugs
within the area of Zamora and Pandacan Streets, Manila. The police officer saw Espano handing over something to an
alleged buyer. After the buyer left, they searched him and discovered two cellophanes of marijuana. His arrest was,
therefore, lawful and the two cellophane bags of marijuana seized were admissible in evidence, being the fruits of the
crime. As for the 10 cellophane bags of marijuana found at Espano's residence, however, the same inadmissible in
evidence. The articles seized from Espano during his arrest were valid under the doctrine of search made incidental to a
lawful arrest. The warrantless search made in his house, however, which yielded ten cellophane bags of marijuana
became unlawful since the police officers were not armed with a search warrant at the time. Moreover, it was beyond
the reach and control of Espano. The right of the people to be secure in their persons, houses, papers and effects against
unreasonable searches and seizures of whatever nature and for any purposes shall be inviolable, and no search warrant
or warrant of arrest shall issue except upon probable cause to be determined personally by the judge after examination
under oath or affirmation of the complainant and the witnesses he may produce, and particularly describing the place to
be searched and the persons or things to be seized." An exception to the said rule is a warrantless search incidental to a
lawful arrest for dangerous weapons or anything which may be used as proof of the commission of an offense. It may
extend beyond the person of the one arrested to include the premises or surroundings under his immediate control.
Herein, the ten cellophane bags of marijuana seized at petitioner's house after his arrest at Pandacan and Zamora
Streets do not fall under the said exceptions.
Papa vs. Mago
GR L-27360, 28 February 1968

Facts:
Martin Alagao, head of the counter-intelligence unit of the Manila Police Department, acting upon a reliable information
received on 3 November 1966 to the effect that a certain shipment of personal effects, allegedly misdeclared and
undervalued, would be released the following day from the customs zone of the port of Manila and loaded on two
trucks, and upon orders of Ricardo Papa, Chief of Police of Manila and a duly deputized agent of the Bureau of Customs,
conducted surveillance at gate 1 of the customs zone. When the trucks left gate 1 at about 4:30 p.m. of 4 November
1966, elements of the counter-intelligence unit went after the trucks and intercepted them at the Agrifina Circle, Ermita,
Manila. The load of the two trucks, consisting of nine bales of goods, and the two trucks, were seized on instructions of
the Chief of Police. Upon investigation, a person claimed ownership of the goods and showed to the policemen a
"Statement and Receipts of Duties Collected on Informal Entry No. 147-5501", issued by the Bureau of Customs in the
name of a certain Bienvenido Naguit. Claiming to have been prejudiced by the seizure and detention of the two trucks
and their cargo, Remedios Mago and Valentin B. Lanopa filed with the Court of First Instance (CFI) of Manila a petition
"for mandamus with restraining order or preliminary injunction (Civil Case 67496), praying for the issuance of a
restraining order, ex parte, enjoining the police and customs authorities, or their agents, from opening the bales and
examining the goods, and a writ of mandamus for the return of the goods and the trucks, as well as a judgment for
actual, moral and exemplary damages in their favor. On 10 November 1966, Judge Hilarion Jarencio issued an order ex
parte restraining Ricardo Papa (as Chief of Police of Manila) and Juan Ponce Enrile (as Commissioner of Customs) in Civil
Case 67496. However, when the restraining order was received by Papa. et. al., some bales had already been opened by
the examiners of the Bureau of Customs in the presence of officials of the Manila Police Department, an assistant city
fiscal and a representative of Remedios Mago. Under date of 15 November 1966, Mago filed an amended petition,
including as party defendants Collector of Customs Pedro Pacis of the Port of Manila and Lt. Martin Alagao of the Manila
Police Department. At the hearing on 9 December 1966, the lower court, with the conformity of the parties, ordered
that an inventory of the goods be made by its clerk of court in the presence of the representatives of the claimant of the
goods, the Bureau of Customs, and the Anti- Smuggling Center of the Manila Police Department. On 23 December 1966,
Mago filed an ex parte motion to release the goods, alleging that since the inventory of the goods seized did not show
any article of prohibited importation, the same should be released as per agreement of the parties upon her posting of
the appropriate bond that may be determined by the court. On 7 March 1967, the Judge issued an order releasing the
goods to Mago upon her filing of a bond in the amount of P40,000.00. On 13 March 1967, Papa, on his own behalf, filed
a motion for reconsideration of the order of the court releasing the goods under bond, upon the ground that the Manila
Police Department had been directed by the Collector of Customs of the Port of Manila to hold the goods pending
termination of the seizure proceedings. Without waiting for the court's action on the motion for reconsideration, and
alleging that they had no plain, speedy and adequate remedy in the ordinary course of law, Papa, et. al. filed the action
for prohibition and certiorari with preliminary injunction before the Supreme Court.

Held:
The Chief of the Manila Police Department, Ricardo G. Papa, having been deputized in writing by the
Commissioner of Customs, could, for the purposes of the enforcement of the customs and tariff laws, effect searches,
seizures, and arrests, and it was his duty to make seizure, among others, of any cargo, articles or other movable property
when the same may be subject to forfeiture or liable for any fine imposed under customs and tariff laws. He could
lawfully open and examine any box, trunk, envelope or other container wherever found when he had reasonable cause
to suspect the presence therein of dutiable articles introduced into the Philippines contrary to law; and likewise to stop,
search and examine any vehicle, beast or person reasonably suspected of holding or conveying such article as aforesaid.
It cannot be doubted, therefore, that Papa, Chief of Police of Manila, could lawfully effect the search and seizure of the
goods in question. The Tariff and Customs Code authorizes him to demand assistance of any police officer to effect said
search and seizure, and the latter has the legal duty to render said assistance. This was what happened precisely in the
case of Lt. Martin Alagao who, with his unit, made the search and seizure of the two trucks loaded with the nine bales of
goods in question at the Agrifina Circle. He was given authority by the Chief of Police to make the interception of the
cargo. Martin Alagao and his companion policemen had authority to effect the seizure without any search warrant
issued by a competent court. The Tariff and Customs Code does not require said warrant herein. The Code authorizes
persons having police authority under Section 2203 of the Tariff and Customs Code to enter, pass through or search any
land, inclosure, warehouse, store or building, not being a dwelling house; and also to inspect, search and examine any
vessel or aircraft and any trunk, package, box or envelope or any person on board, or stop and search and examine any
vehicle, beast or person suspected of holding or conveying any dutiable or prohibited article introduced into the
Philippines contrary to law, without mentioning the need of a search warrant in said cases. But in the search of a
dwelling house, the Code provides that said "dwelling house may be entered and searched only upon warrant issued by
a judge or justice of the peace." Except in the case of the search of a dwelling house, persons exercising police authority
under the customs law may effect search and seizure without a search warrant in the enforcement of customs laws.
Herein, Martin Alagao and his companion policemen did not have to make any search before they seized the two trucks
and their cargo. But even if there was a search, there is still authority to the effect that no search warrant would be
needed under the circumstances obtaining herein. The guaranty of freedom from unreasonable searches and seizures is
construed as recognizing a necessary difference between a search of a dwelling house or other structure in respect of
which a search warrant may readily be obtained and a search of a ship, motorboat, wagon, or automobile for
contraband goods, where it is not practicable to secure a warrant, because the vehicle can be quickly moved out of the
locality or jurisdiction in which the warrant must be sought. Having declared that the seizure by the members of the
Manila Police Department of the goods in question was in accordance with law and by that seizure the Bureau of
Customs had acquired jurisdiction over the goods for the purposes of the enforcement of the customs and tariff laws, to
the exclusion of the Court of First Instance of Manila.
People v. Musa
GR 96177, 27 January 1993

Facts:
On 13 December 1989, the Narcotics Command (NARCOM) in Zamboanga City conducted surveillance and test buy on a
certain Mari Musa of Suterville, Zamboanga City. Information received from civilian informer was that this Mari Musa
was engaged in selling marijuana in said place. The Narcom agent (Sgt. Ani) was able to buy one newspaper-wrapped
dried marijuana for P10.00, which was turned over to the
Narcom office. The next day, a buy-bust was planned with Sgt. Ani being the poseur-buyer. NARCOM teams proceeded
to the target site in 2 civilian vehicles. Ani gave Musa the P20.00 marked money. Musa returned to his house and gave
Ani 2 newspaper wrappers containing dried marijuana. The signal to apprehend Musa was given. The NARCOM team
rushed to the location of Ani, and a NARCOM officer (Sgt. Belarga) frisked Musa but did not find the marked money. The
money was given to Musa’s wife who was able to slip away. Later, Belarga found a plastic bag containing dried
marijuana inside it somewhere in the kitchen. Musa was placed under arrest and was brought to the NARCOM office.
One newspaper-wrapper marijuana and the plastic bag containing more marijuana was sent to the PC Crime Laboratory,
the test of which gave positive results for the presence of marijuana. On the other hand, Mari Musa alleged that the
NARCOM agents, dressed in civilian clothes, got inside his house without any search warrant, neither his permission to
enter the house. The NARCOM agents searched the house and allegedly found a red plastic bag whose contents, Mari
Musa said, he did not know. He also did not know if the plastic bag belonged to his brother, Faisal, who was living with
him, or his father, who was living in another house about ten arms-length away. Mari Musa was handcuffed and was
taken to the NARCOM office where he was joined by his wife. Musa claimed that he was subjected to torture when he
refused to sign the document containing details of the investigation. The next day, he was taken to the fiscal‘s office to
which he was allegedly made to answer to a single question: that if he owned the marijuana. He allegedly was not able
to tell the fiscal that he had been maltreated by the NARCOM agents because he was afraid he might be maltreated in
the fiscal's office. Mari Musa was brought to the City Jail. Still, an information against Musa was filed on 15 December
1989. Upon his arraignment on
11 January 1990, Musa pleaded not guilty. After trial and on 31 August 1990, the RTC Zamboanga City (Branch XII) found
him guilty of selling marijuana in violation of Article II, Section 4 of RA 6425. Musa appealed to the Supreme Court.

Issue:
Whether the contents of the red plastic bag found in the kitchen may be admitted as evidence as evidence acquired
incidental to a lawful arrest.

Held:
Warrantless search incidental to a lawful arrest authorizes the arresting officer to make a search upon the person of the
person arrested. An officer making an arrest may take from the person arrested and money or property found upon his
person which was used in the commission of the crime or was the fruit of the crime or which might furnish the prisoner
with the means of committing violence or of escaping, or which may be used as evidence in the trial of the cause. Hence,
in a buy-bust operation conducted to entrap a drug-pusher, the law enforcement agents may seize the marked money
found on the person of the pusher immediately after the arrest even without arrest and search warrants. The
warrantless search and seizure, as an incident to a suspect's lawful arrest, may extend beyond the person of the one
arrested to include the premises or surroundings under his immediate control. Objects in the "plain view" of an officer
who has the right to be in the position to have that view are subject to seizure and may be presented as evidence. When
the discovery of the evidence did not constitute a search, but where the officer merely saw what was placed before him
in full view, the warrantless seizure of the object was legal on the basis of the "plain view" doctrine and upheld the
admissibility of said evidence. The "plain view" doctrine, however, may not be used to launch unbridled searches and
indiscriminate seizures nor to extend a general exploratory search made solely to find evidence of defendant's guilt. The
"plain view" doctrine is usually applied where a police officer is not searching for evidence against the accused, but
nonetheless inadvertently comes across an incriminating object. What the 'plain view' cases have in common is that the
police officer in each of them had a prior justification for an intrusion in the course of which he came inadvertently
across a piece of evidence incriminating the accused. The doctrine serves to supplement the prior justification —
whether it be a warrant for another object, hot pursuit, search incident to lawful arrest, or some other legitimate reason
for being present unconnected with a search directed against the accused — and permits the warrantless seizure. Of
course, the extension of the original justification is legitimate only where it is immediately apparent to the police that
they have evidence before them; the 'plain view' doctrine may not be used to extend a general exploratory search from
one object to another until something incriminating at last emerges. The "plain view" doctrine neither justify the seizure
of the object where the incriminating nature of the object is not apparent from the "plain view" of the object. Thus, the
exclusion of the plastic bag containing marijuana does not, however, diminish, in any way, the damaging effect of the
other pieces of evidence presented by the prosecution to prove that the appellant sold marijuana, in violation of Article
II, Section 4 of the Dangerous Drugs Act of 1972. By virtue of the testimonies of Sgt. Ani and T/Sgt. Belarga and the two
wrappings of marijuana sold by Musa to Sgt. Ani, among other pieces of evidence, the guilt of Musa of the crime charged
has been proved beyond reasonable doubt.
Valmonte vs. de Villa
GR 83988, 24 May 1990

Facts:
On 20 January 1987, the National Capital Region District Command (NCRDC) was activated pursuant to Letter of
Instruction 02/87 of the Philippine General Headquarters, AFP, with the mission of conducting
security operations within its area of responsibility and peripheral areas, for the purpose of establishing an effective
territorial defense, maintaining peace and order, and providing an atmosphere conducive to the social, economic and
political development of the National Capital Region. 1 As part of its duty to maintain peace and order, the NCRDC
installed checkpoints in various parts of Valenzuela, Metro Manila. Ricardo C. Valmonte and the Union of Lawyers and
Advocates for People's Right (ULAP) filed a petition for prohibition with preliminary injunction and/or temporary
restraining order witht the Supreme Court, seeking the declaration of checkpoints in Valenzuela, Metro Manila or
elsewhere, as unconstitutional and the dismantling and banning of the same or, in the alternative, to direct the
respondents to formulate guidelines in the implementation of checkpoints, for the protection of the people. They aver
that, because of the installation of said checkpoints, the residents of Valenzuela are worried of being harassed and of
their safety being placed at the arbitrary, capricious and whimsical disposition of the military manning the checkpoints,
considering that their cars and vehicles are being subjected to regular searches and check-ups, especially at night or at
dawn, without the benefit of a search warrant and/or court order. Their alleged fear for their safety increased when, at
dawn of 9 July 1988, Benjamin Parpon, a supply officer of the Municipality of Valenzuela, Bulacan, was gunned down
allegedly in cold blood by the members of the NCRDC manning the checkpoint along McArthur Highway at Malinta,
Valenzuela, for ignoring and/or refusing to submit himself to the checkpoint and for continuing to speed off inspire of
warning shots fired in the air. Valmonte also claims that, on several occasions, he had gone thru these checkpoints
where he was stopped and his car subjected to search/check-up without a court order or search warrant. They further
contend that the said checkpoints give Gen. Renato de Villa and the National Capital Region District Command a blanket
authority to make searches and/or seizures without search warrant or court order in violation of the Constitution. In the
Supreme Court's decision dated 29 September 1989, Valmonte’s and ULAP’s petition for prohibition, seeking the
declaration of the checkpoints as unconstitutional and their dismantling and/or banning, was dismissed. Valmonte and
ULAP filed the motion and supplemental motion for reconsideration of said decision.

Issue:
Whether checkpoints serve as a blanket authority for government officials for warrantless search and seizure and, thus,
are violative of the Constitution.

Held:
Nowhere in the Supreme Court's decision of 24 May 1990 did the Court legalize all checkpoints, i.e. at all times and
under all circumstances. What the Court declared is, that checkpoints are not illegal per se. Thus, under exceptional
circumstances, as where the survival of organized government is on the balance, or where the lives and safety of the
people are in grave peril, checkpoints may be allowed and installed by the government. Implicit in this proposition is,
that when the situation clears and such grave perils are removed, checkpoints will have absolutely no reason to remain.
Recent and on-going events have pointed to the continuing validity and need for checkpoints manned by either military
or police forces. Although no one can be compelled, under our libertarian system, to share with the present
government its ideological beliefs and practices, or commend its political, social and economic policies or performance;
one must concede to it the basic right to defend itself from its enemies and, while in power, to pursue its program of
government intended for public welfare; and in the pursuit of those objectives, the government has the equal right,
under its police power, to select the reasonable means and methods for best achieving them. The checkpoint is
evidently one of such means it has selected. Admittedly, the routine checkpoint stop does intrude, to a certain extent,
on motorist's right to "free passage without interruption", but it cannot be denied that, as a rule, it involves only a brief
detention of travellers during which the vehicle's occupants are required to answer a brief question or two. For as long
as the vehicle is neither searched nor its occupants subjected to a body search, and the inspection of the vehicle is
limited to a visual search, said routine checks cannot be regarded as violative of an individual's right against
unreasonable search. These routine checks, when conducted in a fixed area, are even less intrusive. Further, vehicles are
generally allowed to pass these checkpoints after a routine inspection and a few questions. If vehicles are stopped and
extensively searched, it is because of some probable cause which justifies a reasonable belief of the men at the
checkpoints that either the motorist is a law-offender or the contents of the vehicle are or have been instruments of
some offense. By the same token, a warrantless search of incoming and outgoing passengers, at the arrival and
departure areas of an international airport, is a practice not constitutionally objectionable because it is founded on
public interest, safety, and necessity. Lastly, the Court's decision on checkpoints does not, in any way, validate nor
condone abuses committed by the military manning the checkpoints. The Court's decision was concerned with power,
i.e. whether the government employing the military has the power to install said checkpoints. Once that power is
acknowledged, the Court's inquiry ceases. True, power implies the possibility of its abuse. But whether there is abuse in
a particular situation is a different "ball game" to be resolved in the constitutional arena. In any situation, where abuse
marks the operation of a checkpoint, the citizen is not helpless. For the military is not above but subject to the law. And
the courts exist to see that the law is supreme. Soldiers, including those who man checkpoints, who abuse their
authority act beyond the scope of their authority and are, therefore, liable criminally and civilly for their abusive acts.
ABRAHAM MICLAT, JR. v. PEOPLE
G.R. No. 176077

Facts:
Caloocan City Police Station-SDEU received an a information about an illicit and down-right drug-trading activities being
undertaken along Palmera Spring II, Bagumbong, Caloocan City involving Abe Miclat, Wily alias "Bokbok" and one Mic or
Jojo. Immediately, a surveillance team was formed. Thru a small opening in the curtain-covered window, PO3 Antonio
peeped inside and he saw "Abe" arranging several pieces of small plastic sachets containing shabu. Slowly, said
operative inched his way in by gently pushing the door as well as the plywood covering the same. Upon gaining
entrance, PO3 Antonio forthwith introduced himself as a police officer while "Abe," on the other hand, after being
informed of such authority, voluntarily handed over to the former the four (4) pieces of small plastic sachets the latter
was earlier sorting out. PO3 Antonio immediately placed the suspect under arrest and brought him and the four (4)
pieces of plastic sachets of shabu to their headquarters. The suspect was identified as Abraham Miclat y Cerbo a.k.a
"ABE," 19 years old. Thus, an information was filed against the accused, and upon arraignment, he pleaded not guilty.
The RTC convicted the accused with the crime of illegal possession of drugs which the CA affirmed in toto upon appeal.
Hence, this petition where petitioner argues that being seen in the act of arranging several plastic sachets inside their
house by one of
the arresting officers who was peeping through a window is not sufficient reason for the police authorities to enter his
house without a valid search warrant and/or warrant of arrest. He also posits that peeping through a curtain-covered
window cannot be contemplated as within the meaning of the plain view doctrine, rendering the warrantless arrest
unlawful.

Issue:
Whether or not peeping through a curtain is within the meaning of plain view doctrine as to justify the arrest of the
accused and the seizing of the sachets.

Held:
Yes. Verily, no less than the 1987 Constitution mandates that a search and consequent seizure must be carried out with
a judicial warrant; otherwise, it becomes unreasonable, and any evidence obtained therefrom shall be inadmissible for
any purpose in any proceeding. The right against warrantless searches and seizure, however, is subject to legal and
judicial exceptions, namely: 1.) Warrantless search incidental to a lawful arrest; 2.) Search of evidence in "plain view"; 3.)
Search of a moving vehicle; 4.) Consented warrantless search; 5.) Customs search; 6.) Stop and Frisk; and 7.) Exigent and
emergency circumstances.
It is to be noted that petitioner was caught in the act of arranging the heat-sealed plastic sachets in plain sight of PO3
Antonio and he voluntarily surrendered them to him upon learning that he is a police officer. The seizure made by PO3
Antonio of the four plastic sachets from the petitioner was not only incidental to a lawful arrest, but it also falls within
the purview of the "plain view" doctrine.
Objects falling in plain view of an officer who has a right to be in a position to have that view are subject to seizure even
without a search warrant and may be introduced in evidence. The "plain view" doctrine applies when the following
requisites concur: (a) the law enforcement officer in search of the evidence has a prior justification for an intrusion or is
in a position from which he can view a particular area; (b) the discovery of evidence in plain view is inadvertent; (c) it is
immediately apparent to the officer that the item he observes may be evidence of a crime, contraband or otherwise
subject to seizure. The law enforcement officer must lawfully make an initial intrusion or properly be in a position from
which he can particularly view the area. In the course of such lawful intrusion, he came inadvertently across a piece of
evidence incriminating the accused. The object must be open to eye and hand and its discovery
inadvertent.
It is clear, therefore, that an object is in plain view if the object itself is plainly exposed to sight. Since petitioner’s arrest
is among the exceptions to the rule requiring a warrant before effecting an arrest and the evidence seized from the
petitioner was the result of a warrantless search incidental to a lawful arrest, which incidentally was in plain view of the
arresting officer, the results of the ensuing search and seizure were admissible in evidence to prove petitioner’s guilt of
the offense charged.
SECTION 3 PRIVACY OF COMMUNICATION AND CORRESPONDENCE

Ramirez vs. Court of Appeals


GR 93833, 28 September 1995

Facts:
A civil case for damages was filed by Socorro D. Ramirez in the Regional Trial Court of Quezon City
alleging that Ester S. Garcia, in a confrontation in the latter's office, allegedly vexed, insulted and humiliated her in a
"hostile and furious mood" and in a manner offensive to petitioner's dignity and personality," contrary to morals, good
customs and public policy." In support of her claim, Ramirez produced a verbatim transcript of the event and sought
moral damages, attorney's fees and other expenses of litigation in the amount of P610,000.00, in addition to costs,
interests and other reliefs awardable at the trial court's discretion. The transcript on which the civil case was based was
culled from a tape recording of the confrontation made by Ramirez. As a result of Ramirez's recording, of the event and
alleging that the said act of secretly taping the confrontation was illegal, Garcia filed a criminal case before Regional Trial
Court of Pasay City for violation of Republic Act 4200, entitled "An Act to prohibit and penalize wire tapping and other
related violations of private communication, and other purposes." Ramirez was charged of violation of the said Act, in an
information dated 6 October 1988. Upon arraignment, in lieu of a plea, Ramirez filed a Motion to Quash the Information
on the ground that the facts charged do not constitute an offense, particularly a violation of RA 4200. In an order dated
3 May 1989, the trial court granted the Motion to Quash, agreeing with Ramirez that the facts charged do not constitute
an offense under RA 4200; and that the violation punished by RA 4200 refers to a the taping of a communication by a
person other than a participant to the communication. From the trial court's Order, Garcia filed a Petition for Review on
Certiorari with the Supreme Court, which forthwith Constitutional Law II, 2005 referred the case to the Court of Appeals
in a Resolution (by the First Division) of 19 June 1989. On 9 February 1990, the Court of Appeals promulgated its assailed
Decision declaring the trial court's order of 3 May 1989 null and void. Consequently, on 21 February 1990, Ramirez filed
a Motion for Reconsideration which Court of Appeals denied in its Resolution dated 19 June 1990. Hence, the petition.

Issue:
Whether the party sought to be penalized by the Anti-wire tapping law ought to be a party other than or different from
those involved in the private communication

Held:
Section 1 of RA 4200 provides that "It shall be unlawful for any person, not being authorized by all the
parties to any private communication or spoken word, to tap any wire or cable, or by using, any other device or
arrangement, to secretly overhear, intercept, or record such communication or spoken word by using a device
commonly known as a dictaphone or dictagraph or detectaphone or walkie-talkie or tape recorder, or however
otherwise described." The provision clearly and unequivocally makes it illegal for any person, not authorized by all the
parties to any private communication to secretly record such communication by means of a tape recorder. The law
makes no distinction as to whether the party sought to be penalized by the statute ought to be a party other than or
different from those involved in the private communication. The statute's intent to penalize all persons unauthorized to
make such recording is underscored by the use of the qualifier "any". Consequently, "even a (person) privy to a
communication who records his private conversation with another without the knowledge of the latter (will) qualify as a
violator" under said provision of RA 4200. Further, the nature of the conversation is immaterial to a violation of the
statute. The substance of the same need not be specifically alleged in the information. What RA 4200 penalizes are the
acts of secretly overhearing, intercepting or recording private communications by means of the devices enumerated
therein. The mere allegation that an individual made a secret recording of a private communication by means of a tape
recorder would suffice to constitute an offense under Section 1 of RA 4200. Furthermore, the contention that the phrase
"private communication" in Section 1 of RA 4200 does not include "private conversations" narrows the ordinary
meaning of the word "communication" to a point of absurdity.
Zulueta vs. Court of Appeals
GR 107383, 20 February 1996

Facts:
Cecilia Zulueta is the wife of Dr. Alfredo Martin. On 26 March 1982, Zulueta entered the clinic of her husband, a doctor
of medicine, and in the presence of her mother, a driver and Martin's secretary, forcibly opened the drawers and
cabinet in her husband's clinic and took 157 documents consisting of private correspondence between Dr. Martin and
his alleged paramours, greetings cards, cancelled checks, diaries, Dr. Martin's passport, and photographs. The
documents and papers were seized for use in evidence in a case for legal separation and for disqualification from the
practice of medicine which Zulueta had filed against her husband. Dr. Martin brought the action for recovery of the
documents and papers and for damages against Zulueta, with the Regional Trial Court of Manila, Branch X. After trial,
the trial court rendered judgment for Martin, declaring him the capital/exclusive owner of the properties described in
paragraph 3 of Martin's Complaint or those further described in the Motion to Return and Suppress and ordering
Zulueta and any person acting in her behalf to a immediately return the properties to Dr. Martin and to pay him
P5,000.00, as nominal damages; P5,000.00, as moral damages and attorney's fees; and to pay the costs of the suit. On
appeal, the Court of Appeals affirmed the decision of the Regional Trial Court. Zulueta filed the petition for review with
the Supreme Court.

Issue:
Whether the injunction declaring the privacy of communication and correspondence to be inviolable apply even to the
spouse of the aggrieved party.

Held:
The documents and papers are inadmissible in evidence. The constitutional injunction declaring "the privacy of
communication and correspondence [to be] inviolable" is no less applicable simply because it is the wife (who thinks
herself aggrieved by her husband's infidelity) who is the party against whom the constitutional provision is to be
enforced. The only exception to the prohibition in the Constitution is if there is a "lawful order [from a] court or when
public safety or order requires otherwise, as prescribed by law." Any violation of this provision renders the evidence
obtained inadmissible "for any purpose in any proceeding." The intimacies between husband and wife do not justify any
one of them in breaking the drawers and cabinets of the other and in ransacking them for any telltale evidence of
marital infidelity. A person, by contracting marriage, does not shed his/her integrity or his right to privacy as an
individual and the constitutional protection is ever available to him or to her. The law insures absolute freedom of
communication between the spouses by making it privileged. Neither husband nor wife may testify for or against the
other without the consent of the affected spouse while the marriage subsists. Neither may be examined without the
consent of the other as to any communication received in confidence by one from the other during the marriage, save
for specified exceptions. But one thing is freedom of communication; quite another is a compulsion for each one to
share what one knows with the other. And this has nothing to do with the duty of fidelity that each owes to the other.
NAVARRO vs CA
G.R. No. 121087, August 26, 1999

Facts:
Navarro was charged with homicide with the RTC. The trial court convicted him of the crime charged. The court
admitted in evidence the recorded tape allegedly containing the heated exchange between Navarro and the deceased
Lingan in the police station. The exchange in the voice recording was confirmed by the testimony of Jalbuena, one who
took the recording and witness for the prosecution.

Issue:
WON the tape is admissible in evidence under RA No. 4200.

Ruling:
Yes, the tape is admissible in evidence. RA No. 4200 prohibits the overhearing, intercepting, or recording of private
communications. Since the exchange between petitioner Navarro and Lingan was not private, its tape recording is not
prohibited. Nor is there any question that it was duly authenticated. A voice recording is authenticated by the testimony
of a witness (1) that he personally recorded the conversations; (2) that the tape played in the court was the one he
recorded; and (3) that the voices on the tape are those of the persons such are claimed to belong. In the instant case,
Jalbuena testified that he personally made the voice recording; that the tape played in the court was the one he
recorded; and that the speakers on the tape were petitioner Navarro and Lingan. A sufficient foundation was thus laid
for the authentication of the tape presented by the prosecution. The voice recording made by Jalbuena established: (1)
that there was a heated exchange between petitioner Navarro and Lingan on the placing in the police blotter of an entry
against him and Jalbuena; and (2) that some form of violence occurred involving petitioner Navarro and Lingan, with the
latter getting the worst of it.
Ople v. Torres
GR 127685, 23 July 1998

Facts:
On 12 December 1996, President Fidel V. Ramos issued Administrative Order 308, entitled "Adoption
of a National Computerized Identification Reference System." It was published in 4 newspapers of general circulation on
22 and 23 January 1997. On 24 January 1997, Senator Blas F. Ople, as a Senator, taxpayer and member of the
Government Service Insurance System (GSIS), filed instant petition against then Executive Secretary Ruben Torres and
the heads of the government agencies, who as members of the Inter-Agency Coordinating Committee are charged with
the implementation of Administrative Order 308.

Issue:
Whether the Philippine President can issue an Administrative Order for the adoption of a National Computerized
Identification Reference System, independent of a legislative act.

Held:
Administrative Order 308 establishes a system of identification that is all-encompassing in scope, affects the life and
liberty of every Filipino citizen and foreign resident, and more particularly, violates their right to privacy. Such a system
requires a delicate adjustment of various contending state policies: the primacy of national security, the extent of
privacy interest against dossier-gathering by government, the choice of policies, etc. As said administrative order
redefines the parameters of some basic rights of our citizenry vis-a-vis the State as well as the line that separates the
administrative power of the President to make rules and the legislative power of Congress, it ought to be evident that it
deals with a subject that should be covered by law. The Order is a law, negating claims that it confers no right, imposes
no duty, affords no protection, and creates no office. Under it, a citizen cannot transact business with government
agencies delivering basic services to the people without the contemplated identification card. No citizen will refuse to
get this identification card for no one can avoid dealing with government. It is thus clear that without the ID, a citizen
will have difficulty exercising his rights and enjoying his privileges. Administrative Order 308 does not merely
implements the Administrative Code of 1987, but establishes for the first time a National Computerized Identification
Reference System. An administrative order is an ordinance issued by the President which relates to specific aspects in
the administrative operation of government. It must be in harmony with the law and should be for the sole purpose of
implementing the law and carrying out the legislative policy. The authority to prescribe rules and regulations is not an
independent source of power to make laws. AO 308 was beyond the power of the President to issue.
Pollo vs David
GR 181881, 18 Oct. 2011

Facts:
On January 3, 2007, an unsigned letter-complaint addressed to respondent CSC Chairperson David was marked
“Confidential” and sent through a courier service from a certain “Alan San Pascual”. The letter-complaint indicated that
a certain attorney of CSC is lawyering people with pending cases in the Commission. Furthermore, the letter-complaint
said that the lawyer is from the “Mamamayan Muna Hindi Mamaya Na” division.
Chairperson David immediately formed a team of four personnel with background in information technology (IT), and
issued a memo directing them to conduct an investigation and specifically “to back up all the files in the computers
found in the Public Assistance and Liaison Division (PALD) under the Mamamayan Muna division.” Text messages were
also sent informing the employees of the ongoing copying of computer files.
The contents of the diskettes were examined by the CSC’s Office for Legal Affairs (OLA). It was found out that most of
the files in the 17 diskettes containing files copied from the computer assigned to and being used by Briccio “Ricky” A.
Pollo were draft pleadings or letters in connection with administrative cases in the CSC and other tribunals.
Pollo, filed his Comment, denying that he is the person referred to in the letter-complaint which had no attachments to
it, because he is not a lawyer and neither is he “lawyering” for people with cases in the CSC. He accused CSC officials of
conducting a “fishing expedition” when they unlawfully copied and printed personal files in his computer. He asserted
that the files in his computer were his personal files and those of his sister, relatives, friends and some associates and
that he is not authorizing their sealing, copying, duplicating and printing as these would violate his constitutional right to
privacy and protection against self-incrimination and warrantless search and seizure.
The CSC issued a Resolution finding prima facie case against Pollo and charged him with Dishonesty, Grave Misconduct,
Conduct Prejudicial to the Best Interest of the Service and Violation of R.A. No. 6713 (Code of Conduct and Ethical
Standards for Public Officials and Employees). He then filed a motion for reconsideration in the Court of Appeals but it
was denied.

Issue:
Was the search conducted on Pollo’s office computer and the copying of his personal files without his knowledge and
consent violate his constitutional right to privacy?

Ruling:
No. The right to privacy has been accorded recognition in this jurisdiction as a facet of the right protected by the
guarantee against unreasonable search and seizure under Section 2, Article III of the 1987 Constitution. However, the
constitutional guarantee is not a prohibition of all searches and seizures but only of “unreasonable” searches and
seizures.
First, if a person has a reasonable expectation of privacy, he can be protected of “search and seizure”. Justice Harlan of
the US Supreme Court noted that the existence of privacy right under prior decisions involved a two-fold requirement:
first, that a person has exhibited an actual (subjective) expectation of privacy; and second, that the expectation be one
that society is prepared to recognize as reasonable (objective). In the case at bar, petitioner failed to prove that he had
an actual (subjective) expectation of privacy either in his office or government-issued computer which contained his
personal files. Petitioner did not allege that he had a separate enclosed office which he did not share with anyone, or
that his office was always locked and not open to other employees or visitors. He described his office as “full of people,
his friends, unknown people” and that in the past 22 years he had been discharging his functions at the PALD, he is
“personally assisting incoming clients, receiving documents, and drafting cases on appeals”. The CSC also had a policy
regulating the use of office computers.
Second, a search by a government employer of an employee’s office is justified at inception when there are reasonable
grounds for suspecting that it will turn up evidence that the employee is guilty of work-related misconduct. If, indeed, a
CSC employee was found to be furtively engaged in the practice of “lawyering” for parties with pending cases before the
Commission, then such scenario would undeniably cast clouds of doubt upon the institutional integrity of the
Commission as a quasi-judicial agency, and in the process, render it less effective in fulfilling its mandate as an impartial
and objective dispenser of administrative justice.” The fact that these documents were retrieved from the computer of
Pollo raises the presumption that he was the author thereof. This is because he had a control of the said computer. The
ephemeral nature of computer files, that is, they could easily be destroyed at a click of a button, necessitated drastic
and immediate action.
Vivares vs St. Theresa’s College
GR 202666, 29 Sept. 2014

Facts:
Daluz, Suzara, among others, who were graduating high school students of St. Theresa’s College, took digital pictures of
themselves being covered only in their undergarments, drinking hard liquor, and smoking cigarettes. The photos were
shown to Escudero, a computer teacher at STC’s HS department. She reported the matter and the identified students
were then barred from joining their commencement exercises. Petitioners filed before the RTC a Petition for the
Issuance of a Writ of Habeas Data. The RTC dismissed the petition for habeas data. Hence, this certiorari.

Issue:
Was there an actual or threatened violation of the right to privacy in the life, liberty, or security of the minors that will
entitle them to the writ of habeas data? Up to what extent is the right protected in Facebook and other social
networking sites? Did STC violate the right to privacy of the minors?

Ruling:
No. The writ of habeas data is a remedy available to any person whose right to privacy in life, liberty or security is
violated or threatened by an unlawful act or omission of a public official or employee, or of a private individual or entity
engaged in the gathering, collecting or storing of data or information regarding the person, family, home and
correspondence of the aggrieved party. Using Facebook’s privacy tools, users can choose as to when and to what extent
to disclose facts about themselves – and to put others in the position of receiving such confidences. The utilization of
these privacy tools is the manifestation, in cyber world, of the user’s invocation of his or her right to informational
privacy.
In the case at bar, the petitioners’ children’s Facebook accounts, allegedly, were under “Only Friends.” However, the
Court said that setting the privacy to “Friends” is no assurance since a user’s own Facebook friend can share said content
or tag his or her own Facebook friend thereto, resulting to a greater number of users who can view the content. In
addition, respondent STC got the information from persons who had legitimate access to the said posts. Clearly, STC did
not violate petitioners’ daughters’ right to privacy. Hence, the petition for habeas data is denied.
SECTION 4 FREEDOM OF EXPRESSION

United States vs. Bustos


GR L-12592, 8 March 1918

Facts:
In the latter part of 1915, numerous citizens of the Province of Pampanga assembled, the prepared and signed a petition
to the Executive Secretary through the law office of Crossfield & O'Brien, and 5 individuals signed affidavits, charging
Roman Punsalan, justice of the peace of Macabebe and Masantol, Pampanga, with malfeasance in office and asking for
his removal. Crossfield & O'Brien submitted this petition and these affidavits with a complaint to the Executive
Secretary. The petition transmitted by these attorneys was signed by 34 citizens. The Executive Secretary referred the
papers to the judge of first instance for the Seventh Judicial District requesting investigation, proper action and report.
The Honorable Percy M. Moir, recommended to the Governor-General that Punzalan be removed from his position as
justice of the peace of Macabebe and Masantol, Province of Pampanga, and ordered that the proceedings had in the
case be transmitted to the Executive Secretary. Later the justice of the peace filed a motion for a new trial; the judge of
first instance granted the motion and reopened the hearing; documents were introduced, including a letter sent by the
municipal president and is councilors of Masantol, Pampanga, asserting that the justice of the peace was the victim of
prosecution, and that one Agustin Jaime, the auxiliary justice of the peace, had instituted the charges for personal
reasons; and the judge of first instance ordered a suppression of the charges against Punsalan and acquitted him of the
same. Attorneys for complainants thereupon appealed to the Governor-General. On 12 October 1916, Felipe Bustos, et.
al. (the petitioners against Punzalan) were charged for libel. The Honorable Percy M. Moir found all the defendants, with
the exception of Felix Fernandez, Juan S. Alfonso, Restituto Garcia, and Manuel Mallari, guilty and sentenced each of
them to pay a fine of P10 and 1/32 of the costs, or to suffer subsidiary imprisonment in case of insolvency. New
attorneys for the defense, coming into the case, after the handing down of the decision, filed on 16 December 1916, a
motion for a new trial, the principal purpose of which was to retire the objection interposed by then counsel for the
defendants
to the admission of the document consisting of the entire administrative proceedings. The trial court denied the motion.
All the defendants, except Melecio S. Sabado and Fortunato Macalino appealed.

Issue:
Whether the intemperate allegations set forth in the information against the public official may be the basis of a libel
case against the petitioning citizens.

Held:
"No law shall be passed abridging the freedom of speech or of the press or of the rights of the people to peaceably
assemble and petition the Government for a redress of grievances." These paragraphs found in the Philippine Bill of
Rights are not threadbare verbiage. The language carries with it all the applicable jurisprudence of great English and
American Constitutional cases. The interest of society and the maintenance of good government demand a full
discussion of public affairs. Complete liberty to comment on the conduct of public men is a scalpel in the case of free
speech. The sharp incision of its probe relieves the abscesses of officialdom. Men in public life may suffer under a hostile
and an unjust accusation; the wound can be assuaged with the balm of a clear conscience. A public officer must not be
too thin-skinned with reference to comment upon his official acts. Only thus can the intelligence and dignity of the
individual be exalted. Of course, criticism does not authorized defamation. Nevertheless, as the individual is less than
the State, so must expected criticism be born for the common good. Rising superior to any official, or set of officials, to
the Chief Executive, to the Legislature, to the Judiciary — to any or all the agencies of Government — public opinion
should be the constant source of liberty and democracy. The guaranties of a free speech and a free press include the
right to criticize judicial conduct. The administration of the law is a matter of vital public concern. Whether the law is
wisely or badly enforced is, therefore, a fit subject for proper comment. If the people cannot criticize a justice of the
peace or a judge the same as any other public officer, public opinion will be effectively muzzled. Attempted terrorization
of public opinion on the part of the judiciary would be tyranny of the basest sort. The sword of Damocles in the hands of
a judge does not hang suspended over the individual who dares to assert his prerogative as a citizen and to stand up
bravely before any official. On the contrary, it is a duty which every one owes to society or to the State to assist in the
investigation of any alleged misconduct. It is further the duty of all know of any official dereliction on the part of a
magistrate or the wrongful act of any public officer to bring the facts to the notice of those whose duty it is to inquire
into
and punish them. In the words of Mr. Justice Gayner, who contributed so largely to the law of libel. "The
people are not obliged to speak of the conduct of their officials in whispers or with bated breath in a free government,
but only in a despotism." The right to assemble and petition is the necessary consequence of republican institutions and
the complement of the right of free speech. Assembly means a right on the part of citizens to meet peaceably for
consultation in respect to public affairs. Petition means that any person or group of persons can apply, without fear of
penalty, to the appropriate branch or office of the government for a redress of grievances. The persons assembling and
petitioning must, of course, assume responsibility for the charges made. Public policy, the welfare of society, and the
orderly administration of government have demanded protection for public opinion. The inevitable and incontestable
result has been the development and adoption of the doctrine of privilege. Privilege is classified as either absolute or
qualified. With the first, we are not concerned. As to qualified privilege, it is as the words suggest a prima facie privilege
which may be lost by proof of malice. A pertinent illustration of the application of qualified privilege is a complaint made
in good faith and without malice in regard to the character or conduct of a public official when addressed to an officer or
a board having some interest or duty in the matter. Even when the statements are found to be false, if there is probable
cause for belief in their truthfulness and the charge is made in good faith, the mantle of privilege may still cover the
mistake of the individual. But the statements must be made under an honest sense of duty; a self-seeking motive is
destructive. Personal injury is not necessary. All persons have an interest in the pure and efficient administration of
justice and of public affairs. The duty under which a party is privileged is sufficient if it is social or moral in its nature and
this person in good faith believe he is acting in pursuance thereof although in fact he is mistaken. The privilege is not
defeated by the mere fact that the communication is made in intemperate terms. A further element of the law of
privilege concerns the person to whom the complaint should be made. The rule is that if a party applies to the wrong
person through some natural and honest mistake as to the respective functions of various officials such unintentional
error will not take the case out of the privilege. Hence, the Court find the defendants entitled to the protection of the
rules concerning qualified privilege, growing out of constitutional guaranties in our bill of rights.
People vs. Alarcon
GR 46551, 12 December 1939

Facts:
As an aftermath of the decision rendered by the Court of First Instance of Pampanga in criminal case 5733 (People s vs.
Salvador Alarcon, et al.), convicting the accused therein except one — of the crime of robbery committed in band, a
denunciatory letter, signed by one Luis M. Taruc, was addressed to His
Excellency, the President of the Philippines. A copy of said letter found its way to Federico Mangahas who, as columnist
of the Tribune, a newspaper of general circulation in the Philippines, quoted the letter in an article published by him in
the issue of that paper of 23 September 1937. The article provides, in part, that "Fifty-two (52) tenants in Floridablanca,
Pampanga, have been charged and convicted on a trumped up charge of robbery in band because they took each a few
cavans of palay for which they issued the corresponding receipts, from the bodega in the hacienda where they are
working. These tenants contend that they have the right to take the palay for their food as the hacienda owner has the
obligation to give them rations of palay for their main tenance and their families to be paid later with their share of their
crop. But this is not all. When the convicted tenants appealed the case and were released on bail pending their appeal,
court and public officials exerted pressure upon one of their bondsmen, as this bondsman informed the tenants, to
withdraw his bail for them, and the fifty two tenants were arrested again and put in jail." On 29 September 1937, the
provincial fiscal of Pampanga filed with the Court of First Instance of that province to cite Federico Mangahas for
contempt. On the same date, the lower court ordered Mangahas to appear and show cause. Mangahas appeared and
filed an answer,alleging, among others, that “the publication of the letter in question is in line with the constitutional
guarantee of freedom of the press.” On 29 November 1937, the lower court entered an order, imposing upon Mangahas
the nominal fine of P25, or in case of insolvency, 5 days in prison; this without prejudice to the action for libel that the
public prosecutor believes to be advisable to file against Luis M. Taruc. Mañgahas appealed from this order to the Court
of Appeals — which later certified the case to the Supreme Court as involving only a question of law.

Issue:
Whether the trial court properly cited Mangahas for contempt inasmuch as the robbery-in-band case is still pending
appeal.

Held:
Newspaper publications tending to impede, obstruct, embarrass, or influence the courts in administering justice in a
pending suit or proceeding constitutes criminal contempt which is summarily punishable by the courts. The rule is
otherwise after the cause is ended. It must, however, clearly appear that such publications do impede, interfere with,
and embarrass the administration of justice before the author of the publications should be held for contempt. What is
thus sought to be shielded against the influence of newspaper comments is the all-important duty of the court to
administer justice in the decision of a pending case. There is no pending case to speak of when and once the court has
come upon a decision and has lost control either to reconsider or amend it. That is the present case, for here the letter
complained of was published after the Court of First Instance of Pampanga had decided the criminal case for robbery in
band, and after that decision had been appealed to the Court of Appeals. The fact that a motion to reconsider its order
confiscating the bond of the accused therein was subsequently filed may be admitted; but, the important consideration
is that it was then without power to reopen or modify the decision which it had rendered upon the merits of the case,
and could not have been influenced by the questioned publication. If it be contended, however, that the publication of
the questioned letter constitutes contempt of the Court of Appeals where the appeal in the criminal case was then
pending, the interrelation of the different courts forming our integrated judicial system, one court is not an agent or
representative of another and may not, for this reason, punish contempts in vindication of the authority and de corum
which are not its own. The appeal transfers the proceedings to the appellate court, and this last court be comes thereby
charged with the authority to deal with
contempts committed after the perfection of the appeal.
Ayer Production Pty. Ltd. vs. Capulong
GR L-82380, 29 April 1988

Facts:
Hal McElroy, an Australian film maker, and his movie production company,Ayer Productions Pty. Ltd., envisioned,
sometime in 1987, the filming for commercial viewing and for Philippine and international release, the historic peaceful
struggle of the Filipinos at EDSA (Epifanio de los Santos Avenue). McEleroy discussed this project with local movie
producer Lope V. Juban, who advised that they consult with the appropriate government agencies and also with General
Fidel V. Ramos and Senator Juan Ponce Enrile, who had played major roles in the events proposed to be filmed. The
proposed motion picture entitled "The Four Day Revolution" was endorsed by the Movie Television Review and
Classification Board as well as the other government agencies consulted. General Fidel Ramos also signified his approval
of the intended film production. In a letter dated 16 December 1987, McElroy, informed Juan Ponce Enrile about the
projected motion picture enclosing a synopsis of it. On 21 December 1987, Enrile replied that "he would not and will not
approve of the use, appropriation, reproduction and/or exhibition of his name, or picture, or that of any member of his
family in any cinema or television production, film or other medium for advertising or commercial exploitation" and
further advised McElroy that "in the production, airing, showing, distribution or exhibition of said or similar film, no
reference whatsoever (whether written, verbal or visual) should not be made to him or any member of his family, much
less to any matter purely personal to them." It appears that McElroy acceded to this demand and the name of Enrile was
deleted from the movie script, and McElroy proceeded to film the projected motion picture. On 23 February 1988, Enrile
filed a Complaint with application for Temporary Restraining Order and Writ of Preliminary Injunction with the Regional
Trial Court of Makati (Civil Case 88-151; Branch 134), seeking to enjoin McElroy, et. al. from producing the movie "The
Four Day Revolution." The complaint alleged that McElroy, et. al.'s production of the mini-series without Enrile's consent
and over his objection, constitutes an obvious violation of his right of privacy. On 24 February 1988, the trial court issued
ex-parte a Temporary Restraining Order and set for hearing the application for preliminary injunction. On 9 March 1988,
McElroy filed a Motion to Dismiss with Opposition to the Petition for Preliminary Injunction contending that the mini-
series film would not involve the private life of Juan Ponce Enrile nor that of his family and that a preliminary injunction
would amount to a prior restraint on their right of free expression. Ayer Productions also filed its own Motion to Dismiss
alleging lack of cause of action as the mini-series had not yet been completed. In an Order dated 16 March 1988, the
trial court issued a writ of Preliminary Injunction against the McElroy, et. al. On 22 March 1988, Ayer Productions filed a
Petition for Certiorari dated 21 March 1988 with an urgent prayer for Preliminary Injunction or Restraining Order with
the Supreme Court (GR L-82380). A day later, or on 23 March 1988, McElroy also filed a separate Petition for Certiorari
with Urgent Prayer for a Restraining Order or Preliminary Injunction, dated 22 March 1988 (GR L-82398). By a Resolution
dated 24 March 1988, the petitions were consolidated.

Issue:
Whether depiction of Enrile, as part of the events in the 1986 People Power Revolution and not as to his personal life
nor his family, in the film “The Four Day Revolution” requires his prior consent.

Held:
The freedom of speech and of expression includes the freedom to film and produce motion pictures and to exhibit such
motion pictures in theaters or to diffuse them through television. In our day and age, motion pictures are a universally
utilized vehicle of communication and medium of expression. Along with the press, radio and television, motion pictures
constitute a principal medium of mass communication for information, education and entertainment. This freedom is
available in our country both to locally-owned and to foreign-owned motion picture companies. Furthermore, the
circumstance that the production of motion picture films is a commercial activity expected to yield monetary profit, is
not a disqualification for availing of freedom of speech and of expression. In our community, as in many other countries,
media facilities are owned either by the government or the private sector but the private sector-owned media facilities
commonly require to be sustained by being devoted in whole or in part to revenue producing activities. Indeed,
commercial media constitute the bulk of such facilities available in our country and hence to exclude commercially
owned and operated media from the exercise of constitutionally protected freedom of speech and of expression can
only result in the drastic contraction of such constitutional liberties in our country. The counter-balancing claim of Enrile
is to a right of privacy. Our law, constitutional and statutory, does include a right of privacy. It is left to case law,
however, to mark out the precise scope and content of this right in differing types of particular situations. The right of
privacy or "the right to be let alone," like the right of free expression, is not an absolute right. A limited intrusion into a
person's privacy has long been regarded as permissible where that person is a public figure and the information sought
to be elicited from him or to be published about him constitute matters of a public character. Succinctly put, the right of
privacy cannot be invoked to resist publication and dissemination of matters of public interest. The interest sought to be
protected by the right of privacy is the right to be free from "unwarranted publicity, from the wrongful publicizing of the
private affairs and activities of an individual which are outside the realm of legitimate public concern." Herein, there is a
prior and direct restraint on the part of the respondent Judge upon the exercise of speech and of expression by McElroy,
et. al. The Judge has restrained them from filming and producing the entire proposed motion picture. The Judge should
have stayed his hand, instead of issuing an ex-parte Temporary Restraining Order one day after filing of a complaint by
Enrile and issuing a Preliminary Injunction 20 days later; for the projected motion picture was as yet uncompleted and
hence not exhibited to any audience. Neither Enrile nor the trial Judge knew what the completed film would precisely
look like. There was, in other words, no "clear and present danger" of any violation of any right to privacy that Enrile
could lawfully assert. The subject matter of "The Four Day Revolution" relates to the non-bloody change of government
that took place at Epifanio de los Santos Avenue in February 1986, and the train of events which led up to that
denouement.
Clearly, such subject matter is one of public interest and concern, and also of international interest. The subject relates
to a highly critical stage in the history of this country and as such, must be regarded as having passed into the public
domain and as an appropriate subject for speech and expression and coverage by any form of mass media. The subject
matter does not relate to the individual life and certainly not to the private life of Ponce Enrile. "The Four Day
Revolution" is not principally about, nor is it focused upon, the man Juan Ponce Enrile; but it is compelled, if it is to be
historical, to refer to the role played by Juan Ponce Enrile in the precipitating and the constituent events of the change
of government in February 1986. The extent of the intrusion upon the life of Juan Ponce Enrile that would be entailed by
the production and exhibition of "The Four Day Revolution" would, therefore, be limited in character. The extent of that
intrusion may be generally described as such intrusion as is reasonably necessary to keep that film a truthful historical
account. Enrile does not claim that McElroy, et. al. threatened to depict in "The Four Day Revolution" any part of the
private life of Enrile or that of any member of his family. The line of equilibrium in the specific context of the present
case between the constitutional freedom of speech and of expression and the right of privacy, may be marked out in
terms of a requirement that the proposed motion picture must be fairly truthful and historical in its presentation of
events. There must, in other words, be no knowing or reckless disregard of truth in depicting the participation of private
respondent in the EDSA Revolution. There must, further, be no presentation of the private life of the unwilling individual
(Enrile) and certainly no revelation of intimate or embarrassing personal facts. The proposed motion picture should not
enter into a "matters of essentially private concern."
To the extent that "The Four Day Revolution" limits itself in portraying the participation of Enrile in the
EDSA Revolution to those events which are directly and reasonably related to the public facts of the EDSA Revolution,
the intrusion into Enrile's privacy cannot be regarded as unreasonable and actionable. Such portrayal may be carried out
even without a license from Enrile.
Reyes vs. Bagatsing
GR L-65366, 9 November 1983

Facts:
Retired Justice J.B.L. Reyes, on behalf of the Anti-Bases Coalition, sought a permit from the City of Manila to hold a
peaceful march and rally on 26 October 1983 from 2:00 to 5:00 p.m., starting from the Luneta, a public park, to the gates
of the United States Embassy, hardly two blocks away. Once there, and in an open space of public property, a short
program would be held. After the planned delivery of two brief speeches, a petition based on the resolution adopted on
the last day by the International Conference for General Disarmament, World Peace and the Removal of All Foreign
Military Bases held in Manila, would be presented to a representative of the Embassy or any of its personnel who may
be there so that it may be delivered to the United States Ambassador. The march would be attended by the local and
foreign participants of such conference. An assurance was made to observe all the necessary steps "to ensure a peaceful
march and rally." Since Reyes had not been informed of any action taken on his request on behalf of the organization to
hold a rally, on 20 October 1983, he filed a suit for mandamus with alternative prayer for writ of preliminary mandatory
injunction. The oral argument was heard on 25 October 1983, the very same day the answer was filed. The Court then
deliberated on the matter. That same afternoon, a minute resolution was issued by the Court granting the mandatory
injunction prayed for on the ground that there was no showing of the existence of a clear and present danger of a
substantive evil that could justify the denial of a permit. The last sentence of such minute resolution reads: "This
resolution is without prejudice to a more extended opinion." Hence the detailed exposition of the Court's stand on the
matter.

Issue:
Whether Reyes, et. al. can exercise their freedom of speech, press, or to assemble in front of the US embassy.

Held:
The Constitution is quite explicit: "No law shall be passed abridging the freedom of speech, or of the press, or the right
of the people peaceably to assemble and petition the Government for redress of grievances." Free speech, like free
press, may be identified with the liberty to discuss publicly and truthfully any matter of public concern without
censorship or punishment. There is to be then no previous restraint on the communication of views or subsequent
liability whether in libel suits, prosecution for sedition, or action for damages, or contempt proceedings unless there be
a "clear and present danger of a substantive evil that [the State] has a right to prevent." Freedom of assembly connotes
the right of the people to meet peaceably for consultation and discussion of matters of public concern. It is entitled to
be accorded the utmost deference and respect. It is not to be limited, much less denied, except on a showing, as is the
case with freedom of expression, of a clear and present danger of a substantive evil that the state has a right to prevent.
The sole justification for a limitation on the exercise of this right, so fundamental to the maintenance of democratic
institutions, is the danger, of a character both grave and imminent, of a serious evil to public safety, public morals, public
health, or any other legitimate public interest. There can be no legal objection, absent the existence of a clear and
present danger of a substantive evil, on the choice of Luneta as the place where the peace rally would start. Neither can
there be any valid objection to the use of the streets to the gates of the US Embassy, hardly two blocks away at the
Roxas Boulevard. The novel aspect of the case is that there would be a short program upon reaching the public space
between the two gates of the United States Embassy at Roxas Boulevard. Related to this, the second paragraph of its
Article 22 of the Vienna Convention on Diplomatic Relations (to which the Philippines is a signatory) reads: "2. The
receiving State is under a special duty to take appropriate steps to protect the premises of the mission against any
intrusion or damage and to prevent any disturbance of the peace of the mission or impairment of its dignity." That being
the case, if there were a clear and present danger of any intrusion or damage, or disturbance of the peace of the
mission, or impairment of its dignity, there would be a justification for the denial of the permit insofar as the terminal
point would be the Embassy. Moreover, Ordinance 7295 of the City of Manila prohibits the holding or staging of rallies
or demonstrations within a radius of 500 feet from any foreign mission or chancery; and for other purposes. Even then,
if the ordinance is nullified, or declared ultra vires, its invocation as a defense is understandable but not decisive, in view
of the primacy accorded the constitutional rights of free speech and peaceable assembly. There was no showing,
however, that the distance between the chancery and the embassy gate is less than 500 feet. Even if it could be shown
that such a condition is satisfied, it does not follow that the Mayor could legally act the way he did. The validity of his
denial of the permit sought could still be challenged. It could be argued that a case of unconstitutional application of
such ordinance to the exercise of the right of peaceable assembly presents itself. As in this case there was no proof that
the distance is less than 500 feet, the need to pass on that issue was obviated. The high estate accorded the rights to
free speech and peaceable assembly demands nothing less.
PITA VS. COURT OF APPEALS
178 SCRA 362; G.R. NO. 80806; 5 OCT 1989

Facts:

On December 1 and 3, 1983, pursuing an Anti-Smut Campaign initiated by the Mayor of the City of Manila, Ramon D.
Bagatsing, elements of the Special Anti-Narcotics Group, Auxilliary Services Bureau, Western Police District, INP of the
Metropolitan Police Force of Manila, seized and confiscated from dealers, distributors, newsstand owners and peddlers
along Manila sidewalks, magazines, publications and other reading materials believed to be obscene, pornographic and
indecent and later burned the seized materials in public at the University belt along C.M. Recto Avenue, Manila, in the
presence of Mayor Bagatsing and several officers and members of various student organizations.

Among the publications seized, and later burned, was "Pinoy Playboy" magazines published and co-edited by plaintiff
Leo Pita.

Plaintiff filed a case for injunction with prayer for issuance of the writ of preliminary injunction against Mayor Bagatsing
and Narcisco Cabrera, as superintendent of Western Police District of the City of Manila, seeking to enjoin said
defendants and their agents from confiscating plaintiff’s magazines or from preventing the sale or circulation thereof
claiming that the magazine is a decent, artistic and educational magazine which is not per se obscene, and that the
publication is protected by the Constitutional guarantees of freedom of speech and of the press. Plaintiff also filed an
Urgent Motion for issuance of a temporary restraining order against indiscriminate seizure, confiscation and burning of
plaintiff's "Pinoy Playboy" Magazines, pending hearing on the petition for preliminary injunction. The Court granted the
temporary restraining order. The case was set for trial upon the lapse of the TRO. RTC ruled that the seizure was valid.
This was affirmed by the CA.

Issue:
Whether or Not the seizure was illegal.

Held:
The Court ruled that the government authorities have not shown the required proof to justify a ban and to warrant
confiscation of the literature. First of all, they were not possessed of a lawful court order: (1) finding the said materials
to be pornography, and (2) authorizing them to carry out a search and seizure, by way of a search warrant. The court
provides the following guidelines to be observed:
1. The authorities must apply for the issuance of a search warrant from a judge, if in their opinion an obscenity seizure
is in order;
2. The authorities must convince the court that the materials sought to be seized are obscene and pose a clear and
present danger of an evil substantive enough to warrant State interference and action;
3. The judge must determine whether or not the same are indeed obscene. The question is to be resolved on a case-to-
case basis and on the judge’s sound discretion;
4. If in the opinion of the court, probable cause exists, it shall issue the search warrant prayed for;
5. The proper suit is then brought in the court under Article 201 of the RPC (Obscene publications).
6. Any conviction is subject to appeal. The appellate court may assess whether or not the properties seized are indeed
obscene.
ABS-CBN v. COMELEC
G.R. No. 133486, Jan. 28, 2000

Facts:
COMELEC released a resolution which approved the issuance of a restraining order against ABS-CBN to conduct exit
survey. It acted upon reports that the network plans to conduct a TV-radio coverage of the elections and make an exit
survey of the votes cast for Pres and VP and broadcast the results immediately.
SC issued a TRO against the resolution of COMELEC and the exit polls were actually conducted and reported by media.

Arguments:
ABS-CBN:
Holding exit polls and nationwide reporting of results are valid exercises of the freedoms of speech and of the press.
COMELEC committed GAD when it issued such resolution

COMELEC:
1. The conduct of exit polls might confuse the voters and unduly influence them.
2. Exit surveys indirectly violate the sanctity of ballots as enshrined in the Constitution (Sec. 2 Art. 5) because the voters
will be lured to reveal their votes.
3. Exit surveys pose a clear and present danger of destroying the credibility and integrity of the electoral process
because the media is not supervised by any gov’t agency which can easily be manipulated.

Issue:
WON COMELEC in the exercise of its powers can ban exit polls? NO. The measure is overbroad and unnecessarily
restricts fundamental rights of speech and of press.

Ratio
The Constitution mandates that no law shall be passed abridging freedom of speech and press. These freedoms basically
consist of the liberty to discuss publicly and truthfully any matter of public interest without prior restraint. (Gonzales v.
COMELEC) It represents a profound commitment to the principle that debates on public issues should be uninhibited,
robust and wide open.
There are limitations however to this freedom in which the state, in the exercise of its police power, can curtail
whenever these tests are satisfied:
1. Clear and present danger rule – the evil consequence of comment or utterance must be extremely serious and the
degree of imminence must be extremely high before the utterance can be punished. The danger to be guarded
against is the substantive evil sought to be prevented.
2. Dangerous tendency rule - If the words uttered create a dangerous tendency which the state has a right to prevent,
then such words are punishable. It is sufficient if the natural tendency and probable effect of the utterance be to
bring about the substantive evil which the legislative body seeks to prevent. Note that the words need not be
definite so as to incite the listeners to acts of force, violence or unlawfulness.
The court adheres to the clear and present danger rule which is a question on the proximity and degree of the utterance
will result to the danger or evil sought to be avoided. This is a heavy burden because the court is always on the side of
freedom of expression. To justify restriction, the promotion of substantial govt interest must be clearly shown. And even
when its purpose are legitimate and substantial, the means employed should not be broad as to stifle personal liberties
when the end can be more narrowly achieved.
In this case, the freedoms of speech and of the press should all the more be upheld when what is sought to be curtailed
is the dissemination of information meant to add meaning to the equally vital right of suffrage. The interest being
protected is the fundamental right to vote and securing its sanctity through the ballots. However the means employed
are necessarily broad because it effectively prevents other uses of exit poll data – for long term research purposes.
COMELEC tried to justify the restraint in arguing that such conduct of exit polls present a clear and present danger of
destroying the credibility and integrity of electoral process. (na unreliable and surveys and might conflict with the count
of COMELEC and NAMFREL) However its arguments are purely speculative. Why?
1. Because in a survey, the participants are randomly selected so the results will be a representation or reflection of
the general sentiment of the community.
2. It is merely an opinion of the community or group polled. Its result is not meant to replace or be at par with the
official COMELEC count.

COMELEC ‘s restriction on exit polls is overly broad. Its application is without qualification whether the exit polls is
disruptive or not. And assuming arguendo that there is such qualification, there is no showing that exit polls will cause
chaos in voting centers. The absolute prohibition restricts the future use of valuable information for long-term research
on the impact of current events on the voting behavior of people.
ABS-CBN even explained its methodology which has enough precautions against the evils enumerated by COMELEC:
1. communities are randomly selected in each province;
2. residences to be polled in such communities are also chosen at random;
3. only individuals who have already voted, as shown by the indelible ink on their fingers, are interviewed;
4. the interviewers use no cameras of any sort;
5. the poll results are released to the public only on the day after the elections

And lastly, on the issue of violation of ballot secrecy, the court said that such is not at issue here. The exit poll dies not
seek to access the ballots of the interviewees. The contents of their ballots are not exposed. Even the choice of revealing
who they voted for is not mandatory but voluntary.
The reason behind the principle of ballot secrecy is to avoid vote buying through voter identification. Thus, voters are
prohibited from exhibiting the contents of their official ballots to other persons, from making copies thereof, or from
putting distinguishing marks thereon so as to be identified. Also proscribed is finding out the contents of the ballots cast
by particular voters or disclosing those of disabled or illiterate voters who have been assisted. Clearly, what is forbidden
is the association of voters with their respective votes, for the purpose of assuring that the votes have been cast in
accordance with the instructions of a third party. This result cannot, however, be achieved merely through the voters'
verbal and confidential disclosure to a pollster of whom they have voted for.
Social Weather Stations Inc. vs. Commission on Elections
GR 147571, 5 May 2001

Facts:
The Social Weather Stations, Inc. (SWS), is a private non-stock, non-profit social research institution conducting surveys
in various fields, including economics, politics, demography, and social development, and thereafter processing,
analyzing, and publicly reporting the results thereof. On the other hand, Kamahalan Publishing Corporation publishes
the Manila Standard, a newspaper of general circulation, which features news-worthy items of information including
election surveys. SWS and Kamahalan Publishing brought the action for prohibition with the Supreme Court to enjoin
the Commission on Elections from enforcing §5.4 of RA 9006 (Fair Election Act), which provides that "Surveys affecting
national candidates shall not be published fifteen (15) days before an election and surveys affecting local candidates
shall not be published seven (7) days be- fore an election." SWS states that it wishes to conduct an election survey
throughout the period of the elections both at the national and local levels and release to the media the results of such
survey as well as publish them directly. Kamahalan Publishing, on the other hand, states that it intends to publish
election survey results up to the last day of the elections on 14 May 2001. They argue that the restriction on the
publication of election survey results constitutes a prior restraint on the exercise of freedom of speech without any clear
and present danger to justify such restraint. They claim that SWS and other pollsters conducted and published the
results of surveys prior to the 1992, 1995, and 1998 elections up to as close as two days before the election day without
causing confusion among the voters and that there is neither empirical nor historical evidence to support the conclusion
that there is an immediate and inevitable danger to tile voting process posed by election surveys. They point out that no
similar restriction is imposed on politicians from explaining their opinion or on newspapers or broadcast media from
writing and publishing articles concerning political issues up to the day of the election. Consequently, they contend that
there is no reason for ordinary voters to be denied access to the results of election surveys, which are relatively
objective.

Issue:
Whether §5.4 of RA 9006 constitutes an unconstitutional abridgment of freedom of speech, expression, and the press.

Held:
§5.4 of RA 9006 constitutes an unconstitutional abridgment of freedom of speech, expression, and the press. §5.4 lays a
prior restraint on freedom of speech, expression, and the press prohibiting the publication of election survey results
affecting candidates within the prescribed periods of 15 days immediately preceding a national election and 7 days
before a local election. Because of the preferred status of the constitutional rights of speech, expression, and the press,
such a measure is vitiated by a weighty presumption of invalidity. Indeed, any system of prior restraints of expression
comes to the Supreme Court bearing a heavy presumption against its constitutional validity. The Government thus
carries a heavy burden of showing justification for in enforcement of such restraint. There, thus a reversal of the normal
presumption of validity that inheres in every legislation. Sec. 5.4 fails to meet criterion [3] of the O 'Brien test because
the causal connection of expression to the asserted governmental interest makes such interest "not related to the
suppression of free expression." By prohibiting the publication of election survey results because of the possibility that
such publication might undermine the integrity of the election, §5.4 actually suppresses a whole class of expression,
while allowing the expression of opinion concerning the same subject matter by newspaper columnists, radio and TV
commentators, armchair theorists, and other opinion takers. In effect, §5.4 shows a bias for a particular subject matter,
if not viewpoint, by referring personal opinion to statistical results. The constitutional guarantee of freedom of
expression means that "the government has no power to restrict expression because of its message, its ideas, its subject
matter, or its content." The prohibition imposed by §5.4 cannot be justified on the ground that it is only for a limited
period and is only incidental. The prohibition may be for a limited time, but the curtailment of the right of expression is
direct, absolute, and substantial. It constitutes a total suppression of a category of speech and is not made less so
because it is only for a period of 15 days immediately before a national election and 7 days immediately before a local
election. In fine, §5.4 is invalid because
(1) it imposes a prior restraint on the freedom of expression,
(2) it is a direct and total suppression of a category of expression even though such suppression is only for a
limited period, and
(3) the governmental interest sought to be promoted can be achieved by means other than suppression of
freedom of expression.
Francisco Chavez vs. Raul M. Gonzales and NTC
G.R. No. 168338 | February 15, 2008

Facts:
As a consequence of the public release of copies of the “Hello Garci” compact disc audiotapes involving a wiretapped
mobile phone conversation between then-President Gloria Arroyo and Comelec Commissioner Virgilio Garcillano,
respondent DOJ Secretary Gonzales warned reporters that those who had copies of the CD and those broadcasting or
publishing its contents could be held liable under the Anti-Wiretapping Act. He also stated that persons possessing or
airing said tapes were committing a continuing offense, subject to arrest by anybody. Finally, he stated that he had
ordered the NBI to go after media organizations “found to have caused the spread, the playing and the printing of the
contents of a tape.” Meanwhile, respondent NTC warned TV and radio stations that their broadcast/airing of such false
information and/or willful misrepresentation shall be a just cause for the suspension, revocation and/or cancellation of
the licenses or authorizations issued to the said media establishments. Petitioner Chavez filed a petition under Rule 65
against respondents Secretary Gonzales and the NTC directly with the Supreme Court.

Issues:
(1) Will a purported violation of law such as the Anti-Wiretapping Law justify straitjacketing the exercise of freedom of
speech and of the press?
(2) Did the mere press statements of respondents DOJ Secretary and the NTC constitute a form of content-based prior
restraint that has transgressed the Constitution?

Held:
(1) No, a purported violation of law such as the Anti-Wiretapping Law will not justify straitjacketing the exercise of
freedom of speech and of the press. A governmental action that restricts freedom of speech or of the press based on
content is given the strictest scrutiny, with the government having the burden of overcoming the presumed
unconstitutionality by the clear and present danger rule. This rule applies equally to all kinds of media, including
broadcast media. Respondents, who have the burden to show that these acts do not abridge freedom of speech and of
the press, failed to hurdle the clear and present danger test. For this failure of the respondents alone to offer proof to
satisfy the clear and present danger test, the Court has no option but to uphold the exercise of free speech and free
press. There is no showing that the feared violation of the anti-wiretapping law clearly endangers the national security
of the State.

(2) Yes, the mere press statements of respondents DOJ Secretary and the NTC constituted a form of content-based prior
restraint that has transgressed the Constitution. It is not decisive that the press statements made by respondents were
not reduced in or followed up with formal orders or circulars. It is sufficient that the press statements were made by
respondents while in the exercise of their official functions. Any act done, such as a speech uttered, for and on behalf of
the government in an official capacity is covered by the rule on prior restraint. The concept of an “act” does not limit
itself to acts already converted to a formal order or official circular. Otherwise, the non formalization of an act into an
official order or circular will result in the easy circumvention of the prohibition on prior restraint.
Disini, Jr. v. The Secretary of Justice
G.R. No. 203335, February 18, 2014

FACTS:
Petitioners Jose Jesus M. Disini, Jr., Rowena S. Disini, Lianne Ivy P. Medina, Janette Toral and Ernesto Sonido, Jr., as
taxpayers, file a Petition for Certiorari and Prohibition under Rule 65 of the 1997 Rules of Civil Procedure, the petitioners
seek to 1) nullify Sections 4(c)(4), 6, 7, 12 and 19 of RA 10175, otherwise known as the “Cybercrime Prevention Act of
2012” for violating the fundamental rights protected under the Constitution; and 2) prohibit the Respondents, singly and
collectively, from enforcing the afore-mentioned provisions of the Cybercrime Act.

ISSUES/GROUNDS:
1. Sections 4(c)(4), 6, 7, 12 and 19 of The Cybercrime Act violate the petitioners’ constitutionally protected
rights to freedom of expression, due process, equal protection, privacy of communications, as well as the
Constitutional sanctions against double jeopardy, undue delegation of legislative authority and the right
against unreasonable searches and seizure;
• Sections 6 and 7 of the Cybercrime Act more than doubles the liability for imprisonment for any violation of
existing penal laws are in violation of the petitioners’ right against Double Jeopardy;
• Section 12 of the Cybercrime Act, which permits the NBI and the PNP “with due cause” to engage in real
time collection of traffic data without the benefit of the intervention of a judge, violates the Petitioners’
Constitutionally-protected right to be free from unreasonable searches and seizure as well as the right to
the privacy of communications;
• Section 19 of the Cybercrime Act, which authorizes the Respondent Secretary of DOJ to block or restrict
access to any content upon a prima facie finding that the same violates the law, contains an undue
delegation of legislative authority, infringes upon the judicial power of the judiciary, and violates the
Petitioners’ Constitutionally-protected right to due process and freedom of expression; and
• Section 4(c)(4) defines libel as a cybercrime and in relation to Section 6 of the law increased the penalty
from 6 months to 4 years and 2 months to the greater period of 6 years to 10 years, infringes upon the right
to freedom of expression and also restricts the freedom of the press. Under Section 12, a prima facie finding
by the Secretary of DOJ can trigger an order directed at service providers to block access to the said material
without the benefit of a trial or a conviction. Thus, RA 10175 infringes upon the right to freedom of
expression and also restricts the freedom of the press. The increased penalties, plus the ease by which
allegedly libelous materials can be removed from access, work together as a “chilling effect” upon protected
speech.

No other plain, speedy, or adequate remedy in the court of law, and that this Petition is therefore cognizable by the SC’s
judicial power under Article VIII, Section 1 par. 2 of the Constitution and pursuant to Rule 65, Sec. 1 of the 1997 Rules of
Civil Procedure, as amended.

ARGUMENTS/DISCUSSIONS:
The Cybercrime Act Violates Free Speech:
• imposes heavier penalties for online libel than paper-based libel; single act of online libel will result in two
convictions penalized separately under the RP and the Cybercrime Act;
• online libel under the Cybercrime Act will ensure the imprisonment of the accused and for a much longer period.
Such changes will result in a chilling effect upon the freedom of speech;
• with the passage of the Cybercrime Act, Senator Vicente Sotto III’s earlier threat to criminally prosecute all
bloggers and internet users who were critical of his alleged plagiarism of online materials for use in his speech
against the Reproductive Health Bill became real; threat of criminal prosecution under RA 10175 will work to
preclude people such as Petitioners from posting social commentaries online, thus creating a “chilling effect”
upon the freedom of expression;
• gives the DOJ Secretary blanket authority to restrain and block access to content whether authored by private
citizens or the organized press sans any hearing of any kind but merely upon a mere prima facie showing that a
particular Internet article constitutes online libel;
• respondents must demonstrate how the Cybercrime Act will fare under strict scrutiny

Sections 6 and 7 of the Cybercrime Act violate the Double Jeopardy and Equal Protection Clauses of the Constitution:
• Persons who commit crimes using information and communication technologies (ICTs) face the possibility of
being imprisoned more than double the imprisonment laid down in the RPC or special law, simply by the
passage of the Cybercrime Act;
• the cybercrimes defined and punished under Section 6 of the Act are absolutely identical to the crimes defined
in the RPC and special laws which raises the possibility that an accused will be punished twice for the same
offense in violation of the Constitution;
• Congress created a class of offenders who commit crimes “by, through or with the use” of ICTs in violation of the
equal protection clause

The Real Time Collection of Traffic Date Violate the Right to Privacy and the Right Against Unreasonable Searches and
Seizure:
• No compelling state interest that justifies real time collection of data; the authority vested on the Philippine
National Police and the National Bureau of Investigation to collect data is not bounded by any reasonable
standard except “due cause” which presumably, the PNP and NBI will determine for itself;
• While the privacy of suspected terrorists, through the Human Security Act, are protected by the intervention of
the Court of Appeals before surveillance operations are conducted, the privacy of all citizens may be infringed
without judicial participation in the Cybercrime Act;
• Neither the PNP nor the NBI is required to justify the incursion into the right to privacy;
• No limits imposed upon the PNP or the NBI since they can lawfully collect traffic data at all times without
interruption;
• No stated justification for this warrant-free unlimited incursion into the privacy of citizens

The Respondent DOJ Secretary’s Take Down Authority under Section 19 of the Cybercrime Act violates Due Process and
is an Undue Delegation of Legislative Authority
• The DOJ Secretary’s overwhelming powers to order the restriction or blocking of access to certain content upon
a mere prima facie finding without any need for a judicial determination is in clear violation of petitioners’
Constitutionally protected right to due process;
• The Cybercrime Act contemplates that the respondent DOJ Secretary will be “judge, jury and executioner” of all
cybercrime-related complaints;
• To consider that all penal provisions in all specials laws are cybercrimes under Section 6, it follows that:
1. Complaints filed by intellectual property rights owners may be acted upon the Respondent DOJ Secretary to
block access to websites and content upon a mere prima facie showing of an infringement;
2. Foreign sites (e.g. Amazon.com) offering goods on retail to Philippine citizens may be blocked for violating the
Retail Trade Law;
3. Foreign service providers such as Skype may be blocked from offering voice services without securing a license
from the National Telecommunications Communication;
4. YouTube video may be blocked for presumably violating the IP Code.
The Cybercrime Act fails the two tests laid down by the Court in Abakada Guro Party List v. Purisima (GR No. 166715) to
determine the validity of delegation of legislative power: (1) the completeness test and (2) the sufficient standard test
1. Nowhere in the Cybercrime Act’s declaration of policy does it lay down the legislative policy with respect to the
blocking of content. No limits upon the takedown power of the respondent DOJ Secretary;
2. Prima facie standard is not enough to prevent the DOJ Secretary from exercising infinite discretion and
becoming the supreme authority in the Philippine Internet landscape.
GMA NETWORK, INC., et al v. COMMISSION ON ELECTIONS
G.R.Nos. 205357, 2 September 2014,

Facts:
Political speech is one of the most important expressions protected by the Fundamental Law. “Freedom of speech, of
expression, and of the press are at the core of civil liberties and have to be protected at all costs for the sake of
democracy.” The “aggregate-based” airtime limits is unreasonable and arbitrary as it unduly restricts and constrains the
ability of candidates and political parties to reach out and communicate with the people.
Resolution 9615 of the Commission on Elections (COMELEC) changed the airtime limitations for political campaign from
“per station” basis, as used during the 2007 and 2010 elections, to a “total aggregate” basis for the 2013. Various
broadcast networks such as ABS-CBN, ABC, GMA, MBC, NBN, RMN and KBP questioned the interpretation of the
COMELEC on the ground that the provisions are oppressive and violative of the constitutional guarantees of freedom of
expression and of the press.
Collectively, they question the constitutionality of Section 9 (a), which provides for an “aggregate total” airtime instead
of the previous “per station” airtime for political campaigns or advertisements, and also required prior COMELEC
approval for candidates' television and radio guestings and appearances. Petitioners claim that Section 9(a) limits the
computation of “aggregate total” airtime and imposes unreasonable burden on broadcast media of monitoring a
candidate’s or political party’s aggregate airtime. On the other hand, COMELEC alleges that the broadcast networks do
not have locus standi, as the limitations are imposed on candidates, not on media outlets.
Comelec maintains that the per candidate rule or total aggregate airtime limit is in accordance with the Fair Election Act
as this would truly give life to the constitutional objective to equalize access to media during elections. It sees this as a
more effective way of "levelling the playing field" between candidates/political parties with enormous resources and
those without much.

ISSUES:
1. Does Section 9(a) of Comelec Resolution No. 9615 on airtime limit violate the constitutional guaranty of freedom of
expression, of speech and of the press?
2. Does resolution No. 9165 impose unreasonable burden on the broadcast industry?

RULING:
1. Yes, Section 9(a) of COMELEC Resolution No. 9615, with its adoption of the “aggregate-based” airtime limits
unreasonably restricts the guaranteed freedom of speech and of the press.
Political speech is one of the most important expressions protected by the Fundamental Law. “Freedom of speech, of
expression, and of the press are at the core of civil liberties and have to be protected at all costs for the sake of
democracy.”
GMA came up with its analysis of the practical effects of such a regulation: Given the reduction of a candidate’s airtime
minutes in the New Rules, petitioner GMA estimates that a national candidate will only have 120 minutes to utilize for
his political advertisements in television during the whole campaign period of 88 days, or will only have 81.81 seconds
per day TV exposure allotment. If he chooses to place his political advertisements in the 3 major TV networks in equal
allocation, he will only have 27.27 seconds of airtime per network per day. This barely translates to 1 advertisement spot
on a 30-second spot basis in television.
The Court agrees. The assailed rule on “aggregate-based” airtime limits is unreasonable and arbitrary as it unduly
restricts and constrains the ability of candidates and political parties to reach out and communicate with the people.
Here, the adverted reason for imposing the “aggregate-based” airtime limits – leveling the playing field – does not
constitute a compelling state interest which would justify such a substantial restriction on the freedom of candidates
and political parties to communicate their ideas, philosophies, platforms and programs
of government.
2. No, Resolution No. 9615 does not impose an unreasonable burden on the broadcast industry
The Court cannot agree with the contentions of GMA. The apprehensions of COMELEC appear more to be the result of a
misappreciation of the real import of the regulation rather than a real and present threat to its broadcast activities. The
Court is more in agreement with COMELEC when it explained that the legal duty of monitoring lies with the COMELEC.
Broadcast stations are merely required to submit certain documents to aid the COMELEC in ensuring that candidates are
not sold airtime in excess of the allowed limits. There is absolutely no duty on the broadcast stations to do monitoring,
much less monitoring in real time. GMA grossly exaggerates when it claims that the non-existent duty would require
them to hire and train an astounding additional 39,055 personnel working on eight-hour shifts all over the country.
THE DIOCESE OF BACOLOD, REPRESENTED BY THE MOST REV. BISHOP VICENTE M. NAVARRA and THE BISHOP HIMSELF
IN HIS PERSONAL CAPACITY, Petitioners,
vs.
COMMISSION ON ELECTIONS AND THE ELECTION OFFICER OF BACOLOD CITY, ATTY. MAVIL V. MAJARUCON,
Respondents.
G.R. No. 205728, January 21, 2015

TOPIC: Right to expression, right to political speech, right to property

FACTS:
On February 21, 2013, petitioners posted two (2) tarpaulins within a private compound housing the San
Sebastian Cathedral of Bacolod. Each tarpaulin was approximately six feet (6′) by ten feet (10′) in size. They were posted
on the front walls of the cathedral within public view. The first tarpaulin contains the message “IBASURA RH Law”
referring to the Reproductive Health Law of 2012 or Republic Act No. 10354. The second tarpaulin is the subject of the
present case. This tarpaulin contains the heading “Conscience Vote” and lists candidates as either “(Anti-RH) Team
Buhay” with a check mark, or “(Pro-RH) Team Patay” with an “X” mark. The electoral candidates were classified
according to their vote on the adoption of Republic Act No. 10354, otherwise known as the RH Law. Those who voted for
the passing of the law were classified by petitioners as comprising “Team Patay,” while those who voted against it form
“Team Buhay.”
Respondents conceded that the tarpaulin was neither sponsored nor paid for by any candidate. Petitioners also
conceded that the tarpaulin contains names ofcandidates for the 2013 elections, but not of politicians who helped in the
passage of the RH Law but were not candidates for that election.

ISSUES:
1. Whether or not the size limitation and its reasonableness of the tarpaulin is a political question, hence not
within the ambit of the Supreme Court’s power of review.
2. Whether or not the petitioners violated the principle of exhaustion of administrative remedies as the case
was not brought first before the COMELEC EnBanc or any if its divisions.
3. Whether or not COMELEC may regulate expressions made by private citizens.
4. Whether or not the assailed notice and letter for the removal of the tarpaulin violated petitioners’
fundamental right to freedom of expression.
5. Whether the order for removal of the tarpaulin is a content-based or content-neutral regulation.
6. Whether or not there was violation of petitioners’ right to property.
7. Whether or not the tarpaulin and its message are considered religious speech.

HELD:

FIRST ISSUE: No.


The Court ruled that the present case does not call for the exercise of prudence or modesty. There is no political
question. It can be acted upon by this court through the expanded jurisdiction granted to this court through Article VIII,
Section 1 of the Constitution..
The concept of a political question never precludes judicial review when the act of a constitutional organ
infringes upon a fundamental individual or collective right. Even assuming arguendo that the COMELEC did have the
discretion to choose the manner of regulation of the tarpaulin in question, it cannot do so by abridging the fundamental
right to expression.
Also the Court said that in our jurisdiction, the determination of whether an issue involves a truly political and
non-justiciable question lies in the answer to the question of whether there are constitutionally imposed limits on
powers or functions conferred upon political bodies. If there are, then our courts are duty-bound to examine whether
the branch or instrumentality of the government properly acted within such limits.
A political question will not be considered justiciable if there are no constitutionally imposed limits on powers
or functions conferred upon political bodies. Hence, the existence of constitutionally imposed limits justifies subjecting
the officialactions of the body to the scrutiny and review of this court.
In this case, the Bill of Rights gives the utmost deference to the right to free speech. Any instance that this right
may be abridged demands judicial scrutiny. It does not fall squarely into any doubt that a political question brings.

SECOND ISSUE: No.


The Court held that the argument on exhaustion of administrative remedies is not proper in this case.
Despite the alleged non-exhaustion of administrative remedies, it is clear that the controversy is already ripe
for adjudication. Ripeness is the “prerequisite that something had by then been accomplished or performed by either
branch or in this case, organ of government before a court may come into the picture.”
Petitioners’ exercise of their right to speech, given the message and their medium, had understandable
relevance especially during the elections. COMELEC’s letter threatening the filing of the election offense against
petitioners is already an actionable infringement of this right. The impending threat of criminal litigation is enough to
curtail petitioners’ speech.
In the context of this case, exhaustion of their administrative remedies as COMELEC suggested in their
pleadings prolongs the violation of their freedom of speech.

THIRD ISSUE: No.


Respondents cite the Constitution, laws, and jurisprudence to support their position that they had the power to
regulate the tarpaulin. However, the Court held that all of these provisions pertain to candidates and political parties.
Petitioners are notcandidates. Neither do they belong to any political party. COMELEC does not have the authority to
regulate the enjoyment of the preferred right to freedom of expression exercised by a non-candidate in this case.

FOURTH ISSUE: Yes.


The Court held that every citizen’s expression with political consequences enjoys a high degree of protection.
Moreover, the respondent’s argument that the tarpaulin is election propaganda, being petitioners’ way of
endorsing candidates who voted against the RH Law and rejecting those who voted for it, holds no water.
The Court held that while the tarpaulin may influence the success or failure of the named candidates and
political parties, this does not necessarily mean it is election propaganda. The tarpaulin was not paid for or posted “in
return for consideration” by any candidate, political party, or party-list group.
By interpreting the law, it is clear that personal opinions are not included, while sponsored messages are
covered.
The content of the tarpaulin is a political speech
Political speech refers to speech “both intended and received as a contribution to public deliberation about some issue,”
“fostering informed and civic minded deliberation.” On the other hand, commercial speech has been defined as speech
that does “no more than propose a commercial transaction.” The expression resulting from the content of the tarpaulin
is, however, definitely political speech.

FIFTH ISSUE: Content-based regulation.


Content-based restraint or censorship refers to restrictions “based on the subject matter of the utterance or
speech.” In contrast, content-neutral regulation includes controls merely on the incidents of the speech such as time,
place, or manner of the speech.
The Court held that the regulation involved at bar is content-based. The tarpaulin content is not easily divorced
from the size of its medium.
Content-based regulation bears a heavy presumption of invalidity, and this court has used the clear and present
danger rule as measure.
Under this rule, “the evil consequences sought to be prevented must be substantive, ‘extremely serious and the
degree of imminence extremely high.’” “Only when the challenged act has overcome the clear and present danger rule
will it pass constitutional muster, with the government having the burden of overcoming the presumed
unconstitutionality.”
Even with the clear and present danger test, respondents failed to justify the regulation. There is no compelling
and substantial state interest endangered by the posting of the tarpaulin as to justify curtailment of the right of freedom
of expression. There is no reason for the state to minimize the right of non-candidate petitioners to post the tarpaulin in
their private property. The size of the tarpaulin does not affect anyone else’s constitutional rights.

SIXTH ISSUE: Yes.


The Court held that even though the tarpaulin is readily seen by the public, the tarpaulin remains the private
property of petitioners. Their right to use their property is likewise protected by the Constitution.
Any regulation, therefore, which operates as an effective confiscation of private property or constitutes an
arbitrary or unreasonable infringement of property rights is void, because it is repugnant to the constitutional guaranties
of due process and equal protection of the laws.
The Court in Adiong case held that a restriction that regulates where decals and stickers should be posted is “so
broad that it encompasses even the citizen’s private property.” Consequently, it violates Article III, Section 1 of the
Constitution which provides that no person shall be deprived of his property without due process of law.

SEVENTH ISSUE: No.


The Court held that the church doctrines relied upon by petitioners are not binding upon this court. The
position of the Catholic religion in the Philippines as regards the RH Law does not suffice to qualify the posting by one of
its members of a tarpaulin as religious speech solely on such basis. The enumeration of candidates on the face of the
tarpaulin precludes any doubt as to its nature as speech with political consequences and not religious speech.
Doctrine of benevolent neutrality
With religion looked upon with benevolence and not hostility, benevolent neutrality allows accommodation of
religion under certain circumstances. Accommodations are government policies that take religion specifically into
account not to promote the government’s favored form of religion, but to allow individuals and groups to exercise their
religion without hindrance. Their purpose or effect therefore is to remove a burden on, or facilitate the exercise of, a
person’s or institution’s religion.
As Justice Brennan explained, the “government may take religion into account . . . to exempt, when possible,
from generally applicable governmental regulation individuals whose religious beliefs and practices would otherwise
thereby be infringed, or to create without state involvement an atmosphere in which voluntary religious exercise may
flourish.”

Lemon test
A regulation is constitutional when:
1. It has a secular legislative purpose;
2. It neither advances nor inhibits religion; and
3. It does not foster an excessive entanglement with religion.
SECTION 4 ASSEMBLY AND PETITION
Primicias vs. Fugoso
GR L-1800, 27 January 1948

Facts:
The Philippine Legislature has delegated the exercise of the police power to the Municipal Board of the City of Manila,
which according to section 2439 of the Administrative Code is the legislative body of the City. Section 2444 of the same
Code grants the Municipal Board, among others, the following legislative powers, to wit: "(p) to provide for the
prohibition and suppression of riots, affrays, disturbances and disorderly assemblies, (u) to regulate the use of streets,
avenues, parks, cemeteries and other public places" and "for the abatement of nuisances in the same," and "(ee) to
enact all ordinances it may deem necessary and proper for sanitation and safety, the furtherance of prosperity and the
promotion of morality, peace, good order, comfort, convenience, and general welfare of the city and its inhabitants."

Under the above delegated power, the Municipal Board of the City of Manila, enacted sections 844 and 1119. Section
844 of the Revised Ordinances of 1927 prohibits as an offense against public peace, and section 1262 of the same
Revised Ordinance penalizes as a misdemeanor, "any act, in any public place, meeting, or procession, tending to disturb
the peace or excite a riot; or collect with other persons in a body or crowd for any unlawful purpose; or disturb or
disquiet any congregation engaged in any lawful assembly."

And section 1119 provides that "The streets and public places of the city shall be kept free and clear for the use of the
public, and the sidewalks and crossings for the pedestrians, and the same shall only be used or occupied for other
purposes as provided by ordinance or regulation: Provided,
That the holding of athletic games, sports, or exercises during the celebration of national holidays in any streets or
public places of the city and on the patron saint day of any district in question, may be permitted by means of a permit
issued by the Mayor, who shall determine the streets or public places, or portions thereof, where such athletic games,
sports, or exercises may be held: And provided, further,
That the holding of any parade or procession in any streets or public places is prohibited unless a permit therefor is
first secured from the Mayor, who shall, on every such occasion, determine or specify the streets or public places for the
formation, route, and dismissal of such parade or procession: And provided, finally,
That all applications to hold a parade or procession shall be submitted to the Mayor not less than twenty-four hours
prior to the holding of such parade or procession." An action of mandamus was instituted by Cipriano Primicias, a
campaign manager of the Coalesced Minority Parties against Valeriano Fugoso, as Mayor of the City of Manila, to
compel the latter to issue a permit for the holding of a public meeting at Plaza Miranda on Sunday afternoon, 16
November 1947, for the purpose of petitioning the government for redress to grievances on the ground that Fugoso
refused to grant such permit.

Due to the urgency of the case, the Court, after mature deliberation, issued a writ of mandamus, as prayed for in the
petition on 15 November 1947, without prejudice to writing later an extended and reasoned decision.

Issue:
Whether the Mayor of Manila may be compelled to issue a permit to use Plaza Miranda to hold a public meeting.

Held:
The right to freedom of speech, and to peacefully assemble and petition the government for redress of grievances, are
fundamental personal rights of the people recognized and guaranteed by the constitutions of democratic countries. But
it is a settled principle growing out of the nature of well-ordered civil societies that the exercise of those rights is not
absolute for it may be so regulated that it shall not be injurious to the equal enjoyment of others having equal rights, nor
injurious to the rights of the community or society. The power to regulate the exercise of such and other constitutional
rights is termed the sovereign "police power," which is the power to prescribe regulations, to promote the health,
morals, peace, education, good order or safety, and general welfare of the people. This sovereign police power is
exercised by the government through its legislative branch by the enactment of laws regulating those and other
constitutional and civil rights, and it may be delegated to political subdivisions, such as towns, municipalities and cities
by authorizing their legislative bodies called municipal and city councils to enact ordinances for the purpose.
Herein, as there is no express and separate provision in the Revised Ordinance of the City regulating the holding of
public meeting or assembly at any street or public places, the provisions of said section 1119 regarding the holding of
any parade or procession in any street or public places may be applied by analogy to meeting and assembly in any street
or public places. The provisions of the said ordinance are construed to mean that it does not confer upon the Mayor the
power to refuse to grant the permit, but only the discretion, in issuing the permit, to determine or specify the streets or
public places where the parade or procession may pass or the meeting may be held.

The Court cannot adopt the other alternative construction or construe the ordinance under consideration as conferring
upon the Mayor power to grant or refuse to grant the permit, which would be tantamount to authorizing him to prohibit
the use of the streets and other public places for holding of meetings, parades or processions, because such a
construction would make the ordinance invalid and void or violative of the constitutional limitations. As the Municipal
Board is empowered only to regulate the use of streets, parks, and other public places, and the word "regulate," as used
in section 2444 of the Revised Administrative Code, eans and includes the power to control, to govern, and to restrain,
but cannot be construed as synonymous with "suppress" or "prohibit,", the Municipal Board cannot grant the Mayor a
power which it does not have.

In view of all the foregoing, the petition for mandamus was granted and, there appearing no reasonable objection to the
use of the Plaza Miranda, Quiapo, for the meeting applied for, the mayor was ordered to issue the corresponding
permit, as requested.
Malabanan vs. Ramento
GR 62270, 21 May 1984

Facts:
Crispin Malabanan, Evilio Jalos, Ben Luther Lucas, Sotero Leonero and June Lee were officers of the Supreme Student
Council of the Gregorio Araneta University Foundation. They sought and were granted by the school authorities a permit
to hold a meeting from 8:00 A.M. to 12:00 P.M. on 27 August 1982. Pursuant to such permit, along with other students,
they held a general assembly at the Veterinary Medicine and Animal Science basketball court (VMAS), the place
indicated in such permit, not in the basketball court as therein stated but at the second floor lobby. At such gathering
they manifested In vehement and vigorous language their opposition to the proposed merger of the Institute of Animal
Science with the Institute of Agriculture. At 10:30 A.M., the same day, they marched toward the Life Science Building
and continued their rally. It was outside the area covered by their permit. They continued their demonstration, giving
utterance to language severely critical of the University authorities and using megaphones in the process. There was, as
a result, disturbance of the classes being held. Also, the non-academic employees, within hearing distance, stopped their
work because of the noise created. They were asked to explain on the same day why they should not be held liable for
holding an illegal assembly. Then on 9 September 1982, they were informed through a memorandum that they were
under preventive suspension for their failure to explain the holding of an illegal assembly in front of the Life Science
Building.
The validity thereof was challenged by Malabanan, et. al. both before the Court of First Instance of Rizal in a petition for
mandamus with damages against Cesar Mijares, in his capacity as the President of GAUF, Gonzalo del Rosario, in his
capacity as the Director for Academic Affairs of GAUF; Tomas B. Mesina, in his capacity as the Dean of Student Affairs of
GAUF; Atty. Leonardo Padilla, in his capacity as Chief Legal Counsel & Security Supervisor of GAUF; Atty. Fablita Ammay,
Rosendo Galvante and Eugenia Tayao, in their capacities as members of the Ad Hoc Committee of GAUF and before the
Ministry of Education, Culture, and Sports. On 20 October 1982, Anastacio D. Ramento, as Director of the National
Capital Region, found Malabanan, et. al. guilty of the charge of having violated paragraph 146(c) of the Manual for
Private Schools more specifically their holding of an illegal assembly which was characterized by the violation of the
permit granted resulting in the disturbance of classes and oral defamation. The penalty was suspension for one
academic year.
Hence, the petition for certiorari, prohibition and mandamus.

Issue:
Whether the students were properly meted out a year suspension due to the disruption of classes in GAUF attended by
the students’ concerted activity.

Held:
Malabanan, et.al. are entitled to their rights to peaceable assembly and free speech. They enjoy like the rest of the
citizens the freedom to express their views and communicate their thoughts to those disposed to listen in gatherings.
They do not, to borrow from the opinion of Justice Fortas in Tinker v. Des Moines Community School District, "shed their
constitutional rights to freedom of speech or expression at the schoolhouse gate." While, therefore, the authority of
educational institutions over the conduct of students must be recognized, it cannot go so far as to be violative of
constitutional safeguards. On a more specific level, there is persuasive force to this formulation in the Fortas opinion:
"The principal use to which the schools are dedicated is to accommodate students during prescribed hours for the
purpose of certain types of activities. Among those activities is personal intercommunication among the students. This is
not only an inevitable part of the process of attending school; it is also an important part of the educational process. A
student's rights, therefore, do not embrace merely the classroom hours. When he is in the cafeteria, or on the playing
field, or on the campus during the authorized hours, he may express his opinions, even on controversial subjects like the
conflict in Vietnam, if he does so without 'materially and substantially interfer[ing] with the requirements of appropriate
discipline in the operation of the school' and without colliding with the rights of others. But conduct by the student, in
class or out of it, which for any reason — whether it stems from time, place, or type of behavior — materially disrupts
classwork or involves substantial disorder or invasion of the rights of others is, of course, not immunized by the
constitutional guarantee of freedom of speech." It does not follow, however, that Malabanan, et. al. can be totally
absolved for the events that transpired. Admittedly, there was a violation of the terms of the permit. The rally was held
at a place other than that specified, in the second floor lobby, rather than the basketball court, of the VMAS building of
the University. Moreover, it was continued longer than the period allowed. The "concerted activity went on until 5:30
p.m." The University could thus, take disciplinary action. On those facts, however, an admonition, even a censure —
certainly not a suspension — could be the appropriate penalty. A one-year period of suspension is much too severe,
while the discretion of both the University and Director Ramento is recognized, the rule of reason, the dictate of fairness
calls for a much lesser penalty. If the concept of proportionality between the offense committed and the sanction
imposed is not followed, an element of arbitrariness intrudes. That would give rise to a due process question. To avoid
this constitutional objection, it is the holding of the Court that a one-week suspension would be punishment enough.
DELA CRUZ V. COURT OF APPEALS
G.R. No. 126183, March 25, 1999

FACTS
Petitioners are public school teachers who were simultaneously charged, preventively suspended, and eventually
dismissed by Sec. Carino in Oct. 1990. It was alleged that the teachers participated in the mass action/ illegal strike on
Sept. 1990. The teachers also violated the return-to-work order issued by the DECS. Respondents failed to explain to the
DECS despite the 5 day period given. Hence they were found guilty as charged, and subsequently dismissed from office
by Sec. Carino of the DECS. The Civil Service Commission, upon appeal, found the teachers guilty of conduct prejudicial
to the best interest of service, and imposed upon them the reduced penalty of six month’s suspension. However in view
of the length of time that the teachers had been out of service due to the dismissal issued by Sec. Carino, the CSC
likewise ordered their immediate reinstatement without back wages.

ISSUE
1. Whether the teachers’ conducts are prejudicial to the best interest of service.
2. Whether or not the teachers are entitled to back wages for the period of 3 years pending their appeal deducting the 6
months’ suspension eventually meted out to them.

HELD
1. YES, the mass actions amounted to a prohibited strike of civil service servants. Although the right to peaceably
assemble and petition the government for redress of grievances is guaranteed by the Constitution, this liberty must be
exercised within reasonable limits. The public school teachers committed acts prejudicial to the interest of the service by
staging the mass protests on regular school days, abandoning their classes and failing to return despite the return to
work order.
2. NO, they are not entitled to backwages. The teachers were neither exonerated nor unjustifiably suspended, the 2
circumstances necessary for the grant of backwages in administrative disciplinary cases.
PBM Employees vs Philippine Blooming Mills
G.R. No. L-31195, June 5, 1973

Facts:
The petitioner Philippine Blooming Mills Employees Organization (PBMEO) is a legitimate labor union composed of the
employees of the respondent Philippine Blooming Mills Co., Inc., and petitioners. Benjamin Pagcu and Rodulfo Munsod
are officers and members of the petitioner Union. Petitioners claim that on March 1, 1969, they decided to stage a mass
demonstration at Malacañang on March 4, 1969, in protest against alleged abuses of the Pasig police. PBMEO thru
Pagcu confirmed the planned demonstration and stated that the demonstration or rally cannot be cancelled because it
has already been agreed upon in the meeting. Pagcu explained further that the demonstration has nothing to do with
the Company because the union has no quarrel or dispute with Management. The Management, thru Atty. C.S. de Leon,
Company personnel manager, informed PBMEO that the demonstration is an inalienable right of the union guaranteed
by the Constitution but emphasized that any demonstration for that matter should not unduly prejudice the normal
operation of the Company. Workers who without previous leave of absence approved by the Company, particularly , the
officers present who are the organizers of the demonstration, who shall fail to report for work the following morning
shall be dismissed, because such failure is a violation of the existing CBA and, therefore, would be amounting to an
illegal strike. Because the petitioners and their members numbering about 400 proceeded with the demonstration
despite the pleas of the respondent Company that the first shift workers should not be required to participate in the
demonstration and that the workers in the second and third shifts should be utilized for the demonstration from 6 A.M.
to 2 P.M. on March 4, 1969, filed a charge against petitioners and other employees who composed the first shift, for a
violation of Republic Act No. 875(Industrial Peace Act), and of the CBA providing for 'No Strike and No Lockout.'
Petitioners were held guilty in by CIR for bargaining in bad faith, hence this appeal.

Issue:
Whether or Not the petitioners right to freedom of speech and to peaceable assemble violated.

Held:
Yes. A constitutional or valid infringement of human rights requires a more stringent criterion, namely existence of a
grave and immediate danger of a substantive evil which the State has the right to prevent. This is not present in the
case. It was to the interest herein private respondent firm to rally to the defense of, and take up the cudgels for, its
employees, so that they can report to work free from harassment, vexation or peril and as consequence perform more
efficiently their respective tasks enhance its productivity as well as profits. Herein respondent employer did not even
offer to intercede for its employees with the local police. In seeking sanctuary behind their freedom of expression well
as their right of assembly and of petition against alleged persecution of local officialdom, theemployees and laborers of
herein private respondent firm were fighting for their very survival, utilizing only the weapons afforded them by the
Constitution — the untrammelled enjoyment of their basic human rights. The pretension of their employer that it would
suffer loss or damage by reason of the absence of its employees from 6 o'clock in the morning to 2 o'clock in the
afternoon, is a plea for the preservation merely of their property rights. The employees' pathetic situation was a stark
reality — abused, harassment and persecuted as they believed they were by thepeace officers of the municipality. As
above intimated, the condition in which the employees found themselves vis-a-vis the local police of Pasig, was a matter
that vitally affected their right to individual existence as well as that of their families. Material loss can be repaired or
adequately compensated. The debasement of the human being broken in morale and brutalized in spirit-can never be
fully evaluated in monetary terms. As heretofore stated, the primacy of human rights — freedom of expression, of
peaceful assembly and of petition for redress of grievances — over property rights has been sustained. To regard the
demonstration against policeofficers, not against the employer, as evidence of bad faith in collective bargaining and
hence a violation of the collective bargaining agreement and a cause for the dismissal from employment of the
demonstratingemployees, stretches unduly the compass of the collective bargainingagreement, is "a potent means of
inhibiting speech" and therefore inflicts a moral as well as mortal wound on the constitutional guarantees of free
expression, of peaceful assembly and of petition. Circulation is one of the aspects of freedom of expression. If
demonstrators are reduced by one-third, then by that much the circulation of the Issue raised by the demonstration is
diminished. The more the participants, the more persons can be apprised of the purpose of the rally. Moreover, the
absence of one-third of their members will be regarded as a substantial indication of disunity in their ranks which will
enervate their position and abet continued alleged police persecution.
Bayan, et al., Vs. Eduardo Ermita
G.R. No. 169838 April 25, 2006

Facts:
The petitioners, Bayan, et al., alleged that they are citizens and taxpayers of the Philippines and that their right as
organizations and individuals were violated when the rally they participated in on October 6, 2005 was violently
dispersed by policemen implementing Batas Pambansa No. 880.

Petitioners contended that Batas Pambansa No. 880 is clearly a violation of the Constitution and the International
Covenant on Civil and Political Rights and other human rights treaties of which the Philippines is a signatory. They argue
that B.P. No. 880 requires a permit before one can stage a public assembly regardless of the presence or absence of a
clear and present danger. It also curtails the choice of venue and is thus repugnant to the freedom of expression clause
as the time and place of a public assembly form part of the message which the expression is sought. Furthermore, it is
not content-neutral as it does not apply to mass actions in support of the government. The words “lawful cause,”
“opinion,” “protesting or influencing” suggest the exposition of some cause not espoused by the government. Also, the
phrase “maximum tolerance” shows that the law applies to assemblies against the government because they are being
tolerated. As a content-based legislation, it cannot pass the strict scrutiny test. This petition and two other petitions
were ordered to be consolidated on February 14, 2006. During the course of oral arguments, the petitioners, in the
interest of a speedy resolution of the petitions, withdrew the portions of their petitions raising factual issues,
particularly those raising the issue of whether B.P. No. 880 and/or CPR is void as applied to the rallies of September 20,
October 4, 5 and 6, 2005.

Issue:
Whether the Calibrated Pre-emptive response and the Batas Pambansa No. 880, specifically Sections 4, 5, 6, 12, 13(a)
and 14(a) violates Art. III Sec. 4 of the Philippine Constitution as it causes a disturbing effect on the exercise by the
people of the right to peaceably assemble.

Held:
Section 4 of Article III of the Philippine Constitution provides that no law shall be passed abridging the freedom of
speech, of expression, or of the press, or the right of the people peaceably to assemble and petition the government for
redress of grievances. The right to peaceably assemble and petition for redress of grievances, together with freedom of
speech, of expression, and of the press, is a right that enjoys dominance in the sphere of constitutional protection. For
this rights represent the very basis of a functional democratic polity, without which all the other rights would be
meaningless and unprotected.

However, it must be remembered that the right, while sacrosanct, is not absolute. It may be regulated that it shall not
be injurious to the equal enjoyment of others having equal rights, nor injurious to the rights of the community or
society. The power to regulate the exercise of such and other constitutional rights is termed the sovereign “police
power,” which is the power to prescribe regulations, to promote the health, morals, peace, education, good order or
safety, and general welfare of the people.

B.P. No 880 is not an absolute ban of public assemblies but a restriction that simply regulates the time, place and
manner of the assemblies. B.P. No. 880 thus readily shows that it refers to all kinds of public assemblies that would use
public places. The reference to “lawful cause” does not make it content-based because assemblies really have to be for
lawful causes, otherwise they would not be “peaceable” and entitled to protection. Neither the words “opinion,”
“protesting,” and “influencing” in of grievances come from the wording of the Constitution, so its use cannot be
avoided. Finally, maximum tolerance is for the protection and benefit of all rallyist and is independent of the content of
the expression in the rally.
Furthermore, the permit can only be denied on the ground of clear and present danger to public order, public safety,
public convenience, public morals or public health. This is a recognized exception to the exercise of the rights even
under the Universal Declaration of Human Rights and The International Covenant on Civil and Political Rights.

Wherefore, the petitions are GRANTED in part, and respondents, more particularly the Secretary of the Interior and
Local Governments, are DIRECTED to take all necessary steps for the immediate compliance with Section 15 of Batas
Pambansa No. 880 through the establishment or designation of at least one suitable freedom park or plaza in every city
and municipality of the country. After thirty (30) days from the finality of this Decision, subject to the giving of advance
notices, no prior permit shall be required to exercise the right to peaceably assemble and petition in the public parks or
plaza in every city or municipality that has not yet complied with section 15 of the law. Furthermore, Calibrated pre-
emptive response (CPR), insofar as it would purport to differ from or be in lieu of maximum tolerance, is NULL and VOID
and respondents are ENJOINED to REFRAIN from using it and to STRICTLY OBSERVE the requirements of maximum
tolerance, The petitions are DISMISSED in all other respects, and the constitutionality of Batas Pambansa No. 880 is
SUSTAINED.
SECTION 5 FREEDOM OF RELIGION

Aglipay v. Ruiz
GR 45459, 13 March 1937

Facts:
In May 1936, the Director of Posts announced in the dailies of Manila that he would order the issuance of postage
stamps commemorating the celebration in the City of Manila of the 33rd International Eucharistic Congress, organized
by the Roman Catholic Church. Monsignor Gregorio Aglipay, Supreme Head of the Philippine Independent Church, in the
fulfillment of what he considers to be a civic duty, requested Vicente Sotto, Esq., member of the Philippine Bar, to
denounce the matter to the President of the Philippines. In spite of the protest of Sotto, the Director of Posts publicly
announced that the designs of the postage for printing have been sent to the United States. The said stamps were
actually issued and sold though the greater part thereof remained unsold. The further sale of the stamps was sought to
be prevented by Aglipay.

Issue:
Whether the stamp (containing a map of the Philippines, the location of the City of Manila, and an inscription that reads
"Seat XXXIII International Eucharistic Congress, Feb. 3-7, 1937") violate the Nonestablishment clause by allegedly
promoting the Catholic religion.

Held:
Section 13, Article VI, of the 1935 Constitution provides that “no public money or property shall ever be appropriated,
applied, or used, directly or indirectly, for the use, benefit, or support of any sect, church, denomination, sectarian
institution, or system of religion, or for the use, benefit, or support of any priest, preacher, minister, or other religious
teacher or dignitary as such, except when such priest, preacher, minister, or dignitary is assigned to the armed forces or
to any penal institution, orphanage, or leprosarium." The prohibition is a direct corollary of the principle of separation of
church and state. Act 4052 contemplates no religious purpose in view. What it gives the Director of Posts is the
discretionary power to determine when the issuance of special postage stamps would be "advantageous to the
Government." Of course, the phrase "advantageous to the Government" does not authorize the violation of the
Constitution; i.e. to appropriate, use or apply of public money or property for the use, benefit or support of a particular
sect or church. Herein, the issuance of the postage stamps was not inspired by any sectarian feeling to favor a particular
church or religious denominations. The stamps were not issued and sold for the benefit of the Roman Catholic Church,
nor were money derived from the sale of the stamps given to that church. The purpose of the issuing of the stamps was
to take advantage of an event considered of international importance to give publicity to the Philippines and its people
and attract more tourists to the country. Thus, instead of showing a Catholic chalice, the stamp contained a map of the
Philippines, the location of the City of Manila, and an inscription that reads "Seat XXXIII International Eucharistic
Congress, Feb. 3-7, 1937." Thus, while the issuance and sale of the stamps may be said to be inseparably linked with an
event of a religious character, the resulting propaganda received by the Roman Catholic Church, was not the aim and
purpose of the Government. The Government should not be embarrassed in its activities simply because of incidental
results, more or less religious in character, if the purpose had in view is one which could legitimately be undertaken by
appropriate legislation. The main purpose should not be frustrated by its subordination to mere incidental results not
contemplated.
GARCES VS. ESTENZO
104 SCRA 510; G.R. L-53487; 25 MAY 1981

Facts:
Two resolutions of the Barangay Council of Valencia, Ormoc City were passed:
a. Resolution No. 5- Reviving the traditional socio-religious celebration every fifth of April. This provided
for the acquisition of the image of San Vicente Ferrer and the construction of a waiting shed. Funds for
the said projects will be obtained through the selling of tickets and cash donations.
b. Resolution No. 6- The chairman or hermano mayor of the fiesta would be the caretaker of the image of
San Vicente Ferrer and that the image would remain in his residence for one year and until the election
of his successor. The image would be made available to the Catholic Church during the celebration of
the saint’s feast day.
These resolutions have been ratified by 272 voters, and said projects were implemented. The image was temporarily
placed in the altar of the Catholic Church of the barangay. However, after a mass, Father Sergio Marilao Osmeña refused
to return the image to the barangay council, as it was the church’s property since church funds were used in its
acquisition.
Resolution No. 10 was passed for the authorization of hiring a lawyer for the replevin case against the priest for the
recovery of the image. Resolution No. 12 appointed Brgy. Captain Veloso as a representative to the case. The priest, in
his answer assailed the constitutionality of the said resolutions. The priest with Andres Garces, a member of the
Aglipayan Church, contends that Sec. 8 Article IV1 and Sec 18(2) Article VIII) 2 of the constitution was violated.

Issue:
Whether or Not any freedom of religion clause in the Constitution violated.

Held:
No. As said by the Court this case is a petty quarrel over the custody of the image. The image was purchased in
connection with the celebration of the barrio fiesta and not for the purpose of favoring any religion nor interfering with
religious matters or beliefs of the barrio residents. Any activity intended to facilitate the worship of the patron saint
(such as the acquisition) is not illegal. Practically, the image was placed in a layman’s custody so that it could easily be
made available to any family desiring to borrow the image in connection with prayers and novena. It was the council’s
funds that were used to buy the image, therefore it is their property. Right of the determination of custody is their right,
and even if they decided to give it to the Church, there is no violation of the Constitution, since private funds were used.
Not every government activity which involves the expenditure of public funds and which has some religious tint is
violative of the constitutional provisions regarding separation of church and state, freedom of worship and banning the
use of public money or property.
American Bible Society v. City of Manila
GR L-9637, 30 April 1957

Facts:
The American Bible Society, is a foreign, non-stock, non-profit, religious, missionary corporation duly registered and
doing business in the Philippines through its Philippine agency established in Manila in November 1898. The City of
Manila, is a municipal corporation with powers that are to be exercised in conformity with the provisions of Republic Act
409, (Revised Charter of the City of Manila). In the course of its ministry, the Society's Philippine agency has been
distributing and selling bibles and/or gospel portions thereof (except during the Japanese occupation) throughout the
Philippines and translating the same into several Philippine dialects. On 29 May 1953, the acting City Treasurer of the
City of Manila informed the Society that it was conducting the business of general merchandise since November 1945,
without providing itself with the necessary Mayor's permit and municipal license, in violation of Ordinance 3000, as
amended, and Ordinances 2529, 3028 and 3364, and required the Society to secure, within 3 days, the corresponding
permit and license fees, together with compromise covering the period from the 4th quarter of 1945 to the 2nd quarter
of 1953, in the total sum of P5,821.45. On 24 October 1953, the Society paid to the City under protest the said permit
and license fees, giving at the same time notice to the City Treasurer that suit would be taken in court to question the
legality of the ordinances under which the said fees were being collected, which was done on the same date by filing the
complaint that gave rise to the present action. After hearing, the lower court dismissed the complaint for lack of merit.
The Society appealed to the Court of Appeals, which in turn certified the case to the Supreme Court for the reason that
the errors assigned involved only questions of law.

Issue:
Whether the City Treasurer may impose permit fee upon the religious organization before the latter may distribute and
sell bibles with the City of Manila.

Held:
Article III, section 1, clause (7) of the Constitution guarantees the freedom of religious profession and worship. Religion
has been spoken of as 'a profession of faith to an active power that binds and elevates man to its Creator. It has
reference to one's views of his relations to His Creator and to the obligations they impose of reverence to His being and
character, and obedience to His Will. The constitutional guaranty of the free exercise and enjoyment of religious
profession and worship carries with it the right to disseminate religious information. Any restraint of such right can only
be justified like other restraints of freedom of expression on the grounds that there is a clear and present danger of any
substantive evil which the State has the right to prevent.
A tax on the income of one who engages in religious activities is different from a tax on property used or employed in
connection with those activities. It is one thing to impose a tax on the income or property of a preacher. It is quite
another thing to exact a tax from him for the privilege of delivering a sermon. The power to tax the exercise of a
privilege is the power to control or suppress its enjoyment. The power to impose a license tax on the exercise of these
freedoms is indeed as potent as the power of censorship which this Court has repeatedly struck down. It is not a
nominal fee imposed as a regulatory measure to defray the expenses of policing the activities in question. It is in no way
apportioned. It is flat license tax levied and collected as a condition to the pursuit of activities whose enjoyment is
guaranteed by the constitutional liberties of press and religion and inevitably tends to suppress their exercise. Such is
the inherent vice and evil of a flat license tax.
Dissemination of religious information cannot be conditioned upon the approval of an official or manager. The right to
enjoy freedom of the press and religion occupies a preferred position as against the constitutional right of property
owners. Herein, Section 27 (e) of Commonwealth Act 466 (NIRC) -- which exempts corporations or associations
organized and operated exclusively for religious, charitable, or educational purposes, Provided however, That the
income of whatever kind and character from any of its properties, real or personal, or from any activity conducted for
profit, regardless of the disposition made of such income, shall be liable to the tax imposed under the Code -- does not
apply to the Society as its act of distributing and selling bibles, etc. is purely religious in nature. Ordinance 2529, as
amended, cannot as well be applied to the Society, for in doing so it would impair its free exercise and enjoyment of its
religious profession and worship as well as its rights of dissemination of religious beliefs. The fact that the price of the
bibles and other religious pamphlets are little higher than the actual cost of the same does not necessarily mean that it
is already engaged in the business or occupation of selling said “merchandise” for profit.
Lastly, Ordinance 3000 of the City of Manila, which requires the obtention of the Mayor's permit before any person can
engage in any of the businesses, trades or occupations enumerated therein, is not applicable to the Society, as its
business, trade or occupation is not particularly mentioned in Section 3 of the Ordinance, and the record does not show
that a permit is required therefor under existing laws and ordinances for the proper supervision and enforcement of
their provisions governing the sanitation, security and welfare of the public and the health of the employees engaged in
the business of the Society.
Iglesia ni Cristo v. Court of Appeals
G.R. No. 119673, July 26, 1996

FACTS
Several pre-taped episodes of the TV program “Ang Iglesia ni Cristo” of the religious group Iglesia ni Cristo (INC) were
rated “X” – i.e., not for public viewing – by the respondent Board of Review for Moving Pictures and Television (now
MTRCB). These TV programs allegedly “offend[ed] and constitute[d] an attack against other religions which is expressly
prohibited by law” because of petitioner INC’s controversial biblical interpretations and its “attacks” against contrary
religious beliefs.
Petitioner INC went to court to question the actions of respondent Board. The RTC ordered the respondent Board to
grant petitioner INC the necessary permit for its TV programs. But on appeal by the respondent Board, the CA reversed
the RTC. The CA ruled that: (1) the respondent Board has jurisdiction and power to review the TV program “Ang Iglesia ni
Cristo,” and (2) the respondent Board did not act with grave abuse of discretion when it denied permit for the exhibition
on TV of the three series of “Ang Iglesia ni Cristo” on the ground that the materials constitute an attack against another
religion. The CA also found the subject TV series “indecent, contrary to law and contrary to good customs.” Dissatisfied
with the CA decision, petitioner INC appealed to the Supreme Court.

ISSUES
(1) Does respondent Board have the power to review petitioner’s TV program?
(2) Assuming it has the power, did respondent Board gravely abuse its discretion when it prohibited the airing of
petitioner’s religious program?

RULING
[The Court voted 13-1 to REVERSE the CA insofar as the CA sustained the action of the respondent Board’s X-rating
petitioner’s TV Program Series Nos. 115, 119, and 121. It also voted 10-4 to AFFIRM the CA insofar as the CA it sustained
the jurisdiction of the respondent MTRCB to review petitioner’s TV program entitled “Ang Iglesia ni Cristo.”]
1. YES, respondent Board has the power to review petitioner’s TV program.

Petitioner contends that the term “television program” [in Sec. 3 of PD No. 1986 that the respondent Board has the
power to review and classify] should not include religious programs like its program “Ang Iglesia ni Cristo.” A contrary
interpretation, it is urged, will contravene section 5, Article III of the Constitution which guarantees that “no law shall be
made respecting an establishment of religion, or prohibiting the free exercise thereof. The free exercise and enjoyment
of religious profession and worship, without discrimination or preference, shall forever be allowed.”

[The Court however] reject petitioner’s postulate. Petitioner’s public broadcast on TV of its religious program brings it
out of the bosom of internal belief. Television is a medium that reaches even the eyes and ears of children. The Court
iterates the rule that the exercise of religious freedom can be regulated by the State when it will bring about the clear
and present danger of some substantive evil which the State is duty bound to prevent, i.e., serious detriment to the
more overriding interest of public health, public morals, or public welfare. A laissez faire policy on the exercise of
religion can be seductive to the liberal mind but history counsels the Court against its blind adoption as religion is and
continues to be a volatile area of concern in our country today. . . [T]he Court] shall continue to subject any act pinching
the space for the free exercise of religion to a heightened scrutiny but we shall not leave its rational exercise to the
irrationality of man. For when religion divides and its exercise destroys, the State should not stand still.

2. YES, respondent Board gravely abuse its discretion when it prohibited the airing of petitioner’s religious program.
[A]ny act that restrains speech is hobbled by the presumption of invalidity and should be greeted with furrowed brows.
It is the burden of the respondent Board to overthrow this presumption. If it fails to discharge this burden, its act of
censorship will be struck down. It failed in the case at bar.

The evidence shows that the respondent Board x-rated petitioners TV series for “attacking” either religions, especially
the Catholic Church. An examination of the evidence . . . will show that the so-called “attacks” are mere criticisms of
some of the deeply held dogmas and tenets of other religions. The videotapes were not viewed by the respondent court
as they were not presented as evidence. Yet they were considered by the respondent court as indecent, contrary to law
and good customs, hence, can be prohibited from public viewing under section 3(c) of PD 1986. This ruling clearly
suppresses petitioner's freedom of speech and interferes with its right to free exercise of religion. xxx.

The respondent Board may disagree with the criticisms of other religions by petitioner but that gives it no excuse to
interdict such criticisms, however, unclean they may be. Under our constitutional scheme, it is not the task of the State
to favor any religion by protecting it against an attack by another religion. . . In fine, respondent board cannot squelch
the speech of petitioner Iglesia ni Cristo simply because it attacks other religions, even if said religion happens to be the
most numerous church in our country. In a State where there ought to be no difference between the appearance and
the reality of freedom of religion, the remedy against bad theology is better theology. The bedrock of freedom of
religion is freedom of thought and it is best served by encouraging the marketplace of duelling ideas. When the luxury of
time permits, the marketplace of ideas demands that speech should be met by more speech for it is the spark of
opposite speech, the heat of colliding ideas that can fan the embers of truth.

In x-rating the TV program of the petitioner, the respondents failed to apply the clear and present danger rule. In
American Bible Society v. City of Manila, this Court held: “The constitutional guaranty of free exercise and enjoyment of
religious profession and worship carries with it the right to disseminate religious information. Any restraint of such right
can be justified like other restraints on freedom of expression on the ground that there is a clear and present danger of
any substantive evil which the State has the right to prevent.” In Victoriano vs. Elizalde Rope Workers Union, we further
ruled that “. . . it is only where it is unavoidably necessary to prevent an immediate and grave danger to the security and
welfare of the community that infringement of religious freedom may be justified, and only to the smallest extent
necessary to avoid the danger.”

The records show that the decision of the respondent Board, affirmed by the respondent appellate court, is completely
bereft of findings of facts to justify the conclusion that the subject video tapes constitute impermissible attacks against
another religion. There is no showing whatsoever of the type of harm the tapes will bring about especially the gravity
and imminence of the threatened harm. Prior restraint on speech, including religious speech, cannot be justified by
hypothetical fears but only by the showing of a substantive and imminent evil which has taken the life of a reality
already on ground.
Ebralinag vs. Division Superintendent of Schools of Cebu
GR 95770, 1 March 1993

Facts:
43 students of the Daanbantayan National High School, Agujo Elementary School, Calape Barangay National High School,
Pinamungajan Provincial High School, Tabuelan Central School, Canasojan Elementary School, Liboron Elementary
School, Tagaytay Primary School, San Juan Primary School and Northern Central Elementary School of San Fernando,
Cebu, were expelled (23 October 1990) upon order of then Acting Division Superintendent Marcelo Bacalso. Said
students in the towns of Daan Bantayan, Pinamungajan, Carcar, and Taburan, Cebu province (GR 95770, Ebralinag vs.
Division Superintendent) were expelled for refusing to salute the flag, sing the national anthem and recite the patriotic
pledge as required by Republic Act 1265 (11 July 1955), and by Department Order 8 dated 21 July 1955 of the
Department of Education, Culture and Sports (DECS) making the flag ceremony compulsory in all educational
institutions.
This prompted some Jehovah's Witnesses in Cebu to appeal to the Secretary of Education Isidro Cariño but the latter did
not answer their letter. 25 students who were similarly expelled (high school and grade school students enrolled in
public schools in Asturias, Cebu [GR 95887, Amolo vs. Director Superintendent]) because Dr. Pablo Antopina, who
succeeded Susana Cabahug as Division Superintendent of Schools, would not recall the expulsion orders of his
predecessor. Instead, he verbally caused the expulsion of some more children of Jehovah's Witnesses. On 31 October
1990, the students and their parents filed the Special civil actions for Mandamus, Certiorari and Prohibition alleging that
the Division Superintendent of Schools of Cebu, et. al. acted without or in excess of their jurisdiction and with grave
abuse of discretion in ordering their expulsion without prior notice and hearing, hence, in violation of their right to due
process, their right to free public education, and their right to freedom of speech, religion and worship. Jehovah's
Witnesses admittedly teach their children not to salute the flag, sing the national anthem, and recite the patriotic pledge
for they believe that those are "acts of worship" or "religious devotion" which they "cannot conscientiously give to
anyone or anything except God." They feel bound by the Bible's command to "guard ourselves from idols — 1 John
5:21." They consider the flag as an image or idol representing the State. They think the action of the local authorities in
compelling the flag salute and pledge transcends constitutional limitations on the State's power and invades the sphere
of the intellect and spirit which the Constitution protects against official control.
They stress, however, that while they do not take part in the compulsory flag ceremony, they do not engage in "external
acts" or behavior that would offend their countrymen who believe in expressing their love of country through the
observance of the flag ceremony. They quietly stand at attention during the flag ceremony to show their respect for the
right of those who choose to participate in the solemn proceedings. Since they do not engage in disruptive behavior,
there is no warrant for their expulsion. On 27 November 1990, the Court issued a temporary restraining order and a writ
of preliminary mandatory injunction commanding the Division Superintendent to immediately readmit the students to
their respective classes until further orders from the Court. The Court also ordered the Secretary of Education and Cebu
District Supervisor Manuel F. Biongcog to be impleaded as respondents in the cases.

Issue:
Whether the students, who belong to the Jehovah’s Witness sect, should be expelled (following the holding in the case
of Gerona) for not saluting the flag in accordance with RA 1265.

Held:
Religious freedom is a fundamental right which is entitled to the highest priority and the amplest protection among
human rights, for it involves the relationship of man to his Creator. The right to religious profession and worship has a
two-fold aspect, vis., freedom to believe and freedom to act on one's belief. The first is absolute as long as the belief is
confined within the realm of thought. The second is subject to regulation where the belief is translated into external acts
that affect the public welfare. The sole justification for a prior restraint or limitation on the exercise of religious freedom
is the existence of a grave and present danger of a character both grave and imminent, of a serious evil to public safety,
public morals, public health or any other legitimate public interest, that the State has a right (and duty) to prevent.
Absent such a threat to public safety, the expulsion of the students from the schools is not justified. By exempting the
Jehovah's Witnesses from saluting the flag, singing the national anthem and reciting the patriotic pledge, this religious
which admittedly comprises a "small portion of the school population" will not shake up our part of the globe and
suddenly produce a nation "untaught and uninculcated in and unimbued with reverence for the flag, patriotism, love of
country and admiration for national heroes." After all, what the students seek only is exemption from the flag ceremony,
not exclusion from the public schools where they may study the Constitution, the democratic way of life and form of
government, and learn not only the arts, science, Philippine history and culture but also receive training for a vocation or
profession and be taught the virtues of "patriotism, respect for human rights, appreciation for national heroes, the rights
and duties of citizenship, and moral and spiritual values as part of the curricula. Expelling or banning the students from
Philippine schools will bring about the very situation that this Court had feared in Gerona. Forcing a small religious
group, through the iron hand of the law, to participate in a ceremony that violates their religious beliefs, will hardly be
conducive to love of country or respect for duly constituted authorities.
Moreover, the expulsion of members of Jehovah's Witnesses from the schools where they are enrolled will violate their
right as Philippine citizens, under the 1987 Constitution, to receive free education, for it is the duty of the State to
"protect and promote the right of all citizens to quality education and to make such education accessible to all." While it
is certain that not every conscience can be accommodated by all the laws of the land; when general laws conflict with
scruples of conscience, exemptions ought to be granted unless some "compelling state interests" intervenes.
Exemptions may be accorded to the Jehovah's Witnesses with regard to the observance of the flag ceremony out of
respect for their religious beliefs, however "bizarre" those beliefs may seem to others. Nevertheless, their right not to
participate in the flag ceremony does not give them a right to disrupt such patriotic exercises. While the highest regard
must be afforded their right to the exercise of their religion, "this should not be taken to mean that school authorities
are powerless to discipline them" if they should commit breaches of the peace by actions that offend the sensibilities,
both religious and patriotic, of other persons. If they quietly stand at attention during the flag ceremony while their
classmates and teachers salute the flag, sing the national anthem and recite the patriotic pledge, such conduct cannot
possibly disturb the peace, or pose "a grave and present danger of a serious evil to public safety, public morals, public
health or any other legitimate public interest that the State has a right (and duty) to prevent." Thus, although the Court
upholds the students' right under our Constitution to refuse to salute the Philippine flag on account of their religious
beliefs, it hopes, nevertheless, that another foreign invasion of our country will not be necessary in order for our
countrymen to appreciate and cherish the Philippine flag.
Estrada vs. Escritor
AM P-02-1651, 4 August 2003

Facts:
In a sworn letter-complaint dated 27 July 2000, Alejandro Estrada wrote to Judge Jose F. Caoibes, Jr., presiding judge of
Branch 253, Regional Trial Court of Las Piñas City, requesting for an investigation of rumors that Soledad Escritor, court
interpreter in said court, is living with a man not her husband. They allegedly have a child of 18 to 20 years old. Estrada is
not personally related either to Escritor or her partner and is a resident not of Las Piñas City but of Bacoor, Cavite.
Nevertheless, he filed the charge against Escritor as he believes that she is committing an immoral act that tarnishes the
image of the court, thus she should not be allowed to remain employed therein as it might appear that the court
condones her act. Judge Caoibes referred the letter to Escritor who stated that "there is no truth as to the veracity of the
allegation" and challenged Estrada to "appear in the open and prove his allegation in the proper forum." Judge Caoibes
set a preliminary conference on 12 October 2000. Escritor moved for the inhibition of Judge Caoibes from hearing her
case to avoid suspicion and bias as she previously filed an administrative complaint against him and said case was still
pending in the Office of the Court Administrator (OCA). Escritor's motion was denied.

The preliminary conference proceeded with both Estrada and Escritor in attendance. Estrada confirmed that he filed the
letter-complaint for immorality against Escritor because in his frequent visits to the Hall of Justice of Las Piñas City, he
learned from conversations therein that Escritor was living with a man not her husband and that she had an 18-20 year
old son by this man. This prompted him to write to Judge Caoibes as he believed that employees of the judiciary should
be respectable and Escritor's live-in arrangement did not command respect. Escritor, on the other hand, testified that
when she entered the judiciary in 1999, she was already a widow, her husband having died in 1998. She admitted that
she has been living with Luciano Quilapio, Jr. without the benefit of marriage for 20 years and that they have a son. But
as a member of the religious sect known as the Jehovah's Witnesses and the Watch Tower and Bible Tract Society, their
conjugal arrangement is in conformity with their religious beliefs. In fact, after 10 years of living together, she executed
on 28 July 1991 a "Declaration of Pledging Faithfulness." Escritor's partner, Quilapio, executed a similar pledge on the
same day. Both pledges were executed in Atimonan, Quezon and signed by 3 witnesses. At the time Escritor executed
her pledge, her husband was still alive but living with another woman. Quilapio was likewise married at that time, but
had been separated in fact from his wife. During her testimony, Escritor volunteered to present members of her
congregation to confirm the truthfulness of their "Declarations of Pledging Faithfulness," but Judge Caoibes deemed it
unnecessary and considered her identification of her signature and the signature of Quilapio sufficient authentication of
the documents. Judge Caoibes endorsed the complaint to Executive Judge Manuel B. Fernandez, Jr., who, in turn,
endorsed the same to Court Administrator Alfredo L. Benipayo.

On 17 July 2001, the Court, upon recommendation of Acting Court Administrator Zenaida N. Elepaño, directed Escritor
to comment on the charge against her. In her comment, Escritor reiterated her religious congregation's approval of her
conjugal arrangement with Quilapio. Deputy Court Administrator Christopher O. Lock recommended that the case be
referred to Executive Judge Bonifacio Sanz Maceda, RTC Branch 255, Las Piñas City for investigation, report and
recommendation. In his Report and Recommendation, investigating judge Maceda found Escritor's factual allegations
credible as they were supported by testimonial and documentary evidence. He also noted that "(b)y strict Catholic
standards, the live-in relationship of respondent with her mate should fall within the definition of immoral conduct, to
wit: 'that which is willful, flagrant, or shameless, and which shows a moral indifference to the opinion of the good and
respectable members of the community'" He pointed out, however, that "the more relevant question is whether or not
to exact from Escritor, a member of 'Jehovah's Witnesses,' the strict moral standards of the Catholic faith in determining
her administrative responsibility in the case at bar." The investigating judge acknowledged that "religious freedom is a
fundamental right which is entitled to the highest priority and the amplest protection among human rights, for it
involves the relationship of man to his Creator and thereby recommended the dismissal of the complaint against
Escritor.
After considering the Report and Recommendation of Executive Judge Maceda, the Office of the Court Administrator,
through Deputy Court Administrator (DCA) Lock and with the approval of Court Administrator Presbitero Velasco,
concurred with the factual findings of Judge Maceda but departed from his recommendation to dismiss the complaint.
DCA Lock stressed that although Escritor had become capacitated to marry by the time she joined the judiciary as her
husband had died a year before, "it is due to her relationship with a married man, voluntarily carried on, that
respondent may still be subject to disciplinary action." Considering the ruling of the Court in Dicdican v. Fernan, et al.
that "court personnel have been enjoined to adhere to the exacting standards of morality and decency in their
professional and private conduct in order to preserve the good name and integrity of the court of justice," DCA Lock
found Escritor's defense of freedom of religion unavailing to warrant dismissal of the charge of immorality. Accordingly,
he recommended that Escritor be found guilty of immorality and that she be penalized with suspension of 6 months and
one day without pay with a warning that a repetition of a similar act will be dealt with more severely in accordance with
the Civil Service Rules.

Issue:
Whether Escritor's right to religious freedom should carve out an exception from the prevailing jurisprudence on illicit
relations for which government employees are held administratively liable.

Held:
The case being one of first impression, the claim of religious freedom is subjected to the "compelling state interest" test
from a benevolent neutrality stance - i.e. entertaining the possibility that Escritor's claim to religious freedom would
warrant carving out an exception from the Civil Service Law; necessarily, her defense of religious freedom will be
unavailing should the government succeed in demonstrating a more compelling state interest. In applying the test, the
first inquiry is whether Escritor's right to religious freedom has been burdened. There is no doubt that choosing between
keeping her employment and abandoning her religious belief and practice and family on the one hand, and giving up her
employment and keeping her religious practice and family on the other hand, puts a burden on her free exercise of
religion. The burden on Escritor is even greater as the price she has to pay for her employment is not only her religious
precept but also her family which, by the Declaration Pledging Faithfulness, stands "honorable before God and men."

The second step is to ascertain Escritor's sincerity in her religious belief. The accused appears to be sincere in her
religious belief and practice and is not merely using the "Declaration of Pledging Faithfulness" to avoid punishment for
immorality. She did not secure the Declaration only after entering the judiciary where the moral standards are strict and
defined, much less only after an administrative case for immorality was filed against her. The Declaration was issued to
her by her congregation after 10 years of living together with her partner, Quilapio, and 10 years before she entered the
judiciary. In any event, even if the Court deems sufficient Escritor's evidence on the sincerity of her religious belief and
its centrality in her faith, the case cannot still be decided using the "compelling state interest" test. The case is one of
first impression, thus the parties were not aware of the burdens of proof they should discharge in the Court's use of the
"compelling state interest" test.

To properly settle the issue, the government should be given the opportunity to demonstrate the compelling state
interest it seeks to uphold in opposing Escritor's stance that her conjugal arrangement is not immoral and punishable as
it comes within the scope of free exercise protection. Should the Court prohibit and punish her conduct where it is
protected by the Free Exercise Clause, the Court's action would be an unconstitutional encroachment of her right to
religious freedom. The Court cannot therefore simply take a passing look at Escritor's claim of religious freedom, but
must instead apply the "compelling state interest" test. The government must be heard on the issue as it has not been
given an opportunity to discharge its burden of demonstrating the state's compelling interest which can override
respondent's religious belief and practice. Thus, the case was remanded to the Office of the Court Administrator.
ANONYMOUS, complainant, vs. MA. VICTORIA P. RADAM ( Utility Worker, Office of the Clerk of Court,
Regional Trial Court of Alaminos City, Pangasinan), respondent.
A.M. No. P-07-2333, December 19, 2007

Facts:
In an anonymous letter-complaint dated September 30, 2005,
1.respondent Ma. Victoria Radam, utility worker in the Office of the Clerk of Court of the Regional Trial Court of
Alaminos City in Pangasinan, was charged with immorality. The unnamed complainant alleged that respondent was
unmarried but got pregnant and gave birth sometime in October 2005.
2. The complainant claimed that respondent's behavior tainted the image of the judiciary.

On March 1, 2006, respondent submitted a letter addressed to the Honorable Court Administrator, thru the
undersigned, duly subscribed and sworn to before the Clerk of Court VI of the Court, alleging among others, the
following:
1) She admitted that she is single/unmarried, and indeed she was pregnant and actually gave birth to a baby boy named
Christian Jeon Radam on 03 November 2005 at the Western Pangasinan District Hospital, Alaminos City;
2) The reason why she did not yet marry the father of her child Christian Jeon was that she and the child's father have
pending application[s] [to migrate to Canada] as in fact they have mutual plan to remain unmarried [and]
3) Nevertheless, she expressed her remorse and promised not to commit the same mistake and indiscretion in the
future.

Further investigation reveal[ed] the following:


1) That respondent was appointed as Utility Worker on September 4, 2000;
2) The father of Christian Jeon Radam is unknown, as shown by the child's Certificate of Live Birth;
3) It was verbally admitted by the respondent that she had given birth to two (2) other children before Christian Jeon,
but they were conceived and born while respondent was working abroad and before she was employed
in the [Office of the Clerk of Court of the Regional Trial Court of] Alaminos City.

Premises considered, respondent MA. VICTORIA RADAM found GUILTY of IMMORAL CONDUCT or ACT UNBECOMING A
COURT EMPLOYEE. A suspension of one (1) month or a fine of Php5,000.00 is respectfully recommended, with warning
that a repetition of the same or similar act in the future will be dealt with more severely.

Issue:
Whether or not respondent be absolved of the charge of immorality because her alleged misconduct (that is, giving birth
out of wedlock) did not affect the character and nature of her position as a utility worker.

Held:
For purposes of determining administrative responsibility, giving birth out of wedlock is not per se immoral under civil
service laws. For such conduct to warrant disciplinary action, the same must be "grossly immoral," that is, it must be so
corrupt and false as to constitute a criminal act or so unprincipled as to be reprehensible to a high degree.

In Estrada v. Escritor, it was emphasized that in determining whether the acts complained of constitute "disgraceful and
immoral behavior" under civil service laws, the distinction between public and secular morality on the one hand, and
religious morality, on the other should be kept in mind. The distinction between public and secular morality as
expressed — albeit not exclusively — in the law, on the one hand, and religious morality, on the other, is important
because the jurisdiction of the Court extends only to public and secular morality. Thus, government action, including its
proscription of immorality as expressed in criminal law like adultery or concubinage, must have a secular purpose.
For a particular conduct to constitute "disgraceful and immoral" behavior under civil service laws, it must be regulated
on account of the concerns of public and secular morality. It cannot be judged based on personal bias, specifically those
colored by particular mores. Nor should it be grounded on "cultural" values not convincingly demonstrated to have been
recognized in the realm of public policy expressed in the Constitution and the laws. 16 At the same time, the
constitutionally guaranteed rights (such as the right to privacy) should be observed to the extent that they protect
behavior that may be frowned upon by the majority.

In this case, it was not disputed that, like respondent, the father of her child was unmarried. Therefore, respondent
cannot be held liable for disgraceful and immoral conduct simply because she gave birth to the child Christian Jeon out
of wedlock. Respondent was indicted only for alleged immorality for giving birth out of wedlock. It was the only charge
of which she was informed. Judge Abella's investigation focused solely on that matter. Thus, the recommendation of the
OCA that she be held administratively liable in connection with an entry in the birth certificate of Christian Jeon came
like a thief in the night. It was unwarranted. Respondent was neither confronted with it nor given the chance to explain
it. To hold her liable for a totally different charge of which she was totally unaware will violate her right to due process.
The essence of due process in an administrative proceeding is the opportunity to explain one's side, whether written or
verbal. This presupposes that one has been previously apprised of the accusation against him or her. Here, respondent
was deprived of both with regard to her alleged unbecoming conduct in relation to a certain statement in the birth
certificate of her child.

WHEREFORE, the administrative complaint against respondent Ma. Victoria P. Radam is hereby DISMISSED. She is,
however, strongly advised to be more circumspect in her personal and official actuations in the future.
JAMES M. IMBONG vs. HON. PAQUITO N. OCHOA, JR.
G.R. Nos. 204819, April 8, 2014

KEY TAKE-AWAY: The Reproductive Health Law is a consolidation and enhancement of existing reproductive laws. It
seeks to enhance the population control program of the government in order to promote public welfare. However,
when coercive measures are found within the law, provisions must be removed or altered in order to ensure that it does
not defy the Constitution by infringing on the rights of the people.

FACTS
• Petition: to declare provisions of Republic Act No. 10354 as unconstitutional
• Factual Antecedents
 December 21, 2012: Congress enacted RA No. 10354 also known as the Responsible Parenthood and
Reproductive Health Act of 2012 (RH LAW)
 The president’s imprimatur and support for the said law lead to a range of petitions against the law leading to
iuris controversy in court. Petitions for certiorari and prohibition were placed by numerous parties. All in all, 14
petitions and 2 petitions-in-intervention were filed.
 March 15, 2013: the RH-IRR or enforcement of the law took place
 March 19, 2013: After deliberating the issues and arguments raised, the court issued Status Quo Ante Order
(SQAO) which lead to a 120 day halt on the implementation of the legislation
 Due to further arguments and debates from opposing parties, the SQAO was extended until further orders of
the court last July 16, 2013
• Statute Involved:
 Republic Act 10354, “The Responsible Parenthood and Reproductive Health Act of 2012”
 Position of Petitioner:
o Petitioners claim that the provisions of RA 10354 are unconstitutional as they violate the rights to life, to
health, to freedom of expression and speech, to the privacy of families, to academic freedom, to due
process of law, to equal protection, and against involuntary servitude. They also intrude on the autonomy of
local governments and the ARMM, and violate natural law. Furthermore, they claim that Congress’
delegation of authority to the FDA in determining which should be included in the EDL is invalid.
 Position of Respondent
 There is no actual case or controversy and, therefore, the issues are not yet ripe for judicial determination
 Some petitioners lack standing to question the RH Law
 The petitions are essentially petitions for declaratory relief over which the Court has no original jurisdiction.

ISSUES
 Procedural
o Whether or not the Court may exercise its power of judicial review
o Whether or not there is an actual case or controversy
o Whether the Court may apply facial challenge
o Whether or not the petitions are praying for declaratory relief
o Whether the petitions violate the One Subject/One Title Rule
 Substantive
o Whether or not the RH Law is unconstitutional on the grounds that it violates
 Right to Life
 Right to Health
 Freedom of Religion and the Right to Free Speech
 The Family
 Freedom of Expression and Academic Freedom
 Due Process
 Equal Protection
 Involuntary Servitude
 Autonomy of Local Governments/ARMM
 Natural Law
o Whether or not Congress’ delegation of authority to the FDA in determining which should be included in the
EDL is valid
HELD
 Procedural
o Whether or not the court may exercise its power of judicial review - YES
 While the Court may not pass upon questions of wisdom, justice or expediency of the RH Law, it may do so
where an attendant unconstitutionality or grave abuse of discretion results. The following requisites for
judicial review were met: (a) there mustbe an actual case or controversy; (b) the petitioners must possess
locus standi; ( c) the question of constitutionality must be raised at the earliest opportunity; and ( d) the
issue of constitutionality must be the lis mota of the case
o Whether or not there is an actual case or controversy – YES
 Considering that the RH Law and its implementing rules have already taken effect and that budgetary
measures to carry out the law have already been passed, it is evident that the subject petitions present a
justiciable controversy. As stated earlier, when an action of the legislative branch is seriously alleged to have
infringed the Constitution, it not only becomes a right, but also a duty of the Judiciary to settle the dispute.
 Moreover, the petitioners have shown that the case is so because medical practitioners or medical providers
are in danger of being criminally prosecuted under the RH Law for vague violations thereof, particularly
public health officers who are threatened to be dismissed from the service with forfeiture of retirement
and other benefits.
o Whether the Court may apply facial challenge – YES
 The scope of application of facial challenges extends to the regulation of free speech, but also those
involving religious freedom, and other fundamental rights.
 Consequently, considering that the foregoing petitions have seriously alleged that the constitutional human
rights to life, speech and religion and other fundamental rights mentioned above have been violated by the
assailed legislation, the Court has authority to take cognizance of these kindred petitions and to determine if
the RH Law can indeed pass constitutional scrutiny.
o Whether or not Locus Standi applies – YES
 Regardless of whether the petitioners are directly injured of affected by the RH Law or not, the Court leans
on the doctrine that "the rule on standing is a matter of procedure, hence, can be relaxed for non-traditional
plaintiffs like ordinary citizens, taxpayers, and legislators when the public interest so requires, such as when
the matter is of transcendental importance, of overreaching significance to society, or of paramount public
interest." The RH Law falls under transcendental importance as it drastically affects the constitutional
provisions on the right to life and health, the freedom of religion and expression and other constitutional
rights.
o Whether or not the petitions are praying for declaratory relief - YES
 Most of the petitions are praying for injunctive reliefs, not declaratory reliefs, and so the Court would just
consider them as petitions for prohibition under Rule 65, over which it has original jurisdiction. Where the
case has far-reaching implications and prays for injunctive reliefs, the Court may consider them as petitions
for prohibition under Rule 65.
o Whether the petitions violate the One Subject/One Title Rule – NO
 In a textual analysis of the various provisions of the law, both "reproductive health" and "responsible
parenthood" are interrelated and germane to the overriding objective to control the population growth.
Thus, the Court finds no reason to believe that Congress had the intention to deceive the public regarding
the contents of the said law.
 Substantive
o Whether or not the RH Law is unconstitutional on the grounds that it violates
 Right to Life – NO
 Constitution intended that 1.) conception to refer to the time of fertilization and 2.) the protection of
the unborn upon said fertilization
 Not all contraceptives are to be banned (only those that kill a fertilized ovum)
 Contraceptives that prevent union of sperm and egg are thus permissible
 It is the intended by the framers of the 1987 Constitution to prevent the enacting of a law that legalizes
abortion.
 RH law prohibits abortion
 RH law recognizes that abortion is a crime
 RH law prohibits abortifacients
 Right to Health - NO
 With the provisions of RA 4729 still in place, the status quo on the sale of contraceptives is maintained
and the Court believes that there are adequate measures that ensure that the public has access to
contraceptives that have been determined safe following testing, evaluation, and approval by the FDA
 Freedom of Religion and the Right to Free Speech – NO and YES
 RH law does not violate guarantee of religious freedom via the state-sponsored procurement of
contraceptives, which contravene the religious beliefs of the people including the petitioners. This is
because in doing so, the state would be adhering to one religions, making a de facto state religion which
is contrary to religious freedom.
 The separation of Church and State shall be inviolable
 There limits to the exercise of religious freedom (compelling state interest test)
 Benevolent neutrality
 RH law does not violate the guarantee of religious freedom by requiring would-be spouses, as a
condition for the issuance of a marriage license, to attend a seminar on parenthood, family planning,
breastfeeding and infant nutrition (sec.7, 23, 24)
 However, RH Law violates the guarantee of religious freedom by compelling medical health
practitioners, hospitals, and health care providers, under pain of penalty, to refer patients to other
institutions despite their conscientious objections
 The Family - YES
 Section 23(a)(2)(i) of the RH Law, which needs only the consent of the spouse undergoing the provision
in order to undergo reproductive procedures intrudes into martial privacy and autonomy and goes
against the constitutional safeguards for the family as the basic social institution. Not only that, but the
exclusion of parental consent in cases where a minor undergoing a procedure is already a parent or has
had a miscarriage (Section 7 of the RH Law) is also anti-family and violates Article II, Section 12 of the
Constitution, which declares that the rearing of children by parents is a natural right.
 Freedom of Expression and Academic Freedom – UNDECIDED
 The court decided that making a ruling on Section 14 of the RH Law, which mandates the State to
provide Age-and Development-Appropriate Reproductive Health Education, is premature. The
Department of Education has not yet created a curriculum on age-appropriate reproductive health
education, thus the constitutionality of the specifics in such a curriculum still cannot be determined. The
exclusion of private educational institutions from the mandatory RH education program under Section
14 is valid. There is a need to recognize the academic freedom of private educational institutions
especially with respect to religious instruction and to consider their sensitivity towards the teaching of
reproductive health education.
 Due Process - NO
 The definitions of several terms pinpointed by the petitioners in the RH Law are not vague.
 Private health care institution = private health care service provider.
 “service” and “methods” are also broad enough to include giving information and performing
medical procedures, so hospitals run by religious groups can be exempted.
 “incorrect information” connotes a sense of malice and ill motive to mislead the public.
 Equal Protection - NO
 It is pursuant to Section 11, Article XIII of the Constitution, which states that the State shall prioritize the
needs of the underprivileged, sick elderly, disabled, women, and children and that it shall endeavor to
provide medical care to paupers.
 Involuntary Servitude - NO
 The State has the power to regulate the practice of medicine in order to ensure the welfare of the
public. Not only that, but Section 17 only encourages private and non-government RH service providers
to give pro bono service; they do not incur penalties if they refuse. Conscientious objects are exempt if
their religious beliefs do not allow them to provide the said services.
 Autonomy of Local Governments/ARMM – NO
 The RH Law does not infringe upon the autonomy of local governments. Under paragraph (c) of Section
17, unless a local government unit (LGU) is particularly designated as the implementing agency, it has no
power over a program for which funding has been provided by the national government under the
annual General Appropriations Act, even if the program involves the delivery of basic services within the
jurisdiction of the LGUs. Not only that, but LGUs are merely encouraged and not compelled to provide
RH services. Provision of these services are not mandatory. Lastly, Article III, Sections 6, 10, and 11 of RA
9054 deor the Organic Act of the ARMM merely outlines the powers that may be exercised by the
regional government and does not indicate the State’s abdication to create laws in the name of public
welfare.
 Natural Law – disregarded
 Natural law, according to the Court, is not recognized as proper legal basis for making decisions
o Whether or not Congress’ delegation of authority to the FDA in determining which should be included in the
EDL is valid- YES
 Under RA 3720, the FDA, being the primary and sole premiere and only agency that ensures the safety of
food and medicines available to the public, has the power and competency to evaluate, register and cover
health services and methods

Final Ruling
o Petitions partially granted. The RA 10354 is declared constitutional, and Status Quo Ante Order lifted with respect to
provisions of RA 10354 that have been declared as constitutional. However, the following provisions and their
corresponding provisions in the RH-IRR have been declared unconstitutional:
 Section 7 and the corresponding provision in the RH-IRR insofar as they: a) require private health facilities and
non-maternity specialty hospitals and hospitals owned and operated by a religious group to refer patients, not in
an emergency or life-threatening case, as defined under Republic Act No. 8344, to another health facility which
is conveniently accessible; and b) allow minor-parents or minors who have suffered a miscarriage access to
modem methods of family planning without written consent from their parents or guardian/s;
 Section 23(a)(l) and the corresponding provision in the RH-IRR, particularly Section 5 .24 thereof, insofar as
they punish any healthcare service provider who fails and or refuses to disseminate information regarding
programs and services on reproductive health regardless of his or her religious beliefs.
 Section 23(a)(2)(i) and the corresponding provision in the RH-IRR insofar as they allow a married individual, not
in an emergency or life-threatening case, as defined under Republic Act No. 8344, to undergo reproductive
health procedures without the consent of the spouse;
 Section 23(a)(2)(ii) and the corresponding provision in the RH-IRR insofar as they limit the requirement of
parental consent only to elective surgical procedures.
 Section 23(a)(3) and the corresponding provision in the RH-IRR, particularly Section 5.24 thereof, insofar as
they punish any healthcare service provider who fails and/or refuses to refer a patient not in an emergency or
life-threatening case, as defined under Republic Act No. 8344, to another health care service provider within the
same facility or one which is conveniently accessible regardless of his or her religious beliefs;
 Section 23(b) and the corresponding provision in the RH-IRR, particularly Section 5 .24 thereof, insofar as they
punish any public officer who refuses to support reproductive health programs or shall do any act that hinders
the full implementation of a reproductive health program, regardless of his or her religious beliefs;
 Section 17 and the corresponding provision in the RH-IRR regarding the rendering of pro bona reproductive
health service in so far as they affect the conscientious objector in securing PhilHealth accreditation;
 Section 3.0l(a) and Section 3.01 G) of the RH-IRR, which added the qualifier "primarily" in defining
abortifacients and contraceptives, as they are ultra vires and, therefore, null and void for contravening Section
4(a) of the RH Law and violating Section 12, Article II of the Constitution.

You might also like